d119

Réussis tes devoirs et examens dès maintenant avec Quizwiz!

Lactational mastitis usually occurs within first 3 mo of breastfeeding

3. Pharmacologic treatment for more severe symptoms lasting >24 hr; antibiotics for 10 to 14 days to prevent relapse; if rapid response can decrease to 5 to 7 days a) Dicloxacillin 500 mg qid b) Cephalexin 500 mg qid c) Clindamycin 300 mg tid

An advanced practice registered nurse (APRN) examines a six-year-old child who has cerebral palsy and notes uncontrollable, jerky movements of the child's toes and fingers. Which problem is the source of this child's movements? Chorea Ballismus Rigidity Dystonia

A

The Family nurse practitioner is preparing to conduct a well child assessment of an 8-year-old child. How will the nurse practitioner begin the exam? 1/1 Ask the child about school, friends, home activities, and sports Discuss the purpose of the visit and explain the procedures that will be performed Offer age-appropriate information about usual developmental tasks Provide information about healthy nutrition and physical activities

A

when can a child stop using a booster seat in the car? 1/1 when a child is at least 4 feet 9 inches tall when the child is at least five feet tall and the child is at least 4 feet tall when the child is at least 3 ft tall

A

Glucose-6-Phosphate Dehydrogenase Deficiency

A drug-induced hemolytic anemia can be caused by genetic deficiency of the G6PD enzyme in the RBC. Symptoms are generally associated with infections or exposure to oxidant metabolites of certain drugs that cause the precipitation of Hgb, injury to the red cells, and rapid hemolysis.

Achondroplasia

A form of human dwarfism caused by a single dominant allele; the homozygous condition is lethal

Diastema

A space between two teeth

A 2-year-old child has an acute diarrheal illness. The child is afebrile and, with oral rehydration measures, has remained well hydrated. The parent asks what can be done to help shorten the course of this illness. What will the Family nurse practitioner recommend? 1/1 Clear liquids only Lactobacillus Loperamide Peppermint oil

B

A 3-year-old child has head lice. What will the initial treatment recommendation be to treat this child? 1/1 Lindane Permethrin Pyrethrin Spinosad

B

what is an appropriate treatment for a child with roseola? 1/1 antiviral medications, fluids, and rest antipyretics, rest and hydration antibiotics, hydration, and rest hospitalization, antipyretics, and IV fluid replacement

B Feedback the primary treatment for roseola is supportive, including control a fever, adequate rest, and hydration. No preventive measure or immunization is available

An advanced practice registered nurse (APRN) is assessing a patient who has experienced mood instability for 10 years. The patient mentions noticing his feet and hands appear to be enlarged. Which specialty provider should the APRN recommend to assist this patient? Cardiology Psychiatry Endocrinology Podiatry

C

The Family nurse practitioner is examining a 6-year-old child who attends first grade. The child reports "hating" school. The parent states that the child pretends to be sick frequently in order to stay home from school. To further assess this situation, the nurse practitioner will first ask the child: 1/1 about school performance and grades. why school is so distressing. to name one or two friends whether bullying is taking place.

C

The parent of an 8-year-old child tells the Family nurse practitioner that the child has begun to ask questions about why a schoolmate has "2 daddies" and wonders how to talk to the child about this. What will the nurse practitioner recommend? 1/1 Beginning a discussion about different types of sexual relationships and same-sex partners Discussing the issue with the child in terms of the parent's religious values and norms Explaining that not all families are the same and what is most important is that they love and care for their children Telling the child that some adult relationships are complicated and will be understood when the child is older

C

A 21-year-old male presents for dysuria. He admits to recent unprotected sex with possible exposure to a sexually transmitted infection (STI). The in-office dipstick urinalysis is positive for leukocytes and nitrites. You ordered a urine culture and STI testing, but these are sent out and the results will not be available for two to three days. What is the best class of antibiotics for treatment until the results are back? Penicillins Lincomycins Cephalosporins Fluoroquinolones

Cephalosporins are used to treat uncomplicated UTIs and do not cover chlamydia, which is the most common STI. Initial treatment with a fluoroquinolone is recommended, which can be changed if indicated by the STI and urine culture results.

An adolescent patient is being evaluated by the family nurse practitioner for knee pain. The patient is active in sports in his school but can work on those specific injury to the knee. On exam, the family nurse practitioner finds unilateral swelling of the anterior aspect of the tibial tubercle, which is tender. What is the most likely diagnosis? 0/1 stress fracture patellar dislocation Osgood-Schlatters disease Neuman's syndrome

Correct answer Osgood-Schlatters disease Feedback Osgood-Schlatters disease is common in late childhood and Adolescence. The risk for disease increases in patients who are involved in strenuous activity, especially involving the quadriceps muscle. The usual treatment is non-steroidal anti-inflammatory drugs and the rest.

the family nurse practitioner suspects acute pyelonephritis in an older child. What clinical finding is not found? 0/1 flank pain hematuria fever frequency and urgency

Correct answer hematuria Feedback hematuria is not a usual clinical finding. Fever may be the only presenting complaint in the child. Older children are more likely to prevent with flank pain, dysuria, frequency, urgency, and incontinence.

what must children with juvenile rheumatoid arthritis be screened regularly for? 0/1 ulcerative colitis iridocyclitis diabetes mellitus adrenal insufficiency

Correct answer iridocyclitis Feedback development of iridocyclitis maybe Insidious in a symptomatic and, if untreated, may cause blindness. Although children may develop any of the other listed diseases, there is no correlation with JRA.

a child with a sandpaper textured rash probably has: 0/1 rubeola strept infection varicella roseolo

Correct answer strept infection Feedback streptococcal infections can present as a sandpaper textured rash that initially is felt on the trunk. Rubeola or measles produces a blanching erythematous brick-red maculopapular rash that begins on the back of the neck and spreads around the trunk and then the extremities. Varicella infection produces the classic crops of eruptions on the trunk that spread to the face. The rash is maculopapular initially and then crusts. Roseola produces a generalized maculopapular rash preceded by three days of high fever.

Benign prostatic hyperplasia is a risk factor for prostate cancer. True False

Correct! Benign prostatic hyperplasia does not increase the chance of prostate cancer. Prostate enlargement is a normal part of aging.

What is the most important dose-limiting factor when prescribing acetaminophen with hydrocodone? Acetaminophen dose Gastrointestinal side effects Pruritis symptoms Urinary retention

Correct! Exceeding the recommended dose of acetaminophen can lead to liver toxicity. The other options list potential, non-life-threatening side effects.

A prostatic massage to express prostatic secretions is indicated if acute bacterial prostatitis is suspected. True False

Correct! If acute bacterial prostatitis is suspected, prostatic massage should be avoided to minimize risk of bacteremia.

You are performing a well-baby examination on a two-week-old infant. The parent is concerned that the infant sleeps too much. You ask the parent to keep a sleep log. What amount of sleep per day is optimal for this infant? 10-12 hours 12-15 hours 18-20 hours 15-18 hours

Correct! Newborns should sleep a total of 15 to 18 hours per day.

True or False Prophylactic oral oseltamivir is recommended for pregnant patients who have had close contact with someone infected with influenza. True False

Correct! Oral oseltamivir is considered safe for pregnant patients and is recommended prophylactically for those who have had close contact with someone infected with influenza.

PEP must be initiated within Choose...2487212 hours of exposure. Response for: undefined

Correct! PEP is unlikely to be effective in people who have been exposed more than 72 hours before seeking medical assistance.

A urinary tract infection (UTI) should be managed inpatient in a male who appears toxic. True False

Correct! UTIs in males are uncommon and many experts consider any UTI in a male to be complicated. A toxic patient is one who is febrile, lethargic, pale, or has other evidence of poor perfusion indicating impending septic shock, which should be managed in the inpatient setting.

During a well-child examination on a four-month-old infant, you evaluate the parent's mental health. Which statement by the parent indicates a potential problem with the parent-infant relationship? "My baby prefers to nurse in a darkened room." "My baby seems very sensitive to loud noises." "I can sense a difference in my baby's cries." "I let my baby cry a while to learn to be patient."

Correct! When parents respond promptly to their baby's needs, infants typically provide less intense distress signals. Parents who are attentive to their babies' signals, preferences, and sensitivities are more in tune with their infants' needs.

the family nurse practitioner is reviewing strategies to increase physical activity. The patient should also be educated that which of the following are commonly associated with sports injuries? 1/1 low socioeconomic status scoliosis female gender improper training

D

cgd vs idiopathic short stature

Diagnostic Studies The same screening tests used to evaluate GHD are performed to rule out pathologic conditions. A bone age x-ray can often be helpful in distinguishing between CGD and idiopathic short stature. The child with CGD will have a bone age consistent with his or her height age, whereas the child with idiopathic short stature will have a bone age consistent with his or her chronologic age

in which of the following groups of patients is tight glycemic control contraindicated? 1/1 adolescent males middle-aged females middle-aged males children under 6

Feedback Tight gycemic control is contraindicated in infants less than 2 years of age and should be instituted with Extreme Caution in children less than 7 years of age to avoid severe hyperglycemia injuring the developing brain

a nurse practitioner understands that the most likely cause of hypertension in a young child is: 1/1 glomerulonephritis pheochromocytoma rheumatic fever hyperthyroidism

Feedback although all these conditions can lead to hypertension, the most common in infants and young children is secondary hypertension because of renal disease (glomerulonephritis, polycystic kidneys, nephrosis). Endocrine induced hypertension is the second most common cause

an infant with an abnormally pitched cry May demonstrate a genetic disorder or other problems, such as: 0/1 hypothyroidism hypertelorism cleft palate pyloric stenosis Correct answer hypothyroidism

Feedback infants with hypothyroidism often have an abnormally pitched cry because of lethargy and delayed mental responsiveness. hypertelorism does not produce an abnormal cry unless accompanied by microcephaly

the family nurse practitioner is assessing a preadolescent girl for scoliosis. How is this test conducted? 1/1 have the girl band at the waist and look for a symmetry in the back and hip area examine the child fully clothed, paying particular attention to the hips and back have the child walk heel-to-toe and observe the gait and pelvis place the child on her back and flex the knees and observe for misalignment

Feedback the child should remove her shirt but leave on bra or swimsuit top and bend at the waist. The family nurse practitioner should examine for uneven hips and shoulders

in evaluating the laboratory findings for a child with iron deficiency anemia, what should the family nurse practitioner expect? 1/1 low mean corpuscular volume (MCV) and low reticulocyte count high MCV and hemoglobin of 12g / DL normal MCV and hematocrit of 34% High MCD in normal reticulocyte count

Feedback the findings associated with iron deficiency anemia include low MCV, decreased hemoglobin and hematocrit, and low reticulocyte count. Elevated MCV is associated with macrocytic anemias such as pernicious anemia

An advanced practice registered nurse (APRN) examines an infant. There is suspicion of lead poisoning, but the clinic does not have a rapid screening test for lead. The APRN orders a lab test that will show whether an elevated level of lead is present. Which lab test should this APRN order? Complete blood count with differential Complete metabolic profile Plasma cholinesterase Free erythrocyte protoporphyrin

Free erythrocyte protoporphyrin

40 An advanced practice registered nurse (APRN) reviews a patient's history and notes that prednisolone is being taken for severe allergies even though the patient no longer has allergy symptoms. The APRN decides to stop this medication and reviews the history again to determine if the medication must be discontinued gradually instead of abruptly. Under which circumstance should the APRN discontinue this medication gradually? If the patient has been receiving 10 mg daily for the past six days If the patient has been receiving 20 mg daily for the past four days If the patient has been on the medication for four weeks If the patient has been on the medication for two weeks

If the patient has been on the medication for four weeks

A patient who is eight weeks pregnant comes to the clinic because of vaginal bleeding. The advanced practice registered nurse (APRN) completes an assessment with the following findings: vital signs within normal limits dull ache in lower abdomen vaginal bleeding closed cervix Which procedure should the APRN add to this patient's plan of care? CT scan Urine pregnancy test Serum triple screening test Ultrasonography

US

murmurs

Upper right sternal border (URSB): aortic valve clicks of aortic stenosis, venous hum. Upper left sternal border (ULSB): Pulmonary valve clicks of pulmonary stenosis, pulmonary flow murmurs, atrial septal defect, patent ductus arteriosus, venous hum. Lower left sternal border (LLSB): ventricular septal defects, Still murmur, tricuspid valve regurgitation, hypertrophic cardiomyopathy, subaortic stenosis. Apex: aortic or mitral valve clicks, mitral valve regurgitation. Erb point: (left sternal border between 2nd and 3rd intercostal space) aortic ejection click or aortic stenosis, or dilated aortic root.

An advanced practice registered nurse (APRN) is completing a well-child exam on a three-month-old infant. The mother reports that breastfeeding sessions take 30-45 minutes, and the child wakes up each hour for feedings. Upon exam, the infant turns to the sound of a rattle and follows the rattle from left to right. Which assessment finding indicates the need for referral? Following the rattle from left to right Turning to the sound of the rattle Waking up each hour for feedings Feedings taking 30-45 min

Waking up each hour for feedings

A school-age child is seen in the clinic after a fragment from a glass bottle flew into the eye. What will the Family nurse practitioner do? 0/1 Refer immediately to an ophthalmologist. Attempt to visualize the glass fragment. Irrigate the eye with sterile saline. Instill a topical anesthetic.

a

Metrorrhagia

bleeding from the uterus at any time other than normal menstruation

Tietze syndrome

costochondritis + palpable edema

An advanced practice registered nurse (APRN) examines a five-year-old child, and the parent expresses concerns about signs of autism. Which screening should the APRN use to address this parent's concern? Autism Mental Status Exam (ASDE) Modified Checklist for Autism in Toddlers (M-CHAT) Checklist for Autism in Toddlers (CHAT) Childhood Autism Screening Test (CAST)

d

The mother of a 6-week-old breastfeeding infant tells the Family nurse practitioner that her baby, who previously had bowel movements with each feeding, now has a bowel movement once every second day. What will the nurse practitioner tell her? 1/1 Her baby is probably constipated. It may be related to her dietary intake. She should consume more water. This may be normal for breastfed babies.

D

Colic

: Colic is defined as crying for no apparent reason that lasts for 3 hours or more per day and occurs 3 days or more per week in an otherwise healthy infant younger than 3 months of age. Numerous randomized controlled trials (RCTs) over the years have demonstrated mixed results with using probiotics to decrease crying times, with differences noted between infants who are solely breastfed and those who are not

51 An advanced practice registered nurse (APRN) is discussing the treatment options available to a patient who is homeless and has been diagnosed with HIV. The patient admits that the $8,000 per month cost of the preferred treatment is not fiscally possible. Which action should the APRN take to assist this patient? Refer the patient to the case management services Advise the patient to speak with the patient assistance program Delay prescribing the medication Encourage the patient to cut the dose in half

A

A 21-year-old female patient presents to the clinic for a routine exam and asks the advanced practice registered nurse (APRN) about the frequency recommended for a Papanicolaou (Pap) smear. Which recommendation should the APRN share with this patient? Every three years Every five years Biannually Annually

A

A child is diagnosed with tinea versicolor. What is the correct management of this disorder? 1/1 Application of selenium sulfide 2.5% lotion twice weekly for 2 to 4 weeks Oral antifungal treatment with fluconazole once weekly for 2 to 3 weeks Sun exposure for up to an hour every day for 2 to 4 weeks Using ketoconazole 2% shampoo on lesions twice daily for 2 to 4 weeks

A

A child who has sustained a head injury after falling on the playground is brought to the clinic. The parents report that the child cried immediately and was able to walk around after falling. The Family nurse practitioner notes slight slurring of the child's speech and the child has vomited twice in the exam room. Which course of action is warranted? 1/1 Admit the child to the hospital for a neurology consult. Observe the child in the clinic for several hours. Order a head CT and observe the child at home. Send the child home with instructions for follow-up.

A

A previously healthy school-age child develops a cough and a low-grade fever. The Family nurse practitioner auscultates wheezes in all lung fields. Which diagnosis will the nurse practitioner suspect? 1/1 Atypical pneumonia Bacterial pneumonia Bronchiolitis Bronchitis

A

A single mother of an infant worries that living in a household with only one parent will cause her child to be maladjusted. To help address the mother's concerns, the Family nurse practitioner will suggest 1/1 developing consistent daily routines for the child. exposing her child to extended family members when possible. not working outside the home during the first few years. taking her child to regular play date activities with other children.

A

An adult patient with a long history of alcohol abuse is being evaluated by the advanced practice registered nurse (APRN) for chronic lower-back pain. Which action the APRN take? Encourage use of naproxen sodium Prescribe meperidine 100 mg q3h, as needed Encourage use of acetaminophen Recommend calcium intake of 2000 IU QD

A

Because of their inability to ambulate, children with cerebral palsy should be evaluated for which nutrients? 1/1 Calcium and vitamin D fat soluble vitamins iron and zinc sodium and potassium

A

The Family nurse practitioner is evaluating a 2-year-old with a documented speech delay. Screenings to assess motor skills and cognition are normal, and the child passed a recent hearing test. What will the pediatric nurse practitioner do next? 1/1 Ask the child's parents whether they read to the child. Give parents educational materials to encourage speech. Refer the child to an early intervention program. Suggest that they purchase age-appropriate music videos.

A

The Family nurse practitioner is examining a 15-year-old female who reports having her first period at age 13. She states that she has had five periods in the last year, with the last one 2 months prior. She participates in basketball at school. Which action is correct? 1/1 Perform biometric screening to determine lean body mass. Prescribe oral contraceptives pills to regulate her periods. Reassure her that this is perfectly normal at her age. Refer her to an endocrinologist for hormonal evaluation.

A

The parent of a 14-year-old child asks the Family nurse practitioner how to help the child prevent injuries when basketball tryouts begin later in the school year. Which recommendation will be of most benefit? 1/1 Pre-season conditioning proper footwear Protective knee braces stretching before practice

A

The parent of a child diagnosed with attention-deficit/hyperactivity disorder (ADHD) tells the Family nurse practitioner that the child gets overwhelmed by homework assignments, doesn't seem to know which ones to do first, and then doesn't do any assignments. The nurse practitioner tells the parent that this represents impairment in which executive function? 1/1 activation effort emotion focus

A

The parent of a school-age child reports that the child becomes frustrated when unable to perform tasks well and often has temper tantrums and difficulty sleeping. Which disorder may be considered in this child? 1/1 Generalized anxiety disorder (GAD) Obsessive-compulsive disorder (OCD) Pediatric autoimmune neuropsychiatric disorder associated with streptococcal infection (PANDAS) Separation anxiety disorder (SAD)

A

The parent of a school-age girl reports that the child has difficulty getting ready for school and is often late because of a need to check and recheck whether her teeth are clean and her room light has been turned off. What will the Family nurse practitioner recommend to this parent? 1/1 Cognitive-behavioral therapy Deferral of treatment until symptoms worsen Medication management with an SSRI Referral to a child psychiatrist

A

school-age child has a history of chronic otitis media and is seen in the clinic with vertigo. The Family nurse practitioner notes profuse purulent otorrhea from both pressure-equalizing tubes and a pearly-white lesion on one tympanic membrane ™. Which condition is most likely? 1/1 Cholesteatoma mastoiditis otitis externa Otitis media with effusion

A

Which of the following is the leading cause of nonfatal injury in children? 1/1 falls motor vehicle accidents pedestrian injury submersion incidents Feedback

A Falls are the leading cause of nonfatal injury in children and are most common from infancy through age 14. Children tend to fall from objects, balconies, windows, and trees. Falls most often occur in homes, followed by schoolyards and playgrounds. Falls are followed by motor vehicle accidents and pedestrian injury (e.g., struck by a motor vehicle while crossing the street).

the nurse practitioner is examining a 6 week old infant of Latin American descent. There are irregular areas of Deep Blue pigmentation across the infant's buttocks. This would be identified as a characteristic of: 1/1 child abuse telangiectactic nevi cutis marmorata Mongolian spots

A Feedback this best describes Mongolian spots, which are characteristic in newborns of African, Asian, or Latin descent. When closely evaluated, these but do not resemble the ecchymosis that occurs with trauma. Telangiectatic nevi are commonly known as "stork bites" and our deep pink lesions most often found on the back of the neck. Cutis marmorata is the transient mottling that occurs when an infant is cold

LEFT SHIFT

A relative increase in the number of circulating immature neutrophils (bands) is referred to as a "left shift" and typically signifies the presence of an acute bacterial infection or inflammatory process.

A 3-year-old child is brought to the clinic by a parent who reports that the child refuses to use the right arm after being swung by both arms while playing. The child is sitting with the right arm held slightly flexed and close to the body. There is no swelling or ecchymosis present. What will the Family nurse practitioner do? 1/1 Consider maltreatment as a possible cause of injury. Gently attempt a supination and flexion technique. Immobilize the arm with a sling and refer to orthopedics. Obtain a radiograph of the child's right arm and elbow.

B

An advanced practice registered nurse (APRN) examines a 18-month-old child and recognizes a finding that is a red flag for typical development. Which assessment finding for this child should concern the APRN? Must still eat pureed foods Is able to crawl but not walk Is unable to calm self Does not engage in parallel play

B

60 An advanced practice registered nurse (APRN) is leading a project team to ensure that during the discharge process patients are able to describe two ways home care instructions will be followed after release. The team is developing outcomes that will meet the criteria of the SMART tool. Which outcome should the APRN approve for the project? All patients receiving home care instructions will describe how they will follow the instructions prior to discharge. 80% of patients receiving home care instructions will describe at least two ways they will follow the instructions. 80% of patients receiving home care instructions will describe at least two ways they will follow the instructions prior to discharge. All patients will describe at least two ways they will follow the instructions prior to discharge.

C

The Family nurse practitioner (PNP) is performing a sports physical on an adolescent whose history reveals mild aortic stenosis (AS). What will the nurse practitioner recommend? 1/1 Avoidance of all sports to prevent sudden death Clearance for any sports since this is mild Evaluation by a cardiologist prior to participation Low-intensity sports, such as golf or bowling

C

A young adolescent reports chest pain associated with coughing and lifting. Physical examination reveals tenderness over several ribs, radiating to the back. Auscultation of the heart, lungs, and abdomen are normal. There is no history of injury. What will the Family nurse practitioner do? 0/1 Obtain a chest radiograph to evaluate possible causes for these symptoms. Order an electrocardiogram to rule out potential cardiovascular disease. Recommend NSAIDs, stretching exercises, and ice packs to the area. Refer the child to a pediatric orthopedist for evaluation and treatment.

Correct answer Recommend NSAIDs, stretching exercises, and ice packs to the area.

What are the 3 early manifestations of SLE in children? (select 3) 0/1 Recurrent fever of unknown origin irritability malaise arthralgia temper tantrums

Correct answer Recurrent fever of unknown origin malaise arthralgia

The primary care pediatric nurse practitioner elicits positive Ortolani and Barlow signs in a 6-month-old infant not previously noted in the medical record. What is the correct treatment? 0/1 Pavlik harness Spica cast Surgical intervention triple diapering

Correct answer Surgical intervention

a male child has swelling in the upper left thigh below the inguinal ligament. He complains of cramping and intermittent lower abdominal pain. What should be included in the differential diagnosis? 0/1 testicular torsion femoral hernia varicocele hydrocele

Correct answer femoral hernia Feedback this is a probable femoral hernia. The other three diagnosis involves scrotal ideologies. Testicular torsion is a medical emergency. The testicle becomes deprived of its blood supply. Variocele and hydrocele are both confined to the scrotum. Hydrocele is evidence by scrotal swelling, is painless, and is easily transilluminated. Varicocele is confined to the scrotum to but it prevents most commonly in the Adolescent age group with a large, soft scrotal mass that decompresses when the child lays down

differentiation between structural and functional scoliosis can be done by placing the child in Adams position. Which of the following occurs in this position? structural scoliosis disappears, and functional scoliosis is enhanced persistent functional scoliosis is indicated functional scoliosis disappears, and structural scoliosis is enhanced curves greater than 10 degrees are indicated

Correct answer functional scoliosis disappears, and structural scoliosis is enhanced Feedback In Adam's position which is forward bending, arms loose at side, and thumbs hook together, true scoliosis (structural) is demonstrated by an elevated rib hump, whereas the functional top related to other conditions is not a parent. Persistent functional scoliosis can eventually become structural.

clubfoot: 0/1 always requires an urgent Neurosurgical referral involve the foot and lower extremity involves the foot only can be treated with corrective shoes and exercises

Correct answer involve the foot and lower extremity Feedback clubfoot is also known as Talipes equinovarus.clubfoot is usually accompanied by a smaller and shorter leg and calf on the affected side. The foot is plantar flexed and the forefoot and so are thrust immediately. It is an urgent Orthopedic referral not Neurosurgical referral. Casting and splinting usually takes place initially, however, if needed, surgical treatment may occur after three to six months. The majority of patients will be successfully correct with taping, splinting, and / or casting went early intervention occurs. Exercise and corrective shoes are more appropriate for metatarsus adductus

an eight-year-old child comes to the clinic with a history of hard, painful bowel movements. The child's physical examination yields normal findings and an x-ray confirms constipation. The nurse practitioner instructs the parent to increase fiber rich foods for the child and to:* 0/1 increase cheese intake to two slices per day limit milk consumption to 16 oz per day they make a child the one candy bar per day offer 24 oz of a sports drink per day

Correct answer limit milk consumption to 16 oz per day Feedback milk containing products in those high in carbohydrates tend to worse and constipation. The recommendations for children for milk consumption should be 16 oz per day. Increasing the cheese intake could worsen constipation. Candy bar should not be offered daily but only occasionally As a treat. Sports drinks are high in carbohydrates which adds unnecessary calories to the child's diet. Children should consume water and two glasses of milk daily with a boy dance of sugary beverages

which of the following is true about metatarsus adductus? 0/1 this is another term for clubfoot mild flexible metatarsus usually spontaneously corrects x-rays are usually necessary to diagnose this infants frequently require casting to correct

Correct answer mild flexible metatarsus usually spontaneously corrects Feedback metatarsus adductus or MA is the most common congenital foot deformity. It is not related to clubfoot. In MA, the forefoot deviates medially, and the hind foot remains in a neutral position. This is a common cause of in-toeing gait and usually spontaneously corrects. This can be diagnosed on clinical presentation. An x-ray is not necessary. Casting is usually reserved for severe cases characterized by a rigid forefoot

During a cardiac assessment of a four-month-old infant, family nurse practitioner notes a continuous murmur. This finding is consistent with a diagnosis of: 0/1 coarctation of the aorta patent ductus arteriosus (PDA) ventricular septal defect aortic stenosis

Correct answer patent ductus arteriosus (PDA) Feedback a continuous murmur is consistent with PDA. The turbulent flow of blood from the aorta through the PDA to the pulmonary artery results in this characteristic murmur. Coarctation of the aorta presents with upper extremity hypertension, systolic murmur, and weak or absent femoral pulses. Ventricular septal defect is characterized by a loud, harsh pansystolic murmur heard best at the lower left sternal border. Aortic stenosis has a systolic murmur.

obesity related primary hypertension secondary hypertension endocrine related

Correct answer secondary hypertension Feedback secondary hypertension is more common and pre-adolescents, with most cases caused by renal disease. Primary or essential hypertension is more common in adolescence and has multiple risk factors, including obesity in a family history of hypertension.

You are reviewing a child's complete blood count with differential white blood cell values and recognize a left shift. What causes this? A decreased eosinophil count An elevated neutrophil count A decreased lymphocyte count An elevated monocyte count

Correct! A left shift occurs when there is an increase in the number of circulating immature neutrophils. It indicates a bacterial infection or an inflammatory disorder. Eosinophils are associated with an antigen antibody response and are elevated with exposure to allergens, skin inflammation, or parasites. Lymphocytes are nongranulocytes that are elevated with viral infections. Monocytes are nongranulocytes and are elevated in infections, inflammation, and some leukemias. Elevations of nongranulocytes are referred to as a right shift.

A 55-year-old male patient is being seen by an advanced practice registered nurse (APRN) for the inability to complete the act of intercourse due to erectile dysfunction. The APRN suspects the patient has erectile dysfunction but is unsure whether the cause is organic or psychogenic. Which diagnostic test should the APRN order to determine the origin of this problem? Sleep study Sildenafil trial Digital rectal exam Postage stamp test

Correct! A postage stamp test is conducted by placing a roll of stamps around the penis at bedtime; if the stamps are broken apart in the morning, it is determined that erection has occurred. This test has a high sensitivity for differentiating between organic (inability to have an erection) and psychogenic (psychological)

True or False A score of more than 10 on the Edinburgh Postnatal Depression Screen suggests depression may be present. True False

Correct! A score of more than 10 on the Edinburgh Postnatal Depression Screen suggests depression may be present, and further evaluation is recommended.

A 26-year-old male patient is diagnosed with gonorrhea and is speaking with an advanced practice registered nurse (APRN) about treatment. The APRN provides a single-dose antibiotic therapy and then educates the patient about the wait period recommended before resuming sexual activity. Which wait period should the APRN recommend? Seven days Three weeks Seven weeks Three days

Correct! A seven-day wait period before resuming sexual activity is appropriate after receiving a single-dose antibiotic for gonorrhea.

During pregnancy, most aches and pains can be managed with acetaminophen, but a short course of narcotic medications for severe pain or serious injuries is considered safe. True False

Correct! Acetaminophen is safe throughout pregnancy and is recommended for mild pain. In cases of severe pain, a short course of narcotics may be needed, which is also considered safe in pregnancy.

You are performing an examination on an adolescent male who asks about sexual identity because of concern that a friend is worried about being gay. How should you respond to this patient's concerns? Remind the adolescent that mandatory reporting requires disclosure to parents. Suggest that the adolescent discuss sexual concerns with his parents. Tell the adolescent that unless he is at risk, what he says will be confidential. Provide the teen with a questionnaire to gain information about his sexuality.

Correct! Adolescents should be encouraged to divulge information about their sexuality to providers by assuring them that confidentiality will be maintained unless the health of the child or others is at risk. The adolescent may be trying to ask questions about himself in a manner that does not implicate his own sexuality, so you should attempt to gain his confidence.

True or False In studies, most antihistamines do not appear to be linked to birth defects. True False

Correct! Although more research is needed, studies involving prenatal use of H1 receptor antagonists such as diphenhydramine and loratadine have not been linked to birth defects.

Which baseline lab work is required for individuals beginning PEP? Serum liver enzymes Thyroid function Hormone levels Complete blood count

Correct! Baseline liver function should be evaluated before starting PEP

A 10-year-old child is hit in the head with a baseball during practice and is diagnosed with concussion, even though no loss of consciousness occurred. You evaluate the child two weeks after the injury and learn that the child is still experiencing some sleepiness every day. The neurological examination is normal. The child and the parent are adamant that the child should be allowed to return to play baseball. What should you recommend? Continued physical and cognitive rest Return to play Continued cognitive rest only Continued physical rest only

Correct! Both physical and cognitive rest is needed after a diagnosis of concussion in a child, particularly if symptoms continue following the injury. Cognitive recovery may lag behind physical recovery and is a key factor in return-to-play decisions. Only after all symptoms are resolved may athletes progress through steps to gradually return to play.

Which lab value is most concerning in an infant with fever and a suspected bacterial infection? C-reactive protein of 11.5 mg/L Platelet count of 475 White blood cell (WBC) count of 14 Lymphocyte count of 8.7

Correct! C-reactive protein (CRP) levels are nonspecific acute phase indicators of inflammation with low diagnostic value except in predicting the likelihood of sepsis in infants, especially when the level is greater than 10 mg/L. Elevated lymphocyte, platelet, or white blood cell (WBC) counts help with the differential diagnosis, but these values are not especially concerning.

An advanced practice registered nurse (APRN) is examining an eight-year-old patient and using the patient's self-report as part of the pain assessment. Which developmental characteristic should the APRN take into consideration as part of this assessment? Continuity of care dictates the same staff member should assess the pain each time. Younger children may confuse sadness with pain. Some children may overstate pain so they can agitate the healthcare provider. Pain may be under reported if the child fears it may upset the parents.

Correct! Children may under report pain if they lack knowledge that pain can be treated or if they fear their complaint may upset their parents.

The parent of an asthmatic child asks you whether the child may engage in strenuous exercise. What should you tell the parent? Children with asthma should be excluded from vigorous exercise and most strenuous sports. Children with asthma have shown improved aerobic and anaerobic fitness with moderate to vigorous physical activity. Physical activity has been shown to improve overall pulmonary function in children with asthma. Vigorous exercise helps improve symptoms in children with poorly controlled asthma.

Correct! Children with mild or well-controlled asthma may participate in moderate to vigorous sports. Such participation has been shown to benefit aerobic and anaerobic fitness, which helps lung function and overall health outcomes. It is not necessary to exclude children with asthma from sports as long as symptoms are well controlled. Children with poorly controlled asthma should not engage in sports until symptoms are under control.

True or False Ectopic pregnancy is most likely to occur between the sixth and twelfth weeks of gestation. True False

Correct! Ectopic pregnancy is most likely to occur between the sixth and twelfth weeks of gestation.

You are performing a vision screen on a four-month-old infant and note the presence of convergence and accommodation with mild esotropia of the left eye. What should you do? Reassure the parents that the infant will outgrow this Refer the infant to a pediatric ophthalmologist Recheck the infant's eyes in two to four weeks Patch the right eye to improve coordination of the left eye

Correct! Esotropia that continues or occurs at three to four months of age is abnormal, so the infant should be referred to a pediatric ophthalmologist. You should not determine whether an eye patch should be used. Because this symptom is abnormal at this age, you should not reassure the parents that the infant will outgrow this. Esotropia after three to four months of age must be evaluated by a specialist and not reevaluated in two to four weeks.

You evaluate a 15-year-old female basketball player who has secondary amenorrhea and note a body mass index (BMI) in the third percentile. How should you counsel this patient? Begin a program of plyometrics and strength training Consider a different sport such as volleyball Amenorrhea in female athletes is not concerning Work with a dietician to improve healthy weight gain

Correct! Female athletes who have amenorrhea have an increased risk of stress fractures. The adolescent should work to attain a healthy weight, which should allow normal periods to return and reduce this risk. Even though amenorrhea in female athletes is common, it is concerning. Plyometrics and volleyball can increase the risk of stress fractures because both involve jumping and thus should not be suggested.

The parent of a school-age child reports that the child becomes frustrated when unable to perform tasks well and often has tantrums and difficulty sleeping. Which disorder may this child have? Obsessive-compulsive disorder (OCD) Pediatric autoimmune neuropsychiatric disorder associated with streptococcal infection (PANDAS) Generalized anxiety disorder (GAD) Separation anxiety disorder (SAD)

Correct! GAD is characterized by over-concern about competence, significant self-consciousness, irritability and tantrums, and poor sleep.

The term "intersex" means a person whose gender has been changed. True False

Correct! Gender reassignment is a surgery to change an individual's gender. Intersex refers to those individuals with a disorder related to sexual development resulting in chromosomal anomaly or ambiguous genitalia.

Glucophage is contraindicated for gestational diabetes. True False metformin

Correct! Glucophage has been clinically tested and is safe to take for gestational diabetes

An advanced practice registered nurse (APRN) is completing a postoperative examination for a nine-year-old child. The patient is sedated and needs to be assessed for pain. Which behavioral nonverbal cue should the APRN use as an indicator during this assessment? Tachycardia Hypermetabolism Grinding teeth Diaphoresis

Correct! Grinding or clenching of teeth is a behavioral nonverbal cue that indicates pain.

An advanced practice registered nurse (APRN) is examining a four-year-old child. Which screening assessment should be repeated due to this child's age? Scoliosis exam Physical abuse Hearing test Emotional quotient

Correct! Guidelines recommend pure-tone audiometry screening at 3, 4, 5, 6, 8, 10, 12, 15, and 18 years of age.

Many LGBTQIAP+ patients have had prior experiences with healthcare providers that have left them feeling marginalized or stigmatized. True False

Correct! It is important to recognize that many LGBTQIAP+ patients have had negative prior experiences with healthcare providers.

Low testosterone levels in men are associated with increased mortality. True False

Correct! Low testosterone levels have been linked to an increased risk of cardiovascular disease, sudden death, and an overall increase in mortality.

Which elevated white blood cell (WBC) element may indicate a viral infection? Leukocytes Bands Neutrophils Lymphocytes

Correct! Lymphocytes are usually elevated during viral infections. Bands and neutrophils are generally elevated with bacterial infections. Leukocytes comprise all WBCs and are usually, although not always, elevated during bacterial infections.

LGBTQIAP+ patients may avoid Choose...medical carepublicrelationships due to a history of negative interactions. Response for: undefined

Correct! Many LGBTQIAP+ patients avoid needed medical care and preventative screenings due to a history of negative interactions with healthcare providers.

Older LGBTQIAP+ adults are less likely to have children or other family members who are involved in their care. True False

Correct! Many older LGBTQIAP+ patients do not have children and may be estranged from family members. They are more likely to have no family involved in their care.

A four-year-old has had difficulty initiating and maintaining sleep. The parent has tried several nonpharmacological methods with variable success and asks you about medications. What should you recommend? Zolpidem Lorazepam Diphenhydramine Melatonin

Correct! Medications to treat dyssomnias are generally discouraged in children because of the side effects. The main treatment is behavioral therapy and sleep hygiene. If medications are used, melatonin is the most commonly prescribed. Diphenhydramine can lead to parasomnias in some children. Benzodiazepines such as lorazepam can lead to dependence. Sedatives such as zolpidem have high risk of side effects.

True or False Gestational diabetes is a risk factor for developing preeclampsia. True False

Correct! Preeclampsia occurs more frequently in pregnant patients with gestational diabetes.

True or False For LGBTQ+ teens, a group of supportive friends has been shown to be as effective as being linked to LGBT organizations. True False

Correct! The connection and support of an LGBT organization resulted in greater self-esteem and life satisfaction in a recent study by Toomey, et al, 2017.

True or False The normal prostate is heart shaped and measures approximately 4 × 3 × 2 cm. True False

Correct! The normal prostate is heart shaped and measures approximately 4 × 3 × 2 cm.

You are teaching the parent of a child with dry skin about hydrating the skin by bathing. What should you include in your teaching? Keep the child in the bath until the skin begins to "prune" Apply lubricating agents at least one hour after the bath Have the child soak in a lukewarm water bath Soaping should be done at the beginning of the bath

Correct! When using a bath to hydrate dry skin, lukewarm water should be used. Lubricating agents should be applied immediately after patting the skin dry. The bath should last long enough to allow the skin to become moisturized without becoming super saturated or "pruned." Soaping and shampooing should be performed at the end of the bath followed by thorough rinsing.

A 12-month-old patient presents to a clinic with sudden onset of diarrhea and vomiting. An advanced practice registered nurse (APRN) notices that the child cries continuously with no tear production, has cool skin, is irritable, has a slight fever, and is tachycardic. The patient's parent reports that the family was on a camping trip outside the country for one day when the diarrhea and vomiting started, and the problem has persisted for three days. The parent also says, "I have had to change the diaper only once each day since we got back from our trip." Which diagnosis should the APRN make for this patient? Intussusception Pyloric stenosis Gastroesophageal reflux disease Gastroenteritis

D

A 15-year-old female patient presents to an advanced practice registered nurse (APRN) with a sudden onset of high fever, a diffuse macular rash, vomiting, diarrhea, hypotension, and conjunctival injection. Symptoms have been ongoing for the last 14 hours. The patient is currently menstruating and is not sexually active. Which action should the APRN take for this patient? Treat viral etiology and fever immediately with acetaminophen and push fluids, and then recheck in two days Treat anemia with CBC immediately, with acetaminophen for fever, and with increased fluids, and then recheck in two days Treat necrotizing fasciitis with a referral to an emergency room for acute management Treat toxic shock syndrome with a referral to an emergency room for acute management

D

A child with a recent history of URI reports tingling and pain in one ear followed by sagging of one side of the face. The Family nurse practitioner observes that the child cannot close the eye or mouth on the affected side but does not elicit limb weakness on that side. What will the nurse practitioner do? 0/1 Initiate a short course of antibiotic therapy. Perform diagnostic testing to rule out serious causes. Prepare the parents for lifelong complications. Prescribe oral prednisone 1 mg/kg/day initially.

D

A female patient comes to the emergency department and states that she was raped on her way home from classes at the local college. The advanced practice registered nurse (APRN) notes that the patient appears disheveled, has bruising on her face and thighs, and has abrasions present on her knees, back, and elbows. Which action should the APRN take? Prescribe ulipristal acetate 30 mg once daily for three days Give a one-time dose of alprazolam 0.1 mg to decrease post-traumatic stress disorder (PTSD) symptoms Call the police to interview the patient and collect physical evidence Develop a relationship with the patient and get consent to collect evidence

D

A healthy 14-year-old female has a dipstick urinalysis that is positive for 5-6 RBCs per hpf but otherwise normal. What is the first question the Family nurse practitioner will ask this patient? 1/1 "Are you sexually active?" "Are you taking any medications?" "Have you had a recent fever?" "When was your last menstrual period (LMP)?"

D

An adolescent takes ibuprofen, acetaminophen, and a tricyclic antidepressant (TCA) to treat phantom limb pain and reports that the medications are no longer effective. What will the Family nurse practitioner do? 1/1 Change the TCA to a selective serotonin reuptake inhibitor. Evaluate the adolescent for drug-seeking behavior. Increase the TCA dose and reevaluate in 2 to 3 weeks. Refer the adolescent to a pain management specialist

D

An advanced practice registered nurse (APRN) is evaluating a 14-year-old female patient who reports having a lumpy breast. After assessment, the APRN takes action to support the patient's breast health. Which action should the APRN take for this patient? Explain that breast cancer is common in this age group Recommend a BRCA gene test Recommend mammogram screening by age 18 Explain the normal breast variations common during a self-exam

D

During a well child assessment of an 18-month-old child, the Family nurse practitioner observes the child point to a picture of a dog and say, "Want puppy!" The nurse practitioner recognizes this as an example of what type of speech? 1/1 holophrastic receptive semantic telegraphic

D

The Family nurse practitioner is obtaining a medical history about a child. To integrate both nursing and medical aspects of primary care, which will be included in the medical history? 0/1 Complementary medications, alternative health practices, and chief complaint Developmental delays, nutritional status, and linear growth patterns Medication currently taking, allergy information, and family medical history Speech and language development, beliefs about health, and previous illnesses

D

The Family nurse practitioner is performing an exam on an adolescent male who asks about sexual identity because of concern that a friend is worried about being gay. Which response will the nurse practitioner make in this situation? 1/1 Provide the teen with a questionnaire to gain information about his sexuality. Remind the adolescent that mandatory reporting requires disclosure to parents. Suggest that the adolescent discuss sexual concerns with his parents. Tell the adolescent that, unless he is at risk, what he says will be confidential

D

the mother of a 16 month old child informs you that she feeds the baby skim milk. What is the appropriate education to give the mother? 1/1 skim milk contains too little protein skim milk contains an insufficient amount of calcium skim milk is not as easily digested as whole milk skim milk provides an adequate amount of fatty acids

D

a family nurse practitioner examines a two-year-old who has a tender, edematous and erythematous knee. a child's parent reports that two weeks ago the child experienced a five-day episode of fever and bloody diarrhea. Which organism is likely the cause of the child's current symptoms? 0/1 Escherica coli Giardia Lamblia Rotavirus Shigella Flexneri

D Shigella causes for to 7 Days of fever and diarrhea that may contain blood in mucus. Most susceptible children are between six months to three years of age. The child in this scenario has developed reactive arthritis which is a known complication of disinfection. Rotavirus in Giardia do not cause this illness. Ecoli usually infects the urinary track system and not other areas of the body.

the family nurse practitioner is performing a newborn assessment on a full-term infant approximately 6 hours after birth. When evaluating the infant's head, the FNP identifies an edematous area that crosses the cranial suture lines, is soft, and varies with size. The cranial suture lines have minimal space between them. What is the interpretation of these findings? 1/1 a cephalhematoma is present the cranial suture lines indicate premature closing molding of the infant's head is present a caput succedaneum is present Feedback

D caput succedaneum results from pressure against the mother's cervix during labor. The edematous area crosses the suture lines and will resolve within hours to days after birth. There is normally a space between the cranial suture lines, if there is no space or an overriding suture line, it may be caused by Moulding. Widening of the suture line May indicate increased intracranial pressure. Cephalhematoma is characterized by bleeding between the bone and the periosteum, and it does not cross the suture line

Which antibiotic would be appropriate for the FNP to prescribe for B-lactamase production by strains of Haemophilus influenzae and Moraxella catarrhalis in children with Acute otitis media? 0/1 Amoxicillin (Amoxil) Erythromycin-sulfisoxazole (Pediazole) Penicillin V Potassium (Pen-VK) Amoxicillin/Clavulanic acid (augmentin) Correct answer Amoxicillin/Clavulanic acid (augmentin) .

Feedback Augmentin is effective against B-lactamase production. Cephalexin, cefuroxime, or cefixime can be used as alternatives. E-mycin has recently been reported as well as effective. Cephalosporins are effective against B-lactamase production by H. influenzae and M. catarrhalis. Amoxicillin is ineffective against B-lactamase production as is Penicillin V

for the child with congenital heart disease (CHD) and a permanent pacemaker, electrical safety precautions include avoidance of: 0/1 cellular phones microwave ovens household electrical appliances metal detectors Correct answer metal detectors

Feedback metal detectors and electric fences have an electromagnetic field that could alter the Pacemakers function temporarily. In addition, the alarm will be set off as a result of the metal in the pacemaker. For the child with a pacemaker, and electric shock May irreparably damaged the pacemaker, and immediate surgical replacement would be necessary. There is no risk of electromagnetic interference between the permanent Pacemaker and household items such as electrical appliances, radios, electronic equipment, cellular phones, or microwave ovens. Most electrical appliances have filtering systems that prevent interference with the Pacemakers functions, and pacemakers utilize shielding, filters, and bipolar leads to mitigate electromagnetic interference.

a child has 8 to 10 medium brown cafe au lait spots less than 1 cm in diameter. The differential diagnosis should include: 1/1 vitiligo eczema neurofibromatosis neuroblastoma

Feedback neurofibromatosis or NF is a common neurocutaneous disorder. The most common form is Von Recklinghausen's NF. approximately 85% of patients with NS have this type.the incident is about 1 and 2600 people. Children with this disorder may have cognitive deficits, learning disabilities and other neurological related problems. They should be referred to neurology for diagnosis and treatment.

18 year old girl comes to the clinic with complaints of nausea, vomiting, and right upper quadrant pain during vigorous play. She is also more tired than usual. Serologic tests for immunoglobulins IGM antibodies are ordered to rule out: 1/1 cholecystitis hepatitis A infectious mononucleosis hepatitis B

Feedback right upper quadrant pain during exercise, malaise, nausea, and vomiting or early signs of hepatitis A. Elevated IGM antibodies indicate a recent or current infection. Clinical findings of cholecystitis include colicky pain in the right upper quadrant with radiation to the flanks after a large meal. Mononucleosis, common in college age adults and young children, has the classic triad of fever, exudative pharyngitis, and adenopathy, particularly posterior cervical. Hepatitis B causes symptoms ranging from a symptomatic seroconversion to acute illness with malaise, anorexia, and nausea to fatal hepatitis.

the nurse practitioner identifies satellite lesions in a six-month-old infant. These are: 1/1 worrisome in an infant this age indicative of a candidal infection found in the axilla bilaterally completely benign lesions present at birth

Feedback satellite lesions are common in the diaper area of infants with a candida diaper infection. They generally are associated with a beefy red rash. The skin creases are most commonly affected but satellite lesions can be found anywhere that candida can be found

the chest circumference of a 12 month old is: 1/1 1-2 cm larger than head circumference routinely measured on well child visits smaller than head circumference equal to head circumference

Feedback the chest circumference is not routinely measured at well-child visits, but is assessed if there is concerned about the circumference of either the head or the chest. An exception to this observation can occur in premature infants or the head grows very rapidly. Normally, the head exceed the chest circumference by 1 to 2 cm from birth until 6 months. Between 6 and 24 months to head and chest circumference should be about equal and buy two years of age that you should be larger than the head. The chest circumference is measured at the nipple line

a mother brings her school-age child to the clinic and reports a recent history of easy fatigability, unexplained bruising and multiple courses of antibiotics for symptoms of upper respiratory infection during the last six months. Physical exam revealed scattered bruising and no apparent pattern and cervical lymphadenopathy. What should be the first step of the diagnostic workup for this child? 1/1 chest x-ray and electrocardiogram liver function tests and abdominal ultrasound prothrombin partial thromboplastin time complete blood count with differential and platelet count

Feedback the clinical presentation would make the family nurse practitioner consider the diagnosis of leukemia, necessitating a workup. CBC with differential, along with platelet count (indicators of bone marrow function) is the diagnostic approach for leukemia. Further testing may be necessary, but the work up should always include a CBC

a daycare center director calls to confirm when a child can return to the center after having fifth disease (erythma infectiosum). the family nurse practitioners responses based on the knowledge that the. Of communicability lasts until: 1/1 the rash is gone the rash appears upper respiratory symptoms are gone the transient joint pain disappears

Feedback the incubation. For this condition is 4 to 14 days, with communicability until the rash appears

during a gynecological exam at the Family Planning clinic, and underweight 17 year old presents with bruising around her upper torso and genitalia. She is minimally interactive and avoids eye contact as much as possible. Priority intervention should focus on: 1/1 laboratory work to rule out bleeding disorder nutritional assessment to determine possible anemia determination of possible physical abuse finding out if she has a support system

Feedback the presence of bruising, particularly on genitalia, should raise suspicion of abuse. Combined with her nonverbal Behavior, the bruising should prompt the family nurse practitioner to explore the possibility of abuse.

A transgender patient presents to the primary clinic for an annual screening. During the examination, the advanced practice registered nurse (APRN) notes that the patient has transitioned from female to male. Which diagnostic screening tool should the APRN recommend for this patient? Human chorionic gonadotropin (hCG) levels Pap smear Prostate-specific antigen levels Digital rectal exam

Pap smear

Uncomplicated Cystitis

Use regional antibiotic resistance patterns and culture and sensitivity results when choosing antibiotics. Short-term (3 to 5 days) antibiotics may be as effective in treating non-febrile bladder infections as standard 7- to 10-day dosing with no increased risk of recurrence (Elder, 2016d). Children 2 to 24 months old and febrile children should have 7 to 14 days of antibiotics.

The Family nurse practitioner performs a physical examination on a 9-month-old infant with congenital hypothyroidism who takes daily levothyroxine sodium and notes a recent slowing of the infant's growth rate. What will the nurse practitioner order? 1/1 Free serum T4 and TSH levels Serum levothyroxine level Total T4 and free T4 levels TSH and total T4 levels

a

Genu Varum

bow legged

A 14-year-old child has a 2-week history of severe itching and tearing of both eyes. The Family nurse practitioner notes redness and swelling of the eyelids along with stringy, mucoid discharge. What will the nurse practitioner prescribe? 0/1 Saline solution or artificial tears Topical mast cell stabilizer Topical NSAID drops Topical vasoconstrictor drops

c

A 7-month-old infant has had two prior acute ear infections and is currently on the 10th day of therapy with amoxicillin-clavulanate after a failed course of amoxicillin. The Family nurse practitioner notes marked middle ear effusion and erythema of the TM. The child is irritable and has a temperature of 99.8°F. What is the next step in management of this child's ear infection? 0/1 Order a second course of amoxicillin-clavulanate. Perform tympanocentesis for culture. Prescribe clindamycin twice daily. Refer the child to an otolaryngologist.

d

According to the Centers for Disease Control and Prevention (CDC), PrEP should be offered to all adolescents and adults who have had sex in the last six months with inconsistent condom use. True False

. The CDC recommends that PrEP be offered to anyone who has had anal or vaginal sex in the past six months with one of the following additional add-ons: sex with an HIV-positive sexual partner (especially if the partner has an unknown or detectable viral load), the patient has had a recent bacterial STI, or the patient has a history of inconsistent or no condom use with sexual partners, and HIV-positive sexual partner (especially if the partner has an unknown or detectable viral load); or a recent bacterial STI, or a history of inconsistent or no condom use with sexual partner(s).

Ranula

. The cyst should be excised by an oral surgeon

hemolytic

; hemolytic. Hemolytic anemias are caused by premature destruction of RBCs and increased marrow production of reticulocytes. They can be classified as either hereditary or acquired and should be suspected in cases of an elevated reticulocyte count in the absence of bleeding or heparin therapy

The Family nurse practitioner is performing a well child examination on a high school age adolescent who plays football who has hypercalciuria. Which dietary supplement will the nurse practitioner question the adolescent about? 1/1 Protein supplements salt tablets sports drinks Vitamin C

A

An adolescent is diagnosed with major depression, and the mental health specialist has prescribed fluoxetine. What other treatment is important to protect against suicide risk? 0/1 Addition of risperidone therapy Cognitive-behavioral therapy Family therapy Hospitalization

B

A six-week postpartum, lactating mother presents to the clinic with complaints of headaches. The advanced practice registered nurse (APRN) notes that the patient has poor posture while nursing her infant and suggests using a support pillow and starting an analgesic. Which analgesic should the APRN prescribe for this patient? Acetaminophen Aspirin Tramadol Oxycodone

Acetaminophen

A patient speaks to an advanced practice registered nurse (APRN) about a sexual assault by a same-sex partner that took place one week earlier. During the exam, the APRN notes that the patient appears calm, but the patient mentions feeling guilt, shame, anxiety, and disbelief about the incident and having difficulty sleeping. Which phase of psychological sequelae of sexual assault should the APRN assign to this patient's response? Acceptance phase Integration phase Resolution phase Acute phase

Acute phase

An advanced practice registered nurse (APRN) is caring for an intrapartum patient with magnesium sulfate intravenously infusing. The APRN notes that the patient is more somnolent and has patellar reflexes +1 and a serum magnesium sulfate level of 10.8 mg/dL. Which plan of care should the APRN initiate? Administer hydralazine 5 mg IV Increase the oxytocin infusion rate Administer calcium gluconate 10 ml Increase the magnesium sulfate infusion rate

Administer calcium gluconate 10 ml

A 55-year-old male is being seen by an advanced practice registered nurse (APRN) and is upset about being unable to complete the act of intercourse because an erection cannot be sustained. The patient does not take any medications and has a body mass index of 32. The APRN suspects the patient has erectile dysfunction. Which intervention should the APRN recommend at this time? Recommend bi-weekly sessions with a psychologist Start an aggressive treatment of tadalafil Advise to walk 30 minutes daily for five days a week Prescribe amitriptyline for depression

Advise to walk 30 minutes daily for five days a week

An advanced practice nurse (APRN) examines a child who presents with mild pallor and splenomegaly. The APRN suspects beta thalassemia minor. Which action should the APRN take for this patient? Monthly blood transfusions Genetic testing and counseling Iron and calcium supplements Periodic apheresis

B

During an annual wellness exam, a patient presents with 2+ edema in the bilateral lower extremities and feet. The advanced practice registered nurse (APRN) rules out any acute causes. Which sensory change would the APRN suspect is causing this problem? Reduced tactile sensation Alternation in taste Decreased olfactory nerve fibers Increased intraocular pressures

Alternation in taste

Femoral Anteversion

An inward twisting of the femur (thigh bone) that causes a person's knees and feet to turn inward or have what is known as a pigeon-toed appearance

A school-age child has recurrent diarrhea with foul-smelling stools, excessive flatus, abdominal distension, and failure-to-thrive. A 2-week lactose-free trial failed to reduce symptoms. What is the next step in diagnosing this condition? 1/1 Lactose hydrogen breath test Serologic testing for celiac disease Stool for ova and parasites Sweat chloride test for cystic fibrosis

B

An adolescent female has a history of repaired tetralogy of Fallot (TOF). Which long-term complication is a concern for this patient? 1/1 Aortic stenosis Chronic cyanosis Mitral valve prolapse Ventricular failure

C

cvs

Children 5 Years Old or Younger • Cyproheptadine (first choice): 0.25-0.5 mg/kg/day divided bid or tid. Maximum dosage 2-6 years: 12 mg/24 h. Children Older than 5 Years Old • Amitriptyline (first choice): 0.1-0.25 mg/kg at bedtime, increase weekly by 0.1-0.25 until maximum dose of 2 mg/kg/24 h or 75 mg/24 h. For doses >1 mg/kg/24 h, divide daily dose bid and monitor electrocardiogram (ECG). Monitor ECG before starting and 10 days after peak dose. Adult: Initial 10-25 mg/dose qhs PO; reported range oSf 10-400 mg/24 h. • Propranolol

A 23-year-old male is seen in the clinic with unilateral painful testicular swelling. The patient reports that he only has vaginal intercourse with women. After assessment, the advanced practice registered nurse (APRN) determines the patient has epididymitis. Which organism should the APRN suspect as the cause of this problem? Staphylococcus aureus Pseudomonas aeruginosa Chlamydia trachomatis Escherichia coli

Chlamydia trachomatis

An advanced practice registered nurse (APRN) is seeing a 30-year-old patient who is having excessive and prolonged menses that last more than seven days. The interval spacing is typical. The examination and lab work identify no underlying pathology. Which diagnosis should guide the treatment plan for this patient? Menometrorrhagia Menorrhagia Metrorrhagia Polymenorrhea

Correct Menorrhagia

The Family nurse practitioner reviews a child's complete blood count with differential white blood cell values and recognizes a "left shift" because of what event? 0/1 A decreased eosinophil count. A decreased lymphocyte count. An elevated monocyte count. An elevated neutrophil count.

Correct answer An elevated neutrophil count.

An advanced practice registered nurse (APRN) is examining a 15-year-old female patient for a sports physical. The APRN notes scarcely pigmented pubic hair, minimal bilateral breast bud development, and a lack of menarche. Which screening tool is this APRN utilizing? PHQ-2 Denver II CRAFFT Tanner

D

n advanced practice registered nurse (APRN) is working with an integrated multidisciplinary team and recognizes that team processes need to be strengthened. The APRN suggests a strategy to meet that goal. Which strategy should the APRN use to meet this goal? Identifying clear patient outcomes Introducing value-based interviewing Initiating cross-training Instituting clinical case review

D

A 28-year-old woman presents to clinic with complaints of painful vulvar ulcerations. After further evaluation, the advanced practice registered nurse (APRN) diagnoses the client with herpes simplex virus. Which education should the APRN provide to the patient for this condition? The virus is spread when symptoms are present. Daily suppressive therapy should be used for 10 or more occurrences per year. Vesicular lesions will last for two to three days. Episodic therapy should be started whenever the lesions appea

Daily suppressive therapy should be used for 10 or more occurrences per year.

During the annual wellness exam, an older adult patient reports to the advanced practice registered nurse (APRN) problems with sleep, incontinence, and anorexia. The APRN notes the patient has shin spots and recurrent yeast infections of the skin. A falls risk assessment reveals three falls in the past three months. Which condition should the APRN further evaluate for in this patient? Hyperthyroidism Diabetes mellitus Renal insufficiency Osteoarthritis

Diabetes mellitus

A 29-year-old male is being seen by an advanced practice registered nurse (APRN) for malaise, dysuria, and lower-back pain. After examination, the APRN detects a tender, warm, and boggy prostate. Penile secretions are positive for neisseria gonorrhoeae. Which treatment should the APRN prescribe for this patient? Ciprofloxacin 500 mg bid for 5 to 7 days Amoxicillin 875 mg bid for 10 to 14 days Levofloxacin 750 mg qd for 5 to 7 days Doxycycline 100 mg bid for 7 days

Doxycycline 100 mg bid for 7 days

The term caput succedaneum refers to: 1/1 cradle cap atopic dermatitis scalp edema asymmetrical head shape

Feedback caput succedaneum is a common finding in newborns. It is a result of pressure over the presenting part. This results and some ecchymosis of the scalp. While this may be disturbing to new parents who observe this in their newborn, it will resolve in a few days and it's harmless

congenital genitourinary anomalies make place a child at risk for: 1/1 lifelong urinary incontinence, dysmotility, and pain infection, impaired fertility, and impaired renal function constipation, tethered spinal cord, and delayed ambulation chronic diarrhea, kidney stones, and poor nutrition

Feedback congenital genitourinary anomalies can place a patient at risk for urinary tract infection, which include obstruction, impaired renal function, impaired fertility and psychosocial difficulties

circumcision can prevent which of the following? 1/1 paraphimosis epididymitis sexually transmitted disease prostatitis

Feedback paraphimosis is a retraction disorder related to a constricted prepuce that can be relieved through circumcision

A child received a burn on his chest from a cup of hot coffee. On examination, the injured area appeared moist, red to Ivory White in color, and features blisters. It is painful to touch. This burn would be classified as a: 1/1 superficial thickness burn superficial partial thickness burn deep partial thickness burn full thickness burn

Feedback types of burn injuries are chemical, electric, radiation, or thermal. They are classified by the depth of damage skin. Symptoms of superficial partial thickness Burns include moist areas that are red to Ivory white in color,immediate blister formation and painful to touch (pain receptors are intact). superficial thickness Burns appear as erythematous without blisters and usually have local pain. Deep partial thickness Burns have a dry, waxy, whitish appearance and resemble full thickness Burns. sometimes crafts are needed. Full thickness Burns involve the destruction of all skin elements with coagulation of subdermal plexus, muscle, and or tendons.

in doing a physical assessment on a newborn, the family nurse practitioner notes a hip click. What other findings are associated with this condition? 1/1 shortened quadriceps lateral deviation of patella limited abduction lax hamstrings

Feedback typical findings include ortolani's sign which is the hip click,limited abduction, shortening of the extremity on the affected side, and asymmetric gluteal fold. The Lax hamstrings allows for full extension of the hip when the knee is fully flexed. This child may need further evaluation with a hip ultrasound for congenital hip dysplasia. Babies at highest risk are females, breech delivery, first born, family history of congenital hip dysplasia, or oligohydramnios

the most likely cause of bilateral purulent conjunctivitis in a one-day old infant is: 0/1 Symphilitic opthalmia adenovirus infection chlamydia infection Gonorrheal opthalmia

Gonorrheal opthalmia Feedback gonorrhea usually appears in the first three to five days of Life. Chlamydia is between days 5 and 14. The other two are less common in neonates. Most common causes of conjunctivitis in newborns includes, chlamydia, gonorrhea, hsve, and chemical induced conjunctivitis.

Constitutional growth delay

Growth velocity is normal after 3 years of age Delayed puberty with pubertal growth spurt Delayed bone age Positive family history

Hereditary Spherocytosis

Hereditary spherocytosis (HS) is a hemolytic anemia characterized by a deficiency or abnormality of the RBC membrane protein spectrin, which reduces the RBC surface area.

Bronchiolitis

Inflammation of the bronchioles that usually occurs in children younger than 2 years and is often caused by the respiratory syncytial virus.

Premature thelarche

It is important to note that for infants and toddlers with suspected premature thelarche, growth acceleration is not present, bone age is consistent with chronologic age, and there are no other signs of puberty.

· Discuss Fluid replacement in pediatric populations including PO and IV - -240 mL ORS for each

Lactated Ringer solution or normal saline a intravenously in boluses of 20 mL/kg body weight until perfusion and mental status improve./ >10 kg body weight: 120

Long acting

Long acting Duration: Up to 24 h • Detemir (Levemir) • Glargine (Lantus) • Degludec (Tresiba) (duration up to 42 h per manufacturer)

patient presents to the primary clinic complaining of a foul smelling vaginal discharge that worsens before menses and after intercourse. The advanced practice registered nurse (APRN) assesses the patient's diagnostic tests which reveals a vaginal pH >4.5, positive whiff test, and clue cells on wet prep. A diagnosis of bacterial vaginosis is confirmed. Which treatment regimen should the APRN use with this patient? Metronidazole 500 mg PO bid for seven days Ceftriaxone 250 mg IM once Cefoxitin 2 g IM once Levofloxacin 500 mg qd for seven days

Metronidazole 500 mg PO bid for seven days

Neurofibromatoses

Neurofibromatoses are a group of genetic disorders that cause tumors to form on nerve tissue. Six or more café au lait macules greater than 5 mm in greatest diameter in prepubertal individuals and greater than 15 mm in greatest diameter in post pubertal individuals • Two or more neurofibromas of any type or one plexiform neurofibroma • Freckling in the axillary or inguinal regions • Optic glioma • Two or more Lisch nodules (iris hamartomas) • A distinctive osseous lesion such as sphenoid dysplasia or tibial pseudarthrosis

orthostatic proteinuria

Occurs in young kids...increased protein excretion in the standing position, but normal in the supine position Take 2 samples of urine..one day time (which should be abnormal) ...one night time...which should have normal urine protein

• Noonan syndrome

PTPN11, SOS1, KRAS, RAF1 (AD) facial features, renal malformation, bleeding disorders Sequencing in the above order

Phimosis

Phimosis- Phimosis refers to a foreskin that is too tight to be retracted over the glans penis ersistent phimosis can be treated with 0.05% betamethasone cream twice daily for 2 to 4 weeks. This frequently allows successful retraction of the foreskin, promotes awareness of improved hygiene, and offers an alternative to circumcision

Premature adrenarche

Premature adrenarche is the early onset of pubic or axillary hair in either boys (prior to 10 years old) or girls (prior to 8 years old) not associated with other features of true puberty.

An advanced practice registered nurse (APRN) examines a patient who is four-weeks postpartum. The patient is complaining of low-grade fever, chills, and a right breast that is red, indurated, and painful. Which action should the APRN take to assist this patient? Recommend aspirin with limited breastfeeding to the right side Prescribe dicloxacillin 500 mg orally every six hours for 7-10 days Order penicillin G 5 million units IM and then an oral ampicillin for 10 days Stop lactation immediately and collect blood cultures

Prescribe dicloxacillin 500 mg orally every six hours for 7-10 days

A patient presents to an advanced practice registered nurse (APRN) for a prenatal check-up. The APRN completes an assessment and notes the following: B/P 132/78 pulse 84 respirations 22 glucose 105 mg/dL protein 9.5 g/dL creatinine 20 mg/kg/day creatinine clearance 100ml/min Which lab test is of concern? Creatinine clearance Glucose Creatinine Protein

Protein

An advanced practice registered nurse (APRN) is caring for a newly admitted Filipino patient and needs to collect a history prior to starting a care plan. Which fact should the APRN understand about the Filipino culture prior to collecting this data? Handshakes are discouraged as a greeting. Family members will provide opinions about care without prompting. Patients prefer to be called by their first name. Providers should sit, attempt to maintain eye contact, and allow periods of silence.

Providers should sit, attempt to maintain eye contact, and allow periods of silence.

A patient in her last trimester of pregnancy is being seen by an advanced practice registered nurse (APRN) for cold symptoms. The patient discusses the over-the-counter (OTC) medications she uses at home. The APRN notes that the patient has a history of mild hypertension that began two months earlier and decides to provide education on which OTC should be avoided during pregnancy. Which OTC medication should be of concern under these conditions? Loratadine Pseudoephedrine Dextromethorphan Guaifenesin

Pseudoephedrine

protienuria

Qualitative protein in urine, as tested by dipstick, is considered a positive result if it registers 1+ (30 mg/dL) or more in urine with a specific gravity of less than 1.015. Quantitative protein is tested by measuring a volume of urine over a set period. A level of less than 4 mg/m2/h is considered normal, 4 to 40 mg/m2/h is abnormal, and greater than 40 mg/m2/h indicates nephritic disease.

a 15 year old male patient reports a three-day history of sore throat, fever, malaise. Physical examination reveals pharyngitis, anterior cervical adenopathy, and oral temperature of 101.6 Fahrenheit. Which diagnostic test does the family nurse practitioner order next? 0/1 Antistreptolysin O titer cytomegalovirus titer monospot and complete blood count Rapid streptococcal antigen test Correct answer

Rapid streptococcal antigen test Feedback classic symptoms of strep pharyngitis include rapid onset of fever, sore throat, anterior cervical adenopathy, abdominal pain, nausea, vomiting, and headache. The most effective way to test for this disease process safely and quickly as the rapid streptococcal antigen test. The monospot tests for EBV and usually is a more Insidious onset with severe symptoms. Cytomegalovirus usually affects the GI tract primarily. And Antistreptolysin O titer detects antibodies to strep antigens and don't rise until one two three weeks after infection.

An elderly patient with a recent history of several falls is discussing driving privileges with an advanced practice registered nurse (APRN). The patient mentions the desire to remain both mobile and independent. Which initial action should the APRN take to assist this patient? Refer to registry of motor vehicles for a vision screening Report to authorities for unsafe driving Call the family to discuss the patient's concerns Refer to a rehabilitation facility for screening by occupational therapist

Refer to a rehabilitation facility for screening by occupational therapist

Legg-Calvé-Perthes Disease

Referral to an orthopedist is required for the management of LCPD. There are various approaches to the management, and treatment remains controversial. The general approach is guided by the principle of containment of the femoral head within the acetabulum. To be successful, containment must be instituted while the femoral head is still moldable. Nonoperative containment can be achieved in a v

A non-English speaking patient is seeing an advanced practice registered nurse (APRN) for chronic back pain. The patient speaks Spanish and arrived at the clinic with a bilingual spouse and child. The APRN would like to decrease the language barrier and ensure understanding. Which action should the APRN take to meet this goal? Have a clinic nurse who speaks some Spanish speak with the patient Ask the bilingual, adult spouse to assist with translation Request a professional interpreter who has healthcare experience Use a Spanish-English dictionary to ensure accurate word translations

Request a professional interpreter who has healthcare experience

A 15-year-old female presents to the clinic with the complaint of amenorrhea. The advanced practice registered nurse (APRN) completes an assessment with the following results: breast development normal recent weight loss of 15 lb with BMI of 16.5 kg/m2 no drug use The patient reports being on the track team in high school and runs five miles every day as part of the sport. Which care plan should the APRN recommend for this patient? Review complications, refer to counseling, and refer to nutritionist Recommend frequent meals with simple carbohydrates and prescribe spironolactone 100 mg bid Prescribe vitamin E 1000 IU qd, monitor progestin level every three months, and apply ice packs to lower abdomen Prescribe medroxyprogesterone 20 mg daily and monitor FBG and GTT annually

Review complications, refer to counseling, and refer to nutritionist

Scheuermann kyphosis

Scheuermann kyphosis is defined by vertebral wedging of 5 degrees or more on three adjacent vertebral bodies visualized on a standing lateral radiograph of the thoracic and lumbar spine.

An advanced practice registered nurse (APRN) is seeing a 35-year-old male patient who is complaining about a loss of libido and erectile dysfunction. The initial laboratory findings from the CBC, CMP, TSH, and PSA are within normal limits, but the patient has a low testosterone level. Which test should the APRN perform next? Lactate dehydrogenase Serum prolactin Serum iron Alpha-fetoprotein

Serum prolactin

Growth hormone deficiency s/s

Slow growth rate with normal birth weight Signs and symptoms of increased intracranial pressure Microphallus Proportional short stature Delayed bone age

An advanced practice registered nurse (APRN) is conducting the physical portion of a Medicare wellness exam. The APRN determines that it is necessary to measure the patient's gait and observe the patient walking and performing maneuvers. Which assessment should the APRN perform to obtain this information? TUG BADLs FRAIL ARE

TUG

Talipes equinovarus (clubfoot)

Talipes equinovarus (clubfoot) has three elements: the ankle is in equinus (the foot is in a pointed-toe position), the sole of the foot is inverted as a result of hindfoot varus or inversion deformity of the heel, and the forefoot has the convex shape of MA (forefoot adduction). At birth, the foot cannot be manually corrected to a neutral position with the heel down. Refer to an orthopedist upon diagnosis, ideally shortly after the infant is born, because the joints are most flexible in the first hours and days of life. Nonoperative treatment should be initiated as soon as possible after birth. Treatments include taping and strapping, manipulation, and serial casting. The Ponseti method of clubfoot treatment involves a specific technique for manipulation and serial casting. Weekly cast changes are performed; 5 to 10 casts are usually required. Up to 90% of children will need a percutaneous tenotomy of the heel cord as an outpatient followed by a long leg cast with the foot in maximal abduction and dorsiflexion. This is followed by a full-time bracing program for 3 months and then nightly bracing for 3 to 5 years. For older children with untreated club feet or for those who have residual deformity, osteotomies may be required in addition to the soft tissue surgery

A patient who is at 11 weeks gestation presents to the emergency department with a laceration from a rusty fence. An advanced practice registered nurse (APRN) assesses the laceration and learns that the patient's last Td vaccination was 12 years earlier. Which vaccine protocol should the APRN use for this patient? Administer Td at 27 weeks Administer Tdap at the current visit Administer Td at the current visit Administer Tdap at 27 weeks

Td (a booster vaccine for tetanus and diphtheria) should not be given at the current visit or at any time during pregnancy because it will not protect the newborn infant against pertussis.Correct! In circumstances where the patient has a wound requiring the vaccination, the recommendation is to administer a Tdap (a vaccine against tetanus, diphtheria, and pertussis) on the day of the visit. The vaccine would not need to be readministered later in the pregnancy.

Gender role conformity

The extent to which one adheres to cultural norms for his or her anatomical sex

sickle cell

The following laboratory results are seen in sickle cell disease: • Hct of 20% to 29% • Hgb 6 to 10 g/dL (severe) • Reticulocyte count elevated: 5% to 15% • Normal to increased WBCs and platelets • MCV greater than 80 fL; mean corpuscular hemoglobin concentration (MCHC) greater than 37 mg/dL • Hgb electrophoresis (after infancy), isoelectric focusing, or high-performance liquid chromatography showing a predominance of Hgb S and no Hgb A • Morphology: Irreversibly sickled cells or chronic elliptocytes, Howell-Jolly bodies, nucleated RBCs

IDA

The typical profile for IDA is • Microcytic, hypochromic RBCs on CBC • Low or normal MCV; low to normal RBC number • High RDW (>14%), low ferritin, high TIBC • Mentzer index greater than 13 (IDA more likely)IDA-;

Croup (laryngotracheobronchitis) is the most common cause of upper airway obstruction in children.

Therapy depends on the cause, severity, and location of the disease. The aim of therapy is to provide adequate respiratory exchange STERIODS

An advanced practice registered nurse (APRN) is caring for a woman who is six-months pregnant. The patient is complaining of dysuria and frequency. A urine dip suggests a urinary tract infection. Which plan of care should the APRN use for this patient? Order a non-stress test to assess for preterm labor Treat with an appropriate antibiotic Have the patient see her ob-gyn for treatment Avoid antibiotics and wait for the final urine culture

Treat with an appropriate antibiotic

A 26-year-old female patient presents to the clinic with complaints of abdominal pain, vaginal discharge, and vaginal pruritis. The advanced practice registered nurse (APRN) performs a pelvic exam, including pH tape test and wet mount. The APRN notes a vaginal pH of 6.2 and visualizes ovoid, flagellated organisms under the microscopy. What should the APRN diagnose? Chlamydia Candidiasis Trichomoniasis Bacterial vaginosis

Trichomoniasis

Acute laryngotracheitis

URI, seal-bark cough, mild to moderate dyspnea, symptoms worse at nigh Humidification, corticosteroids in selected cases

A 24-weeks-gestation pregnant woman presents to your office with new onset hypertension, with a blood pressure of 154/88. Her urine is negative for protein, but her serum creatinine level is rising. True or False This patient meets the criteria for preeclampsia. True False

When evaluating a pregnant woman with new-onset hypertension who does not have proteinuria, preeclampsia can be diagnosed if there is evidence of one or more of the following abnormalities: thrombocytopenia or disseminated intravascular coagulation (DIC), elevated transaminases or other signs of hepatic injury, CNS symptoms, an elevated or rising serum creatinine level, or pulmonary edema.

Wilms tumor

a rare type of malignant tumor of the kidney that occurs in young children

A child reports itching in both ears and is having trouble hearing. The Family Nurse practitioner notes periauricular edema and marked swelling of the external auditory canal and elicits severe pain when manipulating the external ear structures. Which is an appropriate intervention? 1/1 Obtain a culture of the external auditory canal. Order ototopical antibiotic/corticosteroid drops Prescribe oral amoxicillin-clavulanate. Refer the child to an otolaryngologist.

b

Chondrodystrophy

congenital condition cartilage converted to bone (dwarfism)

epididymitis-

epididymitis- Epididymitis is an inflammation of the epididymis that is painful, acute, and commonly caused by Neisseria gonorrhoeae or Chlamydia trachomatis in the sexually active adolescent, often with infection in the urethra or bladder.; First line: Ceftriaxone

Nephrotic syndrome

is due to excessive excretion of protein in urine as a result of alterations in the integrity of the glomerular filtration barrier. The main mechanism of the massive protein loss is increased glomerular permeability. The loss can be selective (albumin only) or nonselective (including most serum proteins), and such selectivity is an important distinction in diagnosis. The classic definition of nephrotic syndrome is massive proteinuria (3 to 4+ protein with UA, greater than 40 mg/m2/h or a protein:creatinine ratio on a first morning void of greater than 2 to 3:1), hypoalbuminemia (less than 2.5 g/dL), edema, and hyperlipidemia. Prednisone (2 mg/kg/day; maximum 60 mg) to induce remission, which can occur as early as 14 days as evidenced by diuresis. Steroids are continued for at least 4 to 6 weeks (Pais and Avner, 2016b). Relapses are treated with a short course of steroids and the patient is weaned as soon as the proteinuria resolves. •

acute prostatitis

levofloxacin or ciprofloxacin may be selected for treatment. Intravenous therapy is changed to oral therapy when the patient is afebrile for 24 to 48 hours and able to tolerate oral intake.

Slipped Capital Femoral Epiphysis

occurs when the head of the femur becomes displaced due to a separation at the growth plate

UTI when to refer

· -protein in ua

galactosemia

Galactosemia is a rare, hereditary disorder of carbohydrate metabolism that affects the body's ability to convert galactose to glucose.. Measurement of GALT activity in red cells will be deficient, and liver enzymes and gal-1-P levels will be elevated

SLE is rare in children and has a general onset of ___ to ___ years of age. 0/1 11-12 5-6 1-4 18-21

A

Intermediate acting

Duration: 12-24 h • NPH

Short acting

Duration: 5-8 h • Regular insulin

Gender identity disorder

a strong cross-gender identification characterized by the desire to be the other sex, combined with persistent discomfort about one's biological sex or culturally prescribed gender role

the family nurse practitioner knows that climate change will impact the main Health outcome for children by its impact on: 1/1 cost of living cost of Education food production smog

C

An advanced practice registered nurse (APRN) suspects that a child has leukemia. Which diagnostic study should be used by the APRN to support this diagnosis? A metabolic panel Chest X-ray Thyroid levels CBC with differential

D

59 A large healthcare system wants to improve communication between patients, patients' families, and the healthcare system in order to reduce litigation costs by 20% by the end of the fiscal year. To reach this goal, the system has designated a single representative to communicate with patients and families immediately following each adverse event. An advanced practice registered nurse (APRN) in the quality department has been assigned to continuously monitor this project intervention. What data should the APRN monitor to determine if this goal is being met? Number of spokespeople for each event Number of stakeholders affected Number of litigations for each event Number of events during the fiscal year

A

A 14-year-old boy who is overweight develops a unilateral limp with pain in the hip and knee on the affected side. An exam reveals external rotation of the hip when flexed and pain associated with attempts to internally rotate the hip. What is most important initially when managing this child's condition? 1/1 Place the child on crutches or in a wheelchair to prevent weight-bearing Provide information about weight loss to minimize further injury. Recommend seeing an orthopedic specialist as soon as possible. Refer the child to physical therapy to improve range of motion.

A

A 2-year-old child who has sickle cell anemia (SCA) comes to the clinic with a cough and a fever of 101.5°C. The child currently takes penicillin V prophylaxis 125 mg orally twice daily. What will the Family nurse practitioner do? 0/1 Admit the child to the hospital to evaluate for sepsis. Give intravenous fluids and antibiotics in clinic. Increase the penicillin V dose to 250 mg. Order a chest radiograph to rule out pneumonia.

A

A 4-year-old child has just been released from the hospital after orthopedic surgery on one leg following a bicycle accident. The child is sitting quietly on the exam table. When asked to rate pain, the child points to the "1" on a faces rating pain scale. What will the Family nurse practitioner do next? 1/1 Assess the child's vital signs and ability to walk without pain. Refill the prescription for a narcotic analgesic medication. Suggest that the parents give acetaminophen for mild pain. Teach the parent to give analgesics based on the child's report of pain.

A

A child has gross hematuria, abdominal pain, and arthralgia as well as a rash. What diagnosis is most likely? 1/1 Henoch-Schönlein purpura Rhabdomyosarcoma Sickle cell disease Systemic lupus erythematosus

A

A child is diagnosed with community-acquired pneumonia and will be treated as an outpatient. Which antibiotic will the Family nurse practitioner prescribe? 1/1 Amoxicillin Azithromycin Ceftriaxone Oseltamivir

A

A child who plays soccer is in the clinic reporting pain and swelling in both knees. A physical examination reveals swelling and focal tenderness at the tibial tuberosities, with pain worsening when asked to extend the knees against resistance. What is the treatment for this condition? 0/1 Apply ice packs to both knees and avoid activities that cause pain. Begin quadriceps-stretching exercises now to prevent further injury. Obtain radiographic studies to rule out fractures or ligament tears. Refer to a pediatric orthopedic specialist to evaluate the need for surgery. Correct answer

A

A mother has brought in her teenage daughter for a well-child appointment. The teenager has a history of Down syndrome with atlantoaxial instability. The mother asks an advanced practice registered nurse (APRN) about the possibility of the child participating in sports. Which sport should the APRN suggest for this patient? Running Gymnastics Soccer Diving

A

A patient with a history of an acute adrenal insufficiency presents to the clinic. The advanced practice registered nurse (APRN) assesses that the patient has increased edema and weight gain from the corticosteroid therapy. Which action should the APRN take next for this patient? Consult with an endocrinologist Order an ACTH stimulation test Discontinue the corticosteroid Schedule a CT of the adrenal glands

A

A school-age child has had abdominal pain for 3 months that occurs once or twice weekly and is associated with a headache and occasional difficulty sleeping, often causing the child to stay home from school. The child does not have vomiting or diarrhea and is gaining weight normally. The physical exam is normal. According to Bishop, what is included in the initial diagnostic work-up for this child? 1/1 CBC, ESR, amylase, lipase, UA, and abdominal ultrasound CBC, ESR, CRP, and fecal calprotectin CBC, ESR, CRP, UA, stool for ova, parasites, and culture Stool for H. pylori antigen and serum IgA, IgG, tTg

A

An advanced practice registered nurse (APRN) examines a nine-year-old patient with complaints of ear pain for two days. The patient's parent reports that the patient is on a local swim team and is in the water multiple times a week. On examination, pain is elicited with movement of the tragus and when the pinna is pulled. The tympanic membrane is not visual due to swelling of the canal. What should the APRN recommend for this patient? Drops containing antibiotic with or without corticosteroid to treat acute otitis externa Irrigation and broad-spectrum antibiotics to treat the presence of a foreign body Antipyrine/benzocaine/u-policosanol drops and oral antibiotic to treat acute otitis media Oral antibiotic and corticosteroid drops to treat chronic otitis externa

A

An advanced practice registered nurse (APRN) examines a seven-year-old for chest pain that developed five days earlier, right after a cold and cough began. Respiratory effort and auscultation of the lungs, heart, and abdomen are within defined limits. Palpation of the anterior chest reveals tenderness over the costochondral junction and is localized at the sternocostal cartilage of the third, fourth, and fifth ribs. Which diagnosis should this APRN suspect? Tietze syndrome Rib fracture Pneumonia Rheumatic disease

A

An advanced practice registered nurse (APRN) is completing a neurological developmental assessment for an eight-year-old child. Which assessment finding should concern this APRN? Absence of exhibiting objective thought Lack of abstract thinking Inability to define self-concept Presence of role diffusion that causes conflict

A

An advanced practice registered nurse (APRN) is educating an adolescent patient about ways to decrease a modifiable risk factor for developing liver cancer. Which behavior should this APRN target? Using condoms consistently Receiving an influenza vaccination Reducing alcohol consumption Eating a raw vegetable diet

A

An advanced practice registered nurse (APRN) is evaluating a two-year-old for a "barky" cough and fever of 103°F (39.4°C). The APRN notes inspiratory stridor and purulent sputum. A lateral neck film shows subglottic narrowing, and a CBC shows a high WBC count with a left shift. Which differential diagnosis should the APRN make for this patient? Laryngotracheobronchitis Acute laryngotracheitis Epiglottis Diphtheria

A

An advanced practice registered nurse (APRN) is examining a 12-year-old patient, and the parent expresses concern because the child is asking questions about sex. How should this APRN respond? "Understand that human sexuality is a natural part of life." "It is inappropriate for children this age to ask adults about sex." "Recognize that all creatures grow and reproduce." "Be aware that sexuality is important at all ages."

A

An advanced practice registered nurse (APRN) is examining a 17-year-old patient who will be entering college in the fall. The patient's parent wants to know which vaccines are recommended prior to enrollment. Which vaccine should this APRN recommend? Tetanus, diphtheria, and acellular pertussis (Tdap) vaccine Meningococcal vaccine Booster human papillomavirus (HPV) vaccine Updated measles, mumps, and rubella (MMR) vaccine

A

An advanced practice registered nurse (APRN) is examining a four-year-old. The family recently adopted another child, and the parent mentions that there are issues with sibling rivalry. The APRN asks about a condition that often develops when a new sibling enters a family. Which condition is being assessed by this APRN? Nocturnal enuresis Anorexia Depression Speech delay

A

An advanced practice registered nurse (APRN) is treating a seven-year-old for recurrent shoulder pain. The pain has not improved after numerous intervention attempts over the past month. Which action should the APRN take next for this patient? Refer to an orthopedic surgeon Prescribe pain medication Refer to physical therapy Prescribe an NSAID

A

An advanced practice registered nurse (APRN) is using a growth chart for a pediatric patient and finds the child's weight-to-length ratio is near the eighth percentile. Suboptimal nutrition Cushing syndrome Parental discord Chronic renal insufficiency

A

An eight-year-old child presents to an advanced practice registered nurse (APRN) for recurrent episodes of emesis, which has been getting worse since the child began four years earlier. The child's parent says the child typically throws up several times in a short period, and each episode can last for several hours before it abruptly stops. The emesis is usually yellow in color, and the child becomes pale. The child has nausea, severe abdominal pain, and a headache, and light is painful to the eyes. The APRN notes a family history of migraine headaches. Which differential diagnosis should the APRN make for this child? Cyclic vomiting syndrome Gastroesophageal reflux disease Appendicitis Pyloric stenosis

A

During a well child exam of a school-age child, the Family nurse practitioner learns that the child has been having angry episodes at school. The nurse practitioner observes the child to appear withdrawn and sad. Which action is appropriate? 1/1 Ask the child and the parent about stressors at home Make a referral to a child behavioral specialist Provide information about anger management Suggest consideration of a different classroom

A

During a well child examination, a 15-year-old female tells the Family nurse practitioner that some of her friends have begun having sex. She has a boyfriend but denies engaging in sex with him. What will the nurse practitioner do initially? 1/1 Ask her for her definitions of "sex." Discuss the risks of sexually transmitted diseases. Find out if she is considering sexual relations. Give her information about contraception.

A

During an assessment of a 4-week-old infant, the Family nurse practitioner learns that a breastfed infant nurses every 2 hours during the day but is able to sleep for a 4-hour period during the night. The infant has gained 20 grams per day in the interval since last seen in the clinic. What will the nurse practitioner recommend? 1/1 Continuing to nurse the infant using the current pattern Nursing the infant for longer periods every 4 hours Supplementing with formula at the last nighttime feeding Waking the infant every 2 hours to nurse during the night

A

The Family nurse practitioner (FNP) is performing a well child examination on a school-age child who had complete repair of a tetralogy of Fallot (TOF)defect in infancy. What is important in this child's health maintenance regime? 1/1 Cardiology clearance for sports participation Restriction of physical activity to avoid pulmonary complications Sub-acute bacterial endocarditis prophylaxis precautions Teaching about management of hypercyanotic episodes

A

The Family nurse practitioner (FNP) is performing a well child examination on a school-age child who had complete repair of a tetralogy of Fallot (TOF)defect in infancy. What is important in this child's health maintenance regime? 1/1 Cardiology clearance for sports participation Restriction of physical activity to avoid pulmonary complications Sub-acute bacterial endocarditis prophylaxis precautions Teaching about management of hypercyanotic episodesA

A

The Family nurse practitioner (PNP) is examining a 2-week-old infant and auscultates a wide splitting of S2 during expiration. What condition may this finding represent? 0/1 Atrial septal defect (ASD Coarctation of the aorta (COA) Patent ductus arteriosis (PDA) Ventricular septal defect (VSD)

A

The Family nurse practitioner cares for a preschool-age child who was exposed to drugs prenatally. The child bites other children and has tantrums when asked to stop but is able to state later why this behavior is wrong. This child most likely has a disorder related to what process? 1/1 executive function information processing sensory processing social congnition

A

The Family nurse practitioner evaluates a 5-year-old child who presents with pallor and obtains labs revealing a hemoglobin of 8.5 g/dL and a hematocrit of 31%. How will the nurse practitioner manage this patient? 0/1 Prescribe elemental iron and recheck labs in 1 month. Reassure the parent that this represents mild anemia. Recommend a diet high in iron-rich foods. Refer to a hematologist for further evaluation.

A

The Family nurse practitioner is considering use of a relatively new drug for a 15-month-old child. The drug is metabolized by the liver, so the nurse practitioner will consult a pharmacologist to discuss giving the drug: 0/1 less often or at a lower dose. more often or at a higher dose. via a parental route via the oral route.

A

The Family nurse practitioner is counseling the parents of a 13-year-old female who has Down syndrome about sexual maturation. What will the nurse practitioner tell these parents? 1/1 It is important to discuss and support healthy sexuality Providing too much information about sexuality may be confusing given the child's cognitive level of understanding. Suppressing periods with contraceptives will lessen their daughter's distress. They should give her information about periods but not about sexuality.

A

The Family nurse practitioner is discussing fitness and exercise with the parents of a 5-year-old child who ask what kinds of activities are developmentally appropriate for their child. What will the nurse practitioner recommend? 1/1 Bike riding Interactive play Martial arts Organized Sports

A

The Family nurse practitioner is discussing lifestyle changes with an adolescent who has hypertension. What will the nurse practitioner recommend about exercise for this client? 0/1 Regular to vigorous activity initially with a combination of resistance and aerobic exercise to maintain lower blood pressure Moderate daily exercise such as walking for 20 minutes daily with increasing intensity as blood pressure drops Vigorous aerobic exercise combined with maximal strength training to lower blood pressure Vigorous aerobic exercise only to reduce blood pressure and then to maintain lowered blood pressure

A

The Family nurse practitioner is evaluating recurrent stomach pain in a school-age child. The child's exam is normal. The nurse practitioner learns that the child reports pain most evenings after school and refuses to participate in sports but does not have nausea or vomiting. The child's grandmother recently had gallbladder surgery. Which action is correct? 1/1 Encourage the child to keep a log of pain, stool patterns, and dietary intake Order radiologic studies and laboratory tests to rule out systemic causes Reassure the child and encourage resuming sports when symptoms subside Refer the child to a counselor to discuss anxiety about health problems

A

The Family nurse practitioner is examining a 5-year-old child who has had recurrent fevers, bone pain, and a recent loss of weight. The physical exam reveals scattered petechiae, lymphadenopathy, and bruising. A complete blood count shows thrombocytopenia, anemia, and an elevated white cell blood count. The nurse practitioner will refer this child to a specialist for what diagnostic testing? 1/1 bone marrow biopsy corticosteroids and intravenous immunogloblin (IVIG) hemoglobin electrophoresis d. immunoglobulin testing watchful waiting

A

The Family nurse practitioner is examining a school-age child who reports frequent stomach pain and headaches. The parent reports that the child misses several days of school each month. The child has a normal exam. Before proceeding with further diagnostic tests, what will the nurse practitioner initially ask the parent? 1/1 About the timing of the symptoms each day and during the week How well the child performs in school and in extracurricular activities If the parent feels a strong need to protect the child from problems Whether there are any unusual stressors or circumstances at home

A

The Family nurse practitioner is performing a well child exam on a 17-year-old female whose mother is present during the history. The mother expresses concern that her daughter wishes to have an eyebrow piercing and states that she is opposed to the idea. What will the nurse practitioner do? 1/1 Provide information about piercings and encourage continued discussion. Remind the adolescent that her mother is responsible for her health. State that piercings are relatively harmless and are an expression of individuality. Suggest that she wait until she is 18 years old and can make her own decisions.

A

The Family nurse practitioner is reviewing lab work and diabetes management with a school-age child whose HbA1C is 7.6% who reports usual blood sugars before meals as being 80 to 90 mg/dL. The nurse practitioner will consult with the child's endocrinologist to consider which therapy? 1/1 Continuous glucose monitoring Continuous subcutaneous insulin infusion Self-monitoring of blood glucose Use of a long-acting insulin analogue

A

The Family nurse practitioner performs a physical examination on a 12-year-old child and notes poor hygiene and inappropriate clothes for the weather. The child's mother appears clean and well dressed. The child reports getting 6 to 7 hours of sleep each night because of texting with friends late each evening. What action by the nurse practitioner will help promote healthy practices? 1/1 Discuss setting clear expectations about self-care with the mother Give the child information about sleep and self-care Reassure the mother that this "non-compliance" is temporary Tell the mother that experimenting with self-care behaviors is normal

A

The Family nurse practitioner reviews hematology reports on a child with beta-thalassemia minor and notes an Hgb level of 8 g/dL. What will the nurse practitioner do? 0/1 Evaluate serum ferritin. Order Hgb electrophoresis. Prescribe supplemental iron. Refer for RBC transfusions.

A

The mother of a 6-month-old infant is distressed because the infant can say "dada" but not "mama" and asks the Family nurse practitioner why this is when she is the one who spends more time with the infant. How will the nurse practitioner respond? 1/1 "At this age, your baby does not understand the meaning of sounds." "Babies at this age cannot make the 'ma' sound." "Most sounds made by babies at this age are accidental." "This may mean that your baby doesn't hear well."

A

The parent of an adolescent reports noting cutting marks on the teen's arms and asks the Family nurse practitioner what it means. What will the nurse practitioner tell this parent? 1/1 Cutting is a way of dealing with emotional distress. It is a method of fitting in with other adolescents. The behavior is common and will usually stop. This type of behavior is a type of suicide attempt.

A

What will the Family nurse practitioner teach the parents of a child who has new pressure-equalizing tubes (PET) in both ears? 0/1 Parents should notice improved hearing in their child. PET will help by reducing the number of ear infections the child has. The child should use earplugs when showering or bathing. The tubes will most likely remain in place for 3 to 4 years.

A

a 10 week old premature infant, who was not immunized before being discharged from the hospital, is seen for the first time in your office. What immunizations should the provider administer today? 1/1 Haemophilus influenzae type B, diphtheria - tetanus - pertussis, Hepatitis B, inactivated polio vaccine, Roto virus, pneumococcal oral polio vaccine, diphtheria-tetanus-pertussis, haemophilus Haemophilus influenzae type B, diptheria-tetanus-pertussis, rotavirus, inactivated polio vaccine, penumococcal Inactivated polio vaccine, haemophilus influenzae type B, diptheria-tetanus-pertussis, pneumococcal

A

he Family nurse practitioner is performing a pre-participation sports physical examination on a 14-year-old male who will be on the wrestling team at school. What will the nurse practitioner include when discussing healthy practices with this adolescent? 1/1 Risks associated with repeatedly losing and gaining weight The need for an electrocardiogram or echocardiogram prior to participation The need to consume 20 to 30 grams of protein after exercise To consume water with CHO prior to activity lasting up to an hour

A

a parent without health insurance brings an infant with bilateral otitis media to a pediatric clinic. To optimize treatment adherence, the nurse practitioner prescribes which oral medication? 1/1 amoxicillin (Amoxil) two times daily for 10 days amoxicillin/clavulanate (Augmentin) twice daily for 10 days azithromycin (zithromax) once daily for 5 days cefdinir (omnicef) once daily for 10 days .

A Feedback amoxicillin is a very inexpensive medication with the lowest cost being around 3 to $4 compared to Augmentin which can cost four hundred to $900. Azithromycin and Cefdinir are 40 and $20 respectively. The lower cost of the medication is a factor. Also, there is more compliant with a one to two times a day medication. Finally, amoxicillin remains first-line therapy for acute otitis media. Augmentin is second-line therapy and Cefdinir is third line therapy. Azithromycin is not recommended for ear infections

the family nurse practitioner is completing a history and physical exam on a child whom she suspects May Be autistic. Which of the following findings is associated with autism? 1/1 delay in language development delay in physical growth overprotective parents who provide minimal social interaction for the child warm, cuddling child with excessive need for interaction

A Feedback autism is a developmental disorder that starts early in a child's life and is characterized by avoidance of eye contact, indifference to caregivers, language and communication delays, failure to develop a social smile, repetitive movements, and an excessive need for routine

which of the following would be pertinent in the past medical history of a child who is being evaluated for cardiovascular disease? 1/1 Kawasaki disease hypothyroidism osteogenic sarcoma Tourette syndrome

A Feedback the two major conditions known to play a causative role in the development of cardiovascular disease in children are untreated streptococcal infections including group A B-hemolytic streptococci (leads to cardiac valve dysfunction) and kawasaki disease (leads to coronary artery aneurysm)

exam of a non-toxic but ill appearing six-year-old revealed vesicular and ulcerative oral lesions and a maculopapular rash on the hands and feet. Temperature is 100.4 and the child has feelings of malaise. What does the differential diagnosis include? 1/1 hand foot and mouth disease roseola drug rash varicella

A Feedback these symptoms are indicative of Coxsackievirus 5 or 16 group A (usually A16) called hand foot and mouth disease. The lesions may appear in one or all three areas. Children are usually uncomfortable, but not seriously ill, and they refuse fluids with severe oral lesions. Varicella lesions begin on the trunk with the classic teardrop vesicles

which intervention is part of the recommended treatment for erythema multiforme minor? 0/1 oral antihistamine as a supportive measure oral steroid to resolve skin lesions topical antibiotic to prevent infection topical antifungal to treat skin lesions

A Feedback this condition is a self-limiting condition. In most cases, symptomatic treatment is sufficient. Oral antihistamines for 3 or 4 days may reduce the stinging and burning of the skin. Because the skin lesions are not related to a bacterial or fungal infection on the dermis, topical antibiotics and antifungals are not beneficial. Oral steroids are reserved for severe cases with functional impairments.

the family nurse practitioner is measuring a newborn's frontal occipital circumference. What does the correct technique involve? 1/1 placing the paper tape measure at the frontal-occipital prominence and just above the eyebrows placing the cloth tape measure at a level 2 inches above the ears using a cloth tape to prevent an accuracy caused by stretchable materials having another person hold the tape in the center of the forehead and repeating the measurement Feedback using a paper or a plastic tape, rather than cloth, prevents error because of the stretching of the fabric. Measurement should be repeated to confirm accuracy. The tape should not lie over the ears

A using a paper or a plastic tape, rather than cloth, prevents error because of the stretching of the fabric. Measurement should be repeated to confirm accuracy. The tape should not lie over the ears

Which of the following is the MOST appropriate intervention for 2-year-old patient who has not received the MMR vaccine? 1/1 Administer two doses 4 weeks apart Administer one dose and evaluate the patient's immunity Administer one dose now and the second dose at 4 years of age Administer two doses 4 weeks apart and then evaluate the patient's immunity Feedback

A The most appropriate intervention is to administer two doses 4 weeks apart. Administering one dose of the MMR vaccine is not sufficient. It is not necessary to evaluate the patient's immunity after administering any dose of the MMR vaccine.

the family nurse practitioner is examining a full-term infant who developed physiologic jaundice and is being treated with phototherapy. What is the mechanism of action of phototherapy in the treatment of this infant? 1/1 the light is absorbed by bilirubin and promotes the conversion of a toxic bilirubin to and unconjugated product that can be excreted in the bile it increases hemolysis of the excessive red blood cells that are received by the full-term infant during labor and delivery the ultraviolet light decreases sensitivity to the destruction of red blood cells secondary to the RH incompatibility it increases enzymatic activity in breaking down the unconjugated bilirubin to a non toxic for him to be eliminated by the kidney Feedback

A phototherapy is effective secondary to the absorption of the light by bilirubin across the infant skin. The light energy is absorbed by bilirubin and promotes the conversion of bilirubin to a non-toxic form that can be excreted in the bile and eliminated in the urine and stool. The increase in hemolysis will increase the bilirubin level, this is physiologic jaundice that is not associated with an RH incompatibility problem

her mother brings her 2 month old infant to a nurse practitioner clinic for a well-child examination. The nurse practitioner orders the immunizations and provides an overview to the mother. The mother States "I don't want my child to have those shots!" the nurse practitioners most appropriate response is: 1/1 provide a strong recommendation for immunization on schedule inform your mother that state law requires immunization negotiate an alternative immunization plan with the mother recommend that the mother found another provider who shares the mother's philosophy regarding vaccines Feedback

A the family nurse practitioner should assume that all parents will want to vaccinate based on recommendations. Providing a strong recommendation to vaccinate on schedule and adding supported statements is the best response to hesitation or initial refusal. Then listen to and respond to specific questions and concerns. And Alternate immunization plan is not the most appropriate initial response as a nurse practitioner does not have an understanding of why the mother is hesitant yet. Laws vary from state-to-state and recommending the mother find a different provider will prevent future chances to discuss immunizations

Which benign prostatic hyperplasia medication is contraindicated in those with a sulfa allergy? Silodosin Tamsulosin Finasteride Sildenafil

A small daily dose of sildenafil has been successfully used to treat BPH, but sildenafil does not contain sulfa. Tamsulosin should not be used in those with a sulfa allergy.

The mother of a 4-year-old child tells the pediatric nurse that the child's abdomen seems to be swollen. During further assessment, the mother tells the nurse that the child is eating well and that the activity level of the child is unchanged. The nurse, suspecting the possibility of Wilms' tumor, should avoid which during the physical assessment? 1/1 .Palpating the abdomen for a mass Assessing the urine for the presence of hematuria Monitoring the temperature for the presence of fever Monitoring the blood pressure for the presence of hypertension

AFeedback Wilms' tumor is the most common intraabdominal and kidney tumor of childhood. If Wilms' tumor is suspected, the tumor mass should not be palpated by the nurse. Excessive manipulation can cause seeding of the tumor and spread of the cancerous cells. Hematuria, fever, and hypertension are clinical manifestations associated with Wilms' tumor.

A lactating mother presents to your office with a red, painful, swollen area on her right breast, chills, and a fever of 101.6°F. What is an appropriate treatment plan for this patient? Culture the breast milk Azithromycin 500 milligrams (mg) for one dose, then 250 mg per day for four days Amoxicillin 500 mg TID Clindamycin 300 mg PO TID for 10-14 days

Amoxicillin does not have good coverage of the causative organisms for lactational mastitis, especially in cases of drug resistant staph aureus. Clindamycin 300 mg PO TID for 10-14 days is a recommended treatment course for lactation mastitis.

uti

Asymptomatic bacteriuria is bacteria in the urine without other symptoms, is benign, and does not cause renal injury. Cystitis is an infection of the bladder that produces lower tract symptoms but does not cause fever or renal injury. Pyelonephritis is the most severe type of UTI involving the renal parenchyma or kidneys and must be readily identified and treated because of the potential irreversible renal damage. Clinical signs thought to be consistent with pyelonephritis include fever, irritability, and vomiting in an infant, and urinary symptoms associated with fever, bacteriuria, vomiting, and renal tenderness in older children. UTIs are the most common cause of serious bacterial infection in infants younger than 24 months old with fever without a focus (Elder, 2016d). A complicated UTI is defined as a UTI with fever, toxicity, and dehydration, or a UTI occurring in a child younger than 3 to 6 months old.

1The Family nurse practitioner is performing a well child exam on a 12-year-old female who has achieved early sexual maturation. The mother reports that she spends more time with her older sister's friends instead of her own classmates. What will the nurse practitioner tell this parent? 1/1 Early-maturing girls need to identify with older adolescents to feel a sense of belonging. Girls who join an older group of peers may become sexually active at an earlier age. Spending time with older adolescents indicates a healthy adjustment to her maturing body. The association with older adolescents will help her daughter to gain social maturity.

B

A 3-year-old child is recovering from injuries sustained in a motor vehicle accident. How will the FNP nurse practitioner evaluate this child's pain? 1/1 Ask the child to rate pain intensity on a 4- to 5-item pain discrimination scale. Have the child describe any pain as "no pain, a little pain, or a lot of pain." Question the child about the intensity and specific location of any pain. Rely on nonverbal responses such as facial expressions and limb movements.

B

A 32-year-old LGBTQ patient is speaking with an advanced practice registered nurse (APRN) and mentions a concern that is universal for all genders and sexual identities. Which concern was mentioned by this patient? Hormone therapy Diet and exercise HPV Breast cancer screening

B

A 5-year-old child has an elevated blood pressure during a well child exam. The Family nurse practitioner notes mottling and pallor of the child's feet and lower legs and auscultates a systolic ejection murmur in the left infraclavicular region radiating to the child's back. The nurse practitioner will suspect which condition? 0/1 Aortic stenosis (AS) Coarctation of the aorta (COA) Patent ductus arteriosus (PDA) Pulmonic stenosis

B

A 5-year-old child has an elevated blood pressure during a well child exam. The Family nurse practitioner notes mottling and pallor of the child's feet and lower legs and auscultates a systolic ejection murmur in the left infraclavicular region radiating to the child's back. The nurse practitioner will suspect which condition? 1/1 Aortic stenosis (AS) Coarctation of the aorta (COA) Patent ductus arteriosus (PDA) Pulmonic stenosis

B

A 6-month-old infant has a retracted testis that was noted at the 2-month well baby exam. What will the Family nurse practitioner do to manage this condition? 0/1 Reassure the parent that the testis will most likely descend into place on its own. Refer the infant to a pediatric urologist or surgeon for possible orchiopexy. Teach the parent to manipulate the testis into the scrotum during diaper changes. Tell the parent that hormonal therapy may be needed to correct the condition.

B

A child diagnosed with attention-deficit/hyperactivity disorder (ADHD) has difficulty stopping activities to begin other activities at school. The Family nurse practitioner understands that this is due to difficulty with what self-regulation ability? 1/1 emotional control flexibility inhibition problem-solving

B

A child who has had a single non-febrile seizure has a normal neurologic exam. Which diagnostic test is indicated? 1/1 Computerized tomography (CT) Electroencephalogram (EEG) Magnetic resonance imaging (MRI) Polysomnography

B

A child who has nephrotic syndrome is on a steroids and a salt-restricted diet for a relapse of symptoms. A dipstick urinalysis shows 1+ protein, down from 3+ at the beginning of the episode. In consultation with the child's nephrologist, what is the correct course of treatment considering this finding? 0/1 Begin a taper of the steroid medication while continuing salt restrictions. Continue with steroids and salt restrictions until the urine is negative for protein. Discontinue the steroids and salt restrictions now that improvement has occurred. Relax salt restrictions and continue administration of steroids until proteinuria is gone.

B

A complete blood count on a 12-month-old infant reveals microcytic, hypochromic anemia with a hemoglobin of 9.5 g/dL. The infant has mild pallor with no hepatosplenomegaly. The Family nurse practitioner suspects what disorder? 1/1 hereditary spherocytosis iron deficiency anemia lead intoxication sickle-cell anemia

B

A dipstick urinalysis is positive for leukocyte esterase and nitrites in a school-age child with dysuria and foul-smelling urine but no fever who has not had previous urinary tract infections. A culture is pending. What will the Family nurse practitioner do to treat this child? 1/1 Order ciprofloxacin ER once daily for 3 days if the culture is positive. Prescribe trimethoprim-sulfamethoxazole (TMP) twice daily for 3 to 5 days. Reassure the child's parents that this is likely an asymptomatic bacteriuria. Wait for urine culture results to determine the correct course of treatment.

B

A preschool-age child with no previous history has mild flank pain and fever but no abdominal pain or vomiting. A urinalysis is positive for leukocyte esterase and nitrites. A culture is pending. Which is the correct course of treatment for this child? 1/1 Hospitalize for intravenous antibiotics. Order amoxicillin clavulanate. Prescribe trimethoprim-sulfamethoxazole. Refer for a voiding cystourethrogram.

B

A school-age child has a fractured wrist with a Salter-Harris Type II fracture, according to the radiologist. What is true about this type of fracture? 1/1 Growth disturbance of the long bones of the arm is likely. There is a metaphyseal fragment on the compression side of fracture. There is usually a compression or crushing injury to the physis. This will require anatomic reduction using an open approach.

B

A school-age child has begun refusing all cooked vegetables. What will the Family nurse practitioner recommend to the parent? 1/1 Allow the child to make food choices since this is usually a phase Ensure that the child has three nutritious meals and two nutritious snacks each day Prepare vegetables separately for the child to encourage adequate intake Teach the child how important it is to eat healthy fruits and vegetables

B

An adolescent has suspected infectious mononucleosis after exposure to the virus in the past week. The primary care pediatric nurse practitioner examines the adolescent and notes exudate on the tonsils, soft palate petechiae, and diffuse adenopathy. Which test will the Family nurse practitioner perform to confirm the diagnosis? 1/1 Complete blood count EBV-specific antibody testing Heterophile antibody testing Throat culture

B

An advanced practice registered nurse (APRN) examines a 10-month-old patient and notes the following: fever for two days waking up every couple of hours crying refusing to drink from the bottle pulling at the right ear right tympanic membrane that is erythematous and bulging, with decreased mobility What is the likely diagnosis? Mastoiditis Acute otitis media Foreign body Acute otitis externa

B

An advanced practice registered nurse (APRN) examines a 36-month-old child, and a piece of assessment data causes concern about the child's development. Which assessment finding caused this response by the APRN? Speech pattern not 100% understandable Problems with biting playmates Inability to identify a penny, nickel, or dime Inability to draw a person with a body

B

An advanced practice registered nurse (APRN) is conducting a follow-up appointment with an older patient who has recently been diagnosed with depression, constipation, and back pain. The APRN reviews the patient's medications and notes prescriptions for pain management on the list. Which recommendation should the APRN make to this patient? Drink at least five glasses of water per day and switch to lower fat foods Modify diet to include more fiber and increase fluid intake Encourage the patient to follow a low residue diet and keep a food diary Decrease fruit juices and soda intake

B

An advanced practice registered nurse (APRN) is educating an adolescent and her parent about the use of social media. Which information should be provided by this APRN? Using social media decreases depression by increasing peer interactions. Parents should be supervising the adolescent's social media usage. An adolescent should not use social media due to the dangers it poses. Social media is an effective way to network with diverse groups of people.

B

An advanced practice registered nurse (APRN) is examining a 12-year-old immigrant, and the patient mentions having problems with the food at school and having some cultural identity issues. Which cultural patient-centered question should be explored first? Whether meals at school are eaten with friends or eaten alone Which foods are preferred and whether they are cultural and religious preferences Whether fast-food restaurants or cultural restaurants are preferred Which snacks are brought to school and whether there are any economic issues

B

An advanced practice registered nurse (APRN) is examining a 12-year-old who has a gunshot wound to the hand. The APRN asks the patient about the injury, but the parent immediately intervenes and says it was a self-inflicted accident. Later, while the parent is completing the registration process, the patient whispers that the mother and father were fighting when the injury occurred. The APRN treats the patient, and then takes further action. Which additional action should the APRN take for this patient? Arrange for the family to have mental health counseling Notify Child Protective Services Refer the patient to psychiatric counseling Discharge home to the care of the parent

B

An advanced practice registered nurse (APRN) is examining a 16-year-old male patient who is sexually active for nausea, vomiting, fever, dysuria, and painful scrotal swelling. The APRN notes scrotal edema and erythema, a hard epididymis, induration, enlargement, and a tender spermatic cord. The testis has normal position and consistency, and the Prehn sign can be elicited. Which diagnosis should the APRN make for this patient? Testicular torsion Epididymitis Hydrocele Spermatocele

B

An advanced practice registered nurse (APRN) is examining a 16-year-old patient who has a history of exercise-induced asthma for a routine follow-up visit. The patient reports no coughing during the day or night, but the frequency of wheezing after running has increased. The patient runs track six days per week and is treating the symptoms with two puffs of albuterol. Which recommendation should the APRN make for this patient? Pretreat with two puffs of albuterol Begin inhaled corticosteroid therapy twice daily Discontinue treating exercise-induced wheezing Start leukotriene receptor antagonist nightly

B

An advanced practice registered nurse (APRN) is performing a sports physical on a female teen athlete and notes that the patient is in the 50th percentile of weight and height for her age. The APRN concludes the visit by providing sports nutrition education information. Which recommendation should be shared by this APRN? Increase the use of protein supplements and ingest 3 g/kg carbohydrate (CHO) daily Ingest at least 1,200 calories daily and increase calcium intake Ingest high-fat snacks prior to sports activity Increase the use of sports drinks before activity

B

The National Academy of Medicine has determined, after a comprehensive review, determined there is no substantiated evidence of a causal relationship between thimerosal-containing vaccines or measles, mumps, rubella (MMR) vaccine and what disorders? (Select all that apply.) 1/1 childhood schizophrenia autism Asperger syndrome Attention Deficit hyperactivity disorder (ADHD) Petts syndrome

B

An advanced practice registered nurse (APRN) is seeing a 15-year-old patient for a yearly checkup. Two years ago, the patient was diagnosed with a brain tumor and was treated with chemotherapy and radiation. The patient is currently experiencing weight loss, weakness, and vomiting. The APRN reviews the vital signs and notices that the patient has been consistently hypertensive. Which step should the APRN take next to treat this patient? Request a consultation with the allergy and immunology clinic Refer the patient to a pediatric endocrinologist Prescribe antiemetics and have the patient return to the clinic in one month Keep a food diary for three months to evaluate for food allergies or intolerance

B

An advanced practice registered nurse (APRN) needs to assess the functioning of cranial nerve II (CN II) in an eight-year-old after a facial injury. Which procedure should the APRN follow to obtain this information? Shine a penlight into the child's eyes to check pupil reaction Ask the child to read an age-appropriate short story out loud Apply a cotton wisp strand to the child's cornea for reaction Have the child squint his eyes and scrunch his face

B

An advanced practice registered nurse examines an adolescent and makes an assessment of the patient's values. How should a patient in this developmental stage respond if he's healthy in this area? Through attendance at worship services on a regular basis With respect for people and property With fear of punishment when responding to a parent With the belief that rules are absolute

B

Systemic lupus erythematosus (SLE) is typically diagnosed many years after the onset. Which describes why this is often the case? 1/1 Most children are immunized against SLE SLE has varied, nonspecific manifestations Few health care providers are aware of SLE Few patients with SLE seek medical attention

B

The Family nurse practitioner is examining a 3-year-old child who speaks loudly, in a monotone, does not make eye contact, and prefers to sit on the exam room floor moving a toy truck back and forth in a repetitive manner. Which disorder does the nurse practitioner suspect? 1/1 Attention-deficit/hyperactivity disorder Autism spectrum disorder Executive function disorder Sensory processing disorder

B

The Family nurse practitioner is performing a well child assessment on a 13-year-old female whose mother asks when her daughter's periods may start. Which information will the nurse practitioner use to help estimate the onset of periods? 1/1 The age of the mother's menarche The patient's age at thelarche When adrenarche occurred Whether linear growth has stopped

B

The Family nurse practitioner is performing a well child exam on an 8-year-old girl and notes the presence of breast buds. What will the nurse practitioner include when initiating anticipatory guidance for this patient? 1/1 A discussion about the risks of pregnancy and sexually transmitted diseases Information about sexual maturity and menstrual periods Material about the human papillomavirus vaccine Sexual orientation and the nature of sexual relationships

B

The Family nurse practitioner is performing a well child examination on a 8-month-old infant whose hearing is normal but who responds to verbal cues with only single syllable vocalizations. What will the nurse practitioner recommend to the parents to improve speech and language skills in this infant? 1/1 Provide educational videos that focus on language. Read simple board books to the infant at bedtime. Sing to the child and play lullabies in the baby's room. Turn the television to Sesame Street during the day.

B

The Family nurse practitioner is performing a well child examination on a school-age child who has a history of cancer treated with cranial irradiation. What will the nurse practitioner monitor in this child? 0/1 Cardiomyopathy and arrhythmias Leukoencephalopathy Obesity and gonadal dysfunction Peripheral neuropathy and hearing loss

B

The parent of a 5-month-old is worried because the infant becomes fussy but doesn't always seem interested in nursing. What will the nurse practitioner tell this parent? 1/1 The parent should give ibuprofen for teething pain before nursing. The infant may be expressing a desire to play or to rest This is an indication that the infant is ready for solid foods. This may indicate gastrointestinal discomfort such as constipation.

B

The parent of a child who has asthma asks the Family pediatric nurse practitioner about whether the child may engage in strenuous exercise. What will the nurse practitioner tell the parent? 1/1 Children with asthma should be excluded from vigorous exercise and most strenuous sports. Children with asthma show improved aerobic and anaerobic fitness with moderate to vigorous/physical activity. Physical activity has been shown to improve overall pulmonary function in children with asthma. Vigorous exercise helps improve symptoms in children with poorly controlled asthma.

B

The parents of an 18-month-old child bring the child to the clinic after observing a brief seizure of less than 2 minutes in their child. In the clinic, the child has a temperature of 103.1°F, and the Family nurse practitioner notes a left otitis media. The child is alert and responding normally. What will the nurse practitioner do? 1/1 Order a lumbar puncture, complete blood count, and urinalysis. Prescribe an antibiotic for the ear infection and reassure the parents. Refer to a pediatric neurologist for anticonvulsant and antipyretic prophylaxis. Send the child to the emergency department for EEG and possible MRI.

B

The single mother of a 4-year-old who attends day care tells the Family nurse practitioner that she had difficulty giving her child a twice-daily amoxicillin for 10 days to treat otitis media during a previous episode several months earlier because she works two jobs and is too busy. The child has an ear infection in the clinic today. What will the nurse practitioner do? 0/1 Administer an intramuscular antibiotic. Order twice-daily amoxicillin for 5 days. Prescribe azithromycin once daily for 5 days. Reinforce the need to adhere to the plan of care.

B

When performing a neurologic exam to assess for meningeal signs in an infant, the Family nurse practitioner will attempt to elicit the Kernig sign by implementing what action? 1/1 bending the infant at the waist to touch fingers to toes. extending the leg at the knee with the infant supine. flexing the infant's neck to touch chin to chest. turning the infant's head from side to side.

B

n advanced practice registered nurse (APRN) is assessing an 11-year-old for a recent fall on a playground that resulted in a minor open wound to the leg. Which vaccination should this APRN confirm is current? Rabies immune globulin (RIG) Tetanus, diphtheria, and acellular pertussis (Tdap) Hepatitis A (Hep A) Hepatitis B virus (HBV)

B

which of the following information is most important to ask while performing a sports physical? 1/1 does the child have shortness of breath with exercise? has the child ever fainted during exercise? has the child ever art undergone surgery? has the child ever had a head injury?

B

A typically developing 6-month-old would be expected to: 1/1 Self-feed with a spoon sit without support laugh and play a clapping game say two words Feedback

B A developing infant should be able to sit unsupported or with very little assistance by age 6 months. By 17 months, the toddler should be able to self-feed with a spoon. A 10-month-old should be able to play patty-cake or a clapping game. A 15-month-old should be able to say more than two words.

An advanced practice registered nurse (APRN) is assessing a nine-year-old female patient who has a history of slow growth. The patient's current height is at 50% and weight is at 75% for the age. The patient has also been having periods of hypoglycemia. There is no family history of growth-related conditions. Which action should the APRN take for this patient? Schedule a DEXA scan Perform a karyotype Prescribe growth hormones Refer to gastroenterologist

B A karyotype test looks at the size, shape, and number of your chromosomes. Chromosomes are the parts of your cells that contain your genes. Genes are parts of DNA passed down from your mother and father. They carry information that determines your unique traits, such as height and eye color.Jul 7, 2021

The parents of a 3-year-old child are concerned that the child has begun refusing usual foods and wants to eat mashed potatoes and chicken strips at every meal and snack. The child's rate of weight has slowed, but the child remains at the same percentile for weight on a growth chart. What will the Family nurse practitioner tell the parents to do? 1/1

B Allow the child to choose foods for meals to improve caloric intake. Place a variety of nutritious foods on the child's plate at each meal Prepare mashed potatoes and chicken strips for the child at mealtimes. Suggest cutting out snacks to improve the child's appetite at mealtimes.

a 14 year old female patient has non pustular, closed and open comedones. This patient has administered over the counter medications without effect. The nurse practitioner next action is to: 1/1 prescribe oral minocycline prescribed topical tretinoin recommend an antibacterial facial soap refer the patient to a dermatologist

B Feedback because over the counter treatment of mild to moderate acne was ineffective, topical tretinoin is recommended as the next line of treatment. Patients with mild-to-moderate comedone and/or inflammatory acne usually respond well to a combination of topical retinoids, benzoyl peroxide, and antibiotics. Moderate-to-severe papulopustular acne warrants the use of oral antibiotics in combination with topical agents

the sexual development of a 14 year old girl is notably delayed. She is very short, slightly obese, and complains of constipation and fatigue. Her skin is sallow and coarse in texture, and her hair is brittle. Which of the following tests should be obtained? 1/1 chromosome analysis and serum thyroxine serum thyroxine and thyroid stimulating hormone serum luteinizing hormone and chromosome analysis serum luteinizing hormone and thyroid stimulating hormone

B Feedback classic symptoms of growth failure, goiter, delayed or arrested puberty, delayed dentition, weight gain, fatigue, dry skin, hyperlipidemia, decline in school performance, minerals, and fatigue would indicate the need to test for hypothyroidism first in this patient. If these tests are normal than you would want to consider other test for reasons for her growth delay

dermatitis. What would the physical exam reveal? 0/1 dry, scaly rash with pruritus distribution of rash on face and extensor surfaces erythematous raised areas on flexor surfaces moist, crusting rash with no pruritis

B Feedback infantile atopic dermatitis, as contrasted to atopic dermatitis in older children and adolescents, is a moist, oozing, crusting rash found mainly on the extensor surfaces of the body and the face. It usually begins about 2 months of age, often with a family history of a atopy.

a new mother is concerned about the Hemangioma on her infant's neck. What is the treatment of choice for the majority of infants with hemangioma? 1/1 cryosurgery intralesional injection of steroids observation injection of a sclerosing agent

B Feedback the most common treatment is observation because most hemangiomas resolved over time, usually beginning about 18 months of age. The other treatments maybe perform, especially for hemangiomas proliferating rapidly. Treatment with Propranolol, topical, or laser therapy may be used. Propranolol is generally initiated inpatient, with nursing supervision of blood pressure and heart rate and blood glucose.

the family nurse practitioner obtains a sexual history from a 15 year old female patient. The Adolescent reports that her boyfriend has recently been pressuring her to agree to engage in sexual intercourse. The FNP's next action is to: 1/1 explain the risks of having unprotected sexual intercourse explore The Adolescents feelings about her desire to consent to sexual activities offer the Adolescent oral or transdermal birth control medications perform a gynecological examination and obtain a baseline pap smear

B Feedback the next course of action is to encourage the Adolescent to express personal feelings about her desire to consent to sexual activities. Understanding the patient's feelings about the issue will serve to Taylor the education and resources provided. Explaining the risks of having unprotected sex and offering birth control will not address the fact that the patient is feeling pressured to have sex. A gynecologic examination and first pap smear is not indicated until the age of 21 or within three years of having sex.

the family nurse practitioner is assessing an infant that was delivered by cesarean section. What is a complication frequently associated with cesarean delivery? 1/1 increased levels of serum bilirubin respiratory distress meconium aspiration hypoglycemia

B Feedback the physiology of normal vaginal delivery causes pressure against the infant's thorax and helps remove amniotic fluid. If the cesarean section was done is an emergency Before term, the infant may have inadequate surfactant for good pulmonary function. Increased levels of bilirubin, meconium aspiration, and hypoglycemia are not associated with an increase in cesarean delivery.

a teenager comes to the office of the family nurse practitioner in states that she was raped several hours ago by her boyfriend. What immediate action should be taken by the FNP? 1/1 perform a pelvic exam to determine injuries to the patient accompany her to the emergency department for an exam send her immediately for counseling to help her deal with the situation call the patient's parents so they can be with her

B Feedback this patient should be examined by emergency department personnel, many of whom are specially trained to collect the evidence needed to testify in court about right. The exam should not be done in the office unless the FNP has been trained in evidence collection and has a rape evidence collection kit. The exam must be done quickly, before evidence is destroyed. The patient decide who should be called for support. Although she's encouraged to call her parents, she is also offered the support of Rape Crisis and other resources

which of the following children requires a language referral for further evaluation? 1/1 1 year old with a 5 word vocabulary 2 year old who cannot put two words together 2 year old with a 30 word vocabulary 3 year old who is understandable to parents

B a one-year-old can have five words with normal development, a two-year-old should have 20 plus words, and by the age of three the child should be fully understandable by parents and 90% intelligible by strangers. A two year old should be able to put two words sentences together so if this Milestone is not present than the patient needs further evaluation to assess for language delays

in reacting to a parental divorce, a child between the ages of nine and 12 differs from a younger child in his or her: 1/1 acceptance of the family's dissolusion conscious, intense anger positive reaction to the departure of a parent tendency toward emotional decompensation Feedback

B divorce is stressful and depending on the age of the child then you can expect different reactions. Children 2 to 5 years of age will often have regression, sleep disturbances, irritability and aggression. Children 6 to 8 years of age will often express open grieving and feelings of being rejected. They may also experience fussiness, regression ,sadness, fear and exhibit immature behaviors. Children between 9 and 12 years of age show fear and intense anger at one or both parents. The child who is 13 years of age or more is worried about their own future, may act out or have depression symptoms.

a 16 year old adolescent boy is 54 in in height. The family nurse practitioner identifies the following as a positive, effective coping behavior: 1/1 acts as the Class Clown has a rehearsed reply to teasing comments spends most of his free time watching television has predominantly friends that have short stature Feedback

B role playing and planning a rehearsed reply to teasing comments about short stature or helpful tools to deal with this issue. Although humor can be effective, constantly clowning around for attention is not positive coping Behavior. Withdrawal such as watching television or reading is not effective coping and may indicate depression. Spending time and associating only with younger adolescents who are his height is not a positive coping behavior and may hinder normal maturation

while evaluating a three-day-old infant, the nurse practitioner turns the infant on the side to examine the back and notes a sharp line of demarcation dividing the upper and lower parts of the body. The dependent half is bright red and the superior half is pale. The etiology of this phenomenon is: 1/1 an elevated Central hematocrit level, typically greater than 65% and immature autonomic regulatory system, generally referred to as the "Harlequin" sign decreased blood flow to the superior half, usually caused by coarctation of the aorta vasoconstriction of peripheral blood vessels, usually caused by sepsis Feedback

B the classic appearance of a harlequin sign is the division of the body skin color from forehead to pubis into red and pale have. Etiology is unknown but thought to be immature autonomic response. It is a benign finding and his self resolving. The high hematocrit would result in a red overall appearance. Coarctation would be associated with poor pulses and blood pressure differences as well as pale mottled appearance overall (coarctation usually occurs between thetake off of the right subclavian and the remainder of the body. Sepsis results in a mottled appearance with poor capillary refill and cool extremities.

a 5 year old's physical examination is normal and the child has no known allergies to food or medications. To date, the child has not been immunized and has had varicella disease. The nurse practitioner now administers a polio virus vaccine, a diphtheria, tetanus, and pertussis vaccine; measles mumps and rubella virus vaccine and: 1/1 haemophilus influenza type B vaccine hepatitis B vaccine meningococcal vaccine varicella vaccine (varivax)

B' Feedback since the child has not been previously immunized, Hepatitis B should be given. The Haemophilus influenza type B vaccine should not be given to Children over five years of age if they are otherwise healthy. The meningococcal vaccine is not indicated and healthy children until 11 years of age. Because the child has had the varicella disease, the varicella vaccine is not a priority at this visit

An advanced practice registered nurse (APRN) is seeing a child for complaints of fatigue, frequent epistaxis, and right upper abdominal pain. During the exam, the APRN notes increased WBCs, hepatosplenomegaly, and a 10% weight loss since a visit one month earlier. Which diagnosis should the APRN make for this patient? Non-Hodgkin lymphoma Leukemia Neuroblastoma Hemophilia

B-The main difference is that leukemia affects the blood and bone marrow, while lymphomas mainly affect the lymph nodes.Unlike lymphoma, leukemia won't produce any sort of visible lump or bump (a tumor) that you usually associate with cancer.

A 12-month-old child is being seen by an advanced practice registered nurse (APRN) for a routine checkup and for the hepatitis b (HepB); Haemophilus influenzae type B (Hib); inactivated poliovirus (IPV); measles, mumps, and rubella (MMR); and varicella (VAR) immunizations. The child's parent refuses the MMR vaccine because the child received it at six months of age after several cases of measles were diagnosed at the child's day care center. How should the APRN respond to this parent's statement? "Per recommendations, the first dose of MMR is given when the child is four years of age." "The MMR vaccination is recommended to be given in an oral form to prevent localized swelling and pain from an intramuscular injection." "The measles vaccine is to be given at the routine recommended ages even if it was administered because of an exposure." "The measles vaccine given at six months can be counted as the first dose of MMR."

C

A 12-year-old child whose weight and body mass index (BMI) are in the 75th percentile has a diastolic blood pressure that is between the 95th and 99th percentiles for age, sex, and height on three separate occasions. Which test will be prescribed for this child initially? 1/1 complete blood count erythrocyte sedimentation rate renal function urinalysis and electrolytes

C

A 13-year-old Native American female has a body mass index (BMI) at the 90th percentile for age. The Family nurse practitioner notes the presence of a hyperpigmented velvet-like rash in skin folds. The child denies polydipsia, polyphagia, and polyuria. The nurse practitioner will take what action? 1/1 Counsel the child to lose weight to prevent type 2 diabetes. Diagnose type 2 diabetes if the child has a random glucose of 180 mg/dL. Order a fasting blood sample for a metabolic screen for type 2 diabetes. Refer the child to a pediatric endocrinologist.

C

A 14-year-old child has a headache, unilateral weakness, and blurred vision preceded by fever and nausea. The child's parent reports a similar episode several months prior. The Family nurse practitioner will consult with a pediatric neurologist to order what diagnostic test? 1/1 a lumbar puncture. an electroencephalogram (EEG). neuroimaging with magnetic resonance imaging (MRI). positron emission tomography (PET) scan.

C

A 17-year-old female is being seen in the clinic for a wellness visit. The patient tells the advanced practice registered nurse (APRN) that she is currently sexually active, has a history of three different partners, does not routinely use condoms, had a normal menstrual cycle the week before, and denies any complaints. Which testing should the APRN provide for this patient? Urinalysis Pelvic exam STI test Pregnancy test

C

A 32-year-old female presents to the clinic as a new patient to establish care and complains of vague symptoms of abdominal fullness and bloating. On the rectovaginal exam, the advanced practice registered nurse (APRN) notes a solid, irregular, and fixed pelvic mass. Which cancer diagnosis should the APRN make for this patient? Cervical Vulvar Ovarian Anal

C

A 4-year-old child diagnosed with an upper respiratory tract infection has cloudy nasal discharge and moderate nasal congestion interfering with sleep. The parent asks what product to use to help with symptoms. What will the Family nurse practitioner recommend? 1/1 Antihistamines Decongestant sprays Saline rinses Zinc supplements

C

A 6-year-old child has a systolic blood pressure between the 95th and 99th percentile for age, sex, and height and a diastolic blood pressure between the 90th and the 95th percentile on three separate clinic visits. This child's blood pressure is placed in which classification? 0/1 normotensive pre-hypertensive stage 1 hypertensive stage 2 hypertensive

C

A 6-year-old child has a systolic blood pressure between the 95th and 99th percentile for age, sex, and height and a diastolic blood pressure between the 90th and the 95th percentile on three separate clinic visits. This child's blood pressure is placed in which classification? 1/1 normotensive pre-hypertensive stage 1 hypertensive stage 2 hypertensive

C

A 66-year-old male patient comes to the clinic for the first time accompanied by a person who is introduced as his male life partner. The advanced practice registered nurse (APRN) wants to use inclusive language while obtaining the patient's history to help promote a welcoming healthcare environment. Which term should the APRN use during the interview to accomplish this goal? Friend Spouse Significant other Roommate

C

A child has small, firm, flesh-colored papules in both axillae which are mildly pruritic. What is an acceptable initial approach to managing this condition? 1/1 Application of trichloroacetic acid 25% to 50% using a dropper Applying liquid nitrogen for 2 to 3 seconds to each lesion Reassuring the parents that these are benign and may disappear spontaneously Referral to a dermatologist for manual removal of lesions with curettage

C

A child is brought to the clinic with a generalized, annular rash characterized by raised wheals with pale centers. On physical examination, the child's lungs are clear and there is no peripheral edema. A history reveals ingestion of strawberries earlier in the day. What is the initial treatment? 1/1 Aqueous epinephrine 1:1000 subcutaneously Cetirizine once in clinic and then once daily for 2 weeks Diphenhydramine 0.5 to 1 mg/kg/dose every 4 to 6 hours Prednisone 1 to 2 mg/kg/day for 1 week with rapid taper

C

A child who was treated with amoxicillin and then amoxicillin-clavulanate for acute otitis media (OTM) is seen for follow-up. The Family nurse practitioner notes dull-gray tympanic membranes with a visible air-fluid level. The child is afebrile and without pain. What is the next course of action? 0/1 Administering ceftriaxone IM Giving clindamycin orally Monitoring ear fluid levels for 3 months Watchful waiting for 48 to 72 hours

C

A child with Down syndrome who has sleep-disordered breathing with obstructive sleep apnea continues to have symptoms in spite of tonsillectomy and adenoidectomy and treatment with a leukotriene receptor antagonist medication and a nasal steroid spray. The Family nurse practitioner will refer the child to a sleep disorder clinic to discuss which therapy? 1/1 craniofacial surgery oral appliances Positive airway pressure therapy Supplemental oxygen

C

A patient presents to an advanced practice registered nurse (APRN) for lower-back pain that has been returning intermittently for the past year. Corticosteroids have been tried and optimized with no relief. The APRN notes that previous lumbar cervical spine X-rays and an MRI suggest radiculopathy arthritis. Which action should the APRN take to assist this patient? Prescribe a long-acting opioid using the maximum dose Schedule for outpatient mental health counseling Prescribe a short-acting opioid using the lowest dose Schedule reassessment within six to eight weeks

C

A patient receiving antenatal care for a first pregnancy returns for another visit at 20-weeks gestation. The advanced practice registered nurse (APRN) notes that all diagnostic tests indicate the patient is experiencing intrauterine growth restriction and makes recommendations to improve the growth of the fetus. Which recommendation should the APRN make to this patient? Take aspirin 325 mg QD Take heparin 10000 U bid Stop smoking Increase physical activity

C

A patient who is transgender presents to the primary care clinic with a laceration to the forearm. While suturing the laceration, the advanced practice registered nurse (APRN) discusses the most critical topic to address at that moment with the patient. Which topic should the APRN discuss with this patient? The patient's body image perception. The patient's history of drug usage. The patient's reason for the visit. The patient's level of depression.

C

A patient with chronic kidney disease is scheduled for a permanent A/V fistula placement. The patient is experiencing intra-dialysis hypotension. During the physical exam, the advanced practice registered nurse (APRN) notes that the patient has a systolic ejection murmur. Which order should the APRN make prior to this patient's surgery? Stress test Fistulogram Echocardiogram Potassium level

C

A school-age child has an abrupt onset of sore throat, nausea, headache, and a temperature of 102.3°F. An examination reveals petechiae on the soft palate, beefy-red tonsils with yellow exudate, and a scarlet rash. A Rapid Antigen Detection Test (RADT) is negative. What is the next step in management for this child? 1/1 Consider a sexual abuse diagnosis. Obtain an anti-streptococcal antibody titer. Perform a follow-up throat culture Prescribe amoxicillin for 10 days.

C

A school-age child is brought to clinic after a pediculosis capitis infestation is reported at the child's school. If this child is positive, what will the Family Nurse practitioner expect to find on physical examination, along with live lice near the scalp? 1/1 Excoriated macules along the child's collar and underwear lines Inflammation and pustules on the face and neck Itching of the scalp, with skin excoriation on the back of the head Linear or S-shaped lesions in webs of fingers and sides of hands

C

An 18-month-old child has a 1-day history of intermittent, cramping abdominal pain with non-bilious vomiting. The child is observed to scream and draw up his legs during pain episodes and becomes lethargic in between. The Family nurse practitioner notes a small amount of bloody, mucous stool in the diaper. What is the most likely diagnosis? 1/1 Appendicitis Gastroenteritis Intussusception testicular torsion

C

An adolescent female reports unilateral headache pain associated with abdominal pain and nausea occurring just prior to periods each month. The adolescent has been using naproxen sodium for 6 months but reports little relief from symptoms. What will the Family nurse practitioner do? 0/1 Add acetaminophen and ondansetron to the naproxen regimen. Consider prophylactic therapy with a beta-blocker or anticonvulsant drug. Prescribe sumatriptan nasal spray at the onset of headache and again 2 hours if needed Refer the adolescent to a pediatric neurologist for neuroimaging studies.

C

An adolescent is diagnosed with functional abdominal pain (FAP). The child's symptoms worsen during stressful events, especially with school anxiety. What will be an important part of treatment for this child? 1/1 Informing the parents that the pain is most likely not real Instituting a lactose-free diet along with lactobacillus supplements Teaching about the brain-gut interaction causing symptoms Using histamine2-blockers to help alleviate symptoms

C

An advanced practice registered nurse (APRN) assesses a six-year-old child with dyslexia during a well-child check. The parent is concerned because the child is not progressing as quickly as other students at school. Which form of assistance should be provided by this APRN? A consultation with a neurologist Information on special educational programs Information about dyslexia

C

An advanced practice registered nurse (APRN) is assessing a child who woke up with swelling around both eyes. The child's parent reports that the child's shoes would not fit that morning, and the child has not been urinating at the typical frequency. Which intervention should be taken first by this APRN? Prescribe a seven-day course of steroids Place the child on bed rest Consult with a nephrologist Refer to a cardiologist

C

An advanced practice registered nurse (APRN) is assessing an adolescent who has a BMI below the fifth percentile. The adolescent admits to preferring oversize clothing to avoid body comparisons with more attractive peers but denies any problems are present. The parent states that the adolescent is no longer eating like before and often skips dinner for no reason. Which intervention should be taken by this APRN? Recommend specific nutritional guidelines Recommend joining a sports team Refer the patient to a mental health specialist Refer the patient to a nutritionist

C

An advanced practice registered nurse (APRN) is completing the preparticipation examination for a 15-year-old high school student who would like to join the basketball team. The assessment reveals a medical condition that will require the APRN to advise the student against participation in the sport. Which medical condition does this student have? Heart murmur Mitral valve prolapse Acute rheumatic fever with carditis Ehlers-Danlos syndrome, vascular form

C

An advanced practice registered nurse (APRN) is examining a newborn infant and notices a lower oxygen saturation than expected as well as a slightly enlarged liver. Which action should the APRN take to assist this patient? Recheck at two months of age Consult with a respiratory therapist Refer to a pediatric cardiologist Recommend discontinuation of breastfeeding

C

An advanced practice registered nurse (APRN) is examining a newborn infant when the parent mentions that the child sleeps best on the stomach. Which evidence-based recommendation should the APRN make to this parent? This is the best sleep position for a child who is birth to three months old. It is best to alternate between right and left side positions. It is best to place the child on the back to sleep. This is the best sleep position if the head of the bed is elevated three inches.

C

An advanced practice registered nurse (APRN) is providing care to an 11-month-old patient who is hospitalized. The APRN has treated the patient's pain with medications, but the infant is still exhibiting a high pain score. The APRN decides to use a nonpharmacological pain management measure that is appropriate for the patient's developmental stage. Which nonpharmacological measure should be used by this APRN? Taking the patient to look out the window Offering the patient a snack Introducing nonnutritive sucking to the patient Giving the patient stickers

C

An advanced practice registered nurse (APRN) is seeing a 15-month-old child to check for hepatitis A due to an outbreak of the disease in the child's day care center. The child tests negative for the infection, and the APRN educates the child's parents on household prevention. Which recommendation should the APRN provide to these parents? Administer hepatitis B virus (HBV) vaccine Sanitize hands prior to handling food Wash hands after diaper changes Administer hepatitis B immune globulin (HBIG)

C

An advanced practice registered nurse (APRN) is seeing patients in an area where the majority of the families are recent immigrants, and a high number of children are developmentally delayed. Which question should this APRN include in each well-child visit? "Are there exterior paint surfaces on your home that need repair?" You Selected "Has your child lived in a house built in the 1980s?" "Do you cook with ceramic pottery from your homeland?" "Has your child been diagnosed with asthma?"

C

An older adult has a history of taking multiple medications for gastroesophageal reflux disease. The advanced practice registered nurse (APRN) reviews the medications and decides to discontinue the patient's proton pump inhibitor. Which effect of this medication prompted the APRN to remove this prescription? Increased risk for hypertension Increased rate of hypermagnesemia Increased rate of bone fracture Increased risk for Helicobacter pylori

C

During a routine well child exam on a 5-year-old child, the Family nurse practitioner auscultates a grade II/VI, harsh, late systolic ejection murmur at the upper left sternal border that transmits to both lung fields. The child has normal growth and development. What will the nurse practitioner suspect? 0/1 Aortic stenosis (AS) Patent ductus arteriosus (PDA) Pulmonic stenosis Tricuspid atresia

C

During a well child exam on a 6-year-old child, the Family nurse practitioner assesses the child for school readiness. Which finding may be a factor in limiting school readiness for this child? 1/1 Adherence to daily family routines and regular activities Having two older siblings who attend the same school Parental concerns about bullying in the school The child's ability to recognize four different colors

C

During a well child exam, the Family nurse practitioner learns that the parents of a young child fight frequently about finances. The parents state that they do not fight in front of the child and feel that the situation is temporary and related to the father's job layoff. What will the nurse practitioner do? 1/1 Reassure them that the child is too young to understand. Recommend that they continue to not argue in front of the child. Suggest counseling to learn ways to handle stress. Tell them that the conflict will resolve when the situation changes.

C

During a well child examination of a 6-year-old girl, the Family nurse practitioner notes that the child becomes embarrassed and resists taking off her underwear for the exam. What should the nurse practitioner infer from this observation? 1/1 The child has been sexually molested. The child is feeling violated by the examiner. The parent is exhibiting regressive behavior. This is a normal reaction in a child of this age.

C

The Family Nurse practitioner learns that the mother of a 3-year-old child has been treated for depression for over 5 years. Which aspect of this child's development will be of the most concern to the nurse practitioner? 1/1 fine motor gross motor social/emotional speech and language

C

The Family nurse Practitioner is caring for a 3-year-old girl with prolonged swelling and stiffness in her arm. The nurse should know that these symptoms are indicative of which type of juvenile idiopathic arthritis (JIA)? 1/1 Systemic JIA polyarticular JIA oligoarticular JIA Juvenile Rheumatoid arthritis

C

The Family nurse practitioner auscultates a new grade II vibratory, mid-systolic murmur at the mid sternal border in a 4-year-old child that is louder when the child is supine. What type of murmur is most likely? 0/1 Pathologic murmur Pulmonary flow murmur Still's murmur Venous hum

C

The Family nurse practitioner is conducting a follow-up examination on a child who has recently begun taking a low-dose stimulant medication to treat attention-deficit/hyperactivity disorder (ADHD). The child's school performance and home behaviors have improved. The child's parent reports noticing a few tics, such a twitching of the eyelids, but the child is unaware of them and isn't bothered by them. What will the nurse practitioner recommend? 1/1 Adding an alpha-agonist medication Changing to a non-stimulant medication Continuing the medication as prescribed Stopping the medication immediately

C

The Family nurse practitioner is examining a newborn infant recently discharged from the neonatal intensive care unit after a premature birth. The parent is upset and expresses worry about whether the infant will be normal. What will the nurse practitioner do in this situation? 1/1 Explain to the parent that developmental delays often do not manifest at first. Perform a developmental assessment and tell the parent which delays are evident. Point out the tasks that the infant can perform while conducting the assessment Refer the infant to a developmental specialist for a complete evaluation.

C

The Family nurse practitioner is offering anticipatory guidance to the parents of a 6-year-old child who has Down syndrome. What will the nurse practitioner tell the parents about physical activity and sports in school? 1/1 Children with Down syndrome get frustrated easily when engaging in sport Children with Down syndrome should not participate in strenuous aerobic activity Their child should have a cervical spine evaluation before participation in sports. Their child should only participate in sports sanctioned by the Special Olympics.

C

The Family nurse practitioner is performing a well baby examination on a 2-month-old infant who has gained 25 grams per day in the last interval. The mother is nursing and tells the nurse practitioner that her infant seems fussy and wants to nurse more often. What will the nurse practitioner tell her? 1/1 She may not be making as much breastmilk as before. She should keep a log of the frequency and duration of each feeding. The infant may be going through an expected growth spurt. The infant should stay on the previously established nursing schedule.

C

The Family nurse practitioner notes a musty odor when examining a newborn at a 2-week checkup. What will the nurse practitioner suspect? 0/1 Galactosemia Glucose-6-phosphatase deficiency Phenylketonuria Urea cycle disorder

C

The Family nurse practitioner performs a well baby examination on a 7-day-old infant who is nursing well, according to the mother. The nurse practitioner notes that the infant weighed 3250 grams at birth and 2990 grams when discharged on the second day of life. The infant weighs 3080 grams at this visit. Which action is correct? 1/1 Follow up at the 2-month checkup. Refer to a lactation consultant. Schedule a weight check in 1 week Suggest supplementing with formula.

C

The parent of a 10-year-old boy tells the Family nurse practitioner that the child doesn't appear to have any interest in girls and spends most of his time with a couple of other boys. The parent is worried about the child's sexual identity. What will the nurse practitioner tell the parent? 1/1 Children at this age who prefer interactions with same-gender peers usually have a homosexual orientation. Children experiment with sexuality at this age as a means of deciding later sexual orientation. This attachment to other same-gender children is how the child learns to interact with others It's appropriate to encourage mixed-gender interactions in order to promote development of sexual values.

C

The parent of a 14-year-old child tells the Family nurse practitioner that the adolescent has expressed a desire to be a vegetarian, is refusing all meat served at home, and wants the family to eat vegetarian meals. What will the nurse practitioner tell the parent? 1/1 Do not allow a vegetarian diet in order to maintain appropriate limits for the adolescent. Provide vegetarian options for the adolescent that preserve adequate nutrition and protein intake. Suggest that the adolescent prepare appropriate vegetarian dishes to complement family meals Tell the adolescent that a vegetarian diet may be considered in adulthood but not while living at home.

C

The parent of a 14-year-old child tells the Family nurse practitioner that the child skips classes frequently in spite of various disciplinary measures, such as grounding and extra homework and is earning Cs and Ds in most classes. What will the nurse practitioner recommend? 1/1 Counseling for emotional problems Development of an Individual Education Plan Evaluation for possible learning disorders Referral for a behavioral disorder

C

The parent of a 24-month-old child asks the Family nurse practitioner when toilet training should begin. How will the pediatric nurse practitioner respond? 1/1 "Begin by reading to your child about toileting." "Most children are capable by age 2 years." "Tell me about your child's daily habits." "We should assess your child's motor skills."

C

The parent of a newborn infant asks the Family nurse practitioner when to intervene to help the infant's future intellectual growth. What will the nurse practitioner tell the parent? 1/1 Cognitive learning begins during the toddler years. Intellectual growth begin when speech develops. Language and literacy skills begin at birth. Preschool is an optimal time to begin general learning.

C

The parent of a toddler and a 4-week-old infant tells the Family nurse practitioner that the toddler has just been diagnosed with pertussis. What will the nurse practitioner do to prevent disease transmission to the infant? 0/1 Administer the initial diphtheria, pertussis, and tetanus vaccine. Instruct the parent to limit contact between the toddler and the infant. Order azithromycin 10 mg/kg/day in a single dose daily for 5 days Prescribe erythromycin 10 mg/kg/dose four times daily for 14 days.

C

What is an initial key part of management of a child suspected of having an inborn error of metabolism(IEM)? 0/1 consulting a metabolic specialist. obtaining a complete family history. ordering metabolic screening tests. referring the family to a dietician.

C

When meeting with a new family, the Family nurse practitioner develops a database that identifies family members and others living in the household, relationships with others outside the household, and significant behavioral and emotional problems. Which tool will the nurse practitioner use to record this information? 1/1 CRAFFT Ecomap genogram Pedigree

C

the Family nurse practitioner evaluates a school-age client whose body mass index (BMI) is greater than the 97th percentile. The FNP is concerned in order several laboratory tests to evaluate this. Which diagnosis will the FNP document for this visit? 1/1 precocious puberty Prader Willis syndrome obesity Cushing syndrome

C

the parent of a three-year-old is concerned that the child may have autism. The provider complete a modified checklist for autism in toddlers (M-CHAT) tool and the results indicate several areas of concern. What will the family nurse practitioner do? 0/1 administer a childhood autism rating scale (CARS) in the clinic consult a specialist to determine early intervention strategies refer the child to a behavioral specialist for further evaluation inform the parent that results from the M-CHAT tool indicates that the child has autism

C

which is an accurate statement about vegetarian diets during childhood? 0/1 a child who eats lacto vegetarian diet is unlikely to consume adequate amounts of protein and vitamins for proper growth and development algae is a good source of bioavailable vitamin B12 for a child who eats a vegetarian diet bioavailable vitamin B12 is present in both animal and plant based foods vitamin B12 and zinc deficiencies place a child at risk for developmental delays

C B12 and Zinc are most commonly found in meat and egg products. Sources of these nutrients in a vegan diet would include fortified soy milk, fortified soy-based meat substitutes, nutritional yeast, fortified cereals, almonds, brown rice, wheat germ, tofu, pecans, and spinach. Other deficiencies with vegan or variations of vegetarian diets can include vitamin D, riboflavin, calcium, and iron

the family nurse practitioner is comparing the typical signs of depression in the Adolescent with the adult patient. The depressed that elicits would present with: 1/1 lonely feelings sad, flat affect anger and acting-out behavior feelings of powerlessness and anxiety

C Feedback adolescents often act out to protect themselves from feelings of vulnerability and dependency. It is important to evaluate signs of anger and frustration in the Adolescent because the significance of the behavior May indicate symptoms of depression. Adults who are depressed typically display findings noted in the other three options

which of the following findings with the family nurse practitioner expect to find in a child with pubertal gynecomastia? 1/1 Tanner stage two with testes less than or equal to 4 cm in length breast and nipples non-tender and equal in size breast tissue enlargement mainly glandular, movable, and non-adherent to skin or underlying tissue lymphadenopathy, goiter, asymmetric testes, and repaired hypospadias Feedback pubertal or physiologic gynecomastia is a visible or palpable glandular enlargement of the male breast that can occur in healthy adolescents. Typically, the breasts are unequal in size and may be Tender, nipples are often irritated from rubbing against clothing, and Tanner stage is 2 through 4 of pubertal development are noted. The symptoms of Lymphedenopathy, goiter, asymmetric testes, and repaired hypospadias are associated with pathologic gynecomastia.

C Feedback pubertal or physiologic gynecomastia is a visible or palpable glandular enlargement of the male breast that can occur in healthy adolescents. Typically, the breasts are unequal in size and may be Tender, nipples are often irritated from rubbing against clothing, and Tanner stage is 2 through 4 of pubertal development are noted. The symptoms of Lymphedenopathy, goiter, asymmetric testes, and repaired hypospadias are associated with pathologic gynecomastia.

a teenage female patient is brought to the family nurse practitioner for evaluation by her grandmother with whom she lives. The teenager has been vomiting and her grandmother believes that she is becoming confused. The grandmother relates that the patient has been upset lately over a breakup with her boyfriend. What will the family nurse practitioner investigate as a possible cause for the teenagers symptoms? 1/1 appendicitis ectopic pregnancy drug overdose sexually transmitted disease

C Feedback the grandmother's concern is warranted. In teenage girls, the most common form of suicide attempts by drug overdose. The combination of vomiting and confusion suggest a drug overdose, and the family nurse practitioner should run a toxicology screen

The mother of a 3-year-old child takes the child to a play group once a week. She expresses concern that the child plays with toys but does not interact with the other toddlers. What will the Family nurse practitioner counsel the mother? 1/1

C The child probably is very shy but will outgrow this tendency with repeated exposure to other children. The toddler may have a language delay that interferes with socialization with other children. Toddlers may be interested in other children but usually do not engage in interactive play. Toddlers need more structured play to encourage interaction and socialization with others.

which newborn screening tests are mandatory State requirements? 0/1 complete blood count in urinalysis thyroid function tests and phenylketonuria (PKU) PK you and Alpha fetoprotein glucose and thyroid function tests Correct answer thyroid function tests and phenylketonuria (PKU) Feedback

C neonatal screening for PKU and congenital hypothyroidism is done in all states. Other tests are generally mandated by the state or obtained by the provider because of family history. Alpha fetoprotein is usually a maternal blood test done during the prenatal period to screen for genetic defects, primarily neural tube anomalies.

the family nurse practitioner notes an undescended testicle on a newborn. She understands that testicular function and ability to produce healthy sperm as an adult may be impaired if the repair is not made by age: 1/1 6 years 2 years 1 year 6 months Feedback

C normal morphology and testes tissue development will be impaired if the testes are not descended fully by age one year. These children are at a higher risk for development of testicular cancer in the young adult male between 20 and 30 years of age

A 55-year-old male patient is being seen by an advanced practice registered nurse (APRN) for the inability to complete intercourse due to erectile dysfunction. The APRN completes a thorough review of the patient's previous health history and medication usage. What should the APRN consider as a possible cause? Side effect of taking levofloxacin History of untreated prostate cancer Side effect of taking atenolol History of treatment for HIV

Correct! Atenolol is a beta-blocker used to treat angina and hypertension that has a side effect of erectile dysfunction.

A young adolescent female is observed to have mild unequal scapula prominences on gross examination while standing. In the Adams forward bending position, this inequality disappears. What will the Family practitioner do? 0/1 Discuss posture and exercise and ask about backpacks and books Obtain radiographic studies of the entire spine and neck. Reassure the child's parent that functional scoliosis will self-resolve. Refer to an orthopedic specialist for evaluation and possible bracing.

Correct answer Discuss posture and exercise and ask about backpacks and books

What will the Family nurse practitioner elicit when obtaining a positive Barlow maneuver when screening for developmental dysplasia of the hip? 0/1 Dislocation of an unstable hip Dropping of the iliac crest with a raised leg Reduction of a dislocated hip Unequal knee heights in a supine child

Correct answer Dislocation of an unstable hip

A 10-year-old child has had abdominal pain for 2 days, which began in the periumbilical area and then localized to the right lower quadrant. The child vomited once today and then experienced relief from pain followed by an increased fever. What is the likely diagnosis? 0/1 Appendicitis with perforation Gastroenteritis Pelvic inflammatory disease (PID) Urinary tract infection (UTI)

Correct answer Appendicitis with perforation

A 15-year-old patient is diagnosed with juvenile idiopathic arthritis (JIA). The Family nurse Practitioner should know that which methods are used to treat JIA? (Select all that apply) 0/1 Biologic drug therapy systemic corticosteroids Assisted mobility therapy joint replacement therapy Oral nonsteroidal anti-inflammatory drug (NSAID) therapy

Correct answer Biologic drug therapy systemic corticosteroids Oral nonsteroidal anti-inflammatory drug (NSAID) therapy

The Family nurse practitioner is managing care for a child diagnosed with iron-deficiency anemia who had an initial hemoglobin of 8.8 g/dL and hematocrit of 32% who has been receiving ferrous sulfate as 3 mg/kg/day of elemental iron for 4 weeks. The child's current lab work reveals elevations in Hgb/Hct and reticulocytes with a hemoglobin of 10.5 g/dL and a hematocrit of 36%. What is the next step in management of this patient? 0/1 Continue the current dose of ferrous sulfate and recheck labs in 1 to 2 months. Discontinue the supplemental iron and encourage an iron-enriched diet. Increase the ferrous sulfate dose to 4 to 6 mg/kg/day of elemental iron. Refer the child to a pediatric hematologist to further evaluate the anemia.

Correct answer Continue the current dose of ferrous sulfate and recheck labs in 1 to 2 months.

nighttime extremity pain in school-age children that is deep but not present in the joints and that may be caused by inflammation of the muscle bodies and tight fascial sheaths during periods of high activity is: 0/1 Osgood-Schlatter disease patellofemoral stress syndrome Growing Pains shin splints .

Correct answer Growing Pains Feedback Growing Pains usually occur at night and resolved by morning. The pain is deep and does not involve Two Joints. Osgood-Schlatters disease results from degeneration of the tibial tubercle because of overuse and a rapid growth spurt. Pain and swelling of her over the tibial tubercle. symptoms are exacerbated by activities that involve the quadriceps muscle. Another form of overuse syndrome is patellofemoral stress syndrome. Pain of a dull, aching quality is present in the knee, sometimes with clicking. Long periods of sitting or activities that involve knee flexion and compression of the patella in the groove cause increased pain. In shin splints, inflammation of muscles on the medial shaft of the tibia results from overuse and causes aching pain. Rest improves the pain. Improper warm up exercises or extended exercise by an unconditioned person, especially in unsuitable shoes, can lead to this pain

A school-age child falls off a swing and suffers a closed fracture of the right clavicle. How will this be managed? 0/1 Application of a figure-eight clavicle brace for 6 to 8 weeks Hospitalization for traction of the affected extremity and shoulder Immobilization with a sling to support the affected extremity Referral to an orthopedic specialist for possible surgical reduction

Correct answer Immobilization with a sling to support the affected extremity

A child is diagnosed with Crohn disease. What are likely complications for this child? Cancer of the colon and possible colectomy Intestinal obstruction with scarring and strictures Intestinal perforation and hemorrhage Liver disease and sepsis

Correct answer Intestinal obstruction with scarring and strictures

During a well child examination of an infant, the Family nurse practitioner notes 10 café au lait spots on the infant's trunk. What is the potential concern associated with this finding? 0/1 Endocrine disorders malignancy Neurofibromatosis Sturge-Weber syndrome

Correct answer Neurofibromatosis

an adolescent complains of right knee pain immediately after running in track practice. On exam, then he is warm to touch, and a tender, swollen tibial tuberosity is noted. What does the family nurse practitioner suspect? 0/1 Osgood-Schlatters disease rheumatoid arthritis acute tendonitis post traumatic knee effusion

Correct answer Osgood-Schlatters disease Feedback Osgood-Schlatters disease or tibial tubercle apophysis is characterized by a painful, self-limiting tibial tubercle swelling that leads to knee pain, especially during periods of rapid growth. Extension of the knee against resistance or application of pressure over the tibial tubercle aggravates the pain. Pain worsens with activity and subsides with rest.

A 9-month-old infant has vesiculopustular lesions on the palms and soles, on the face and neck, and in skin folds of the extremities. The Family nurse practitioner notes linear and S-shaped burrow lesions on the parent's hands and wrists. What is the treatment for this rash for this infant? 0/1 Ivermectin 200 mcg/kg for 7 to 14 days, along with symptomatic treatment for itching Permethrin 5% cream applied to face, neck, and body and rinsed off in 8 to 14 hours Treatment of all family members except the infant with permethrin 5% cream and ivermectin Treatment with permethrin 5% cream for 7 days in conjunction with ivermectin 200 mcg/kg

Correct answer Permethrin 5% cream applied to face, neck, and body and rinsed off in 8 to 14 hours

A preschool-age child who attends day care has a 2-day history of matted eyelids in the morning and burning and itching of the eyes. The Family nurse practitioner notes yellow-green purulent discharge from both eyes, conjunctival erythema, and mild URI symptoms. Which action is correct? 0/1 Culture the conjunctival discharge. Observe the child for several days. Order an oral antibiotic medication. Prescribe topical antibiotic drops.

Correct answer Prescribe topical antibiotic drops.

The Family nurse practitioner provides a teaching session to the nursing staff regarding osteosarcoma. Which statement by a member of the nursing staff indicates a need for information? 0/1 The femur is the most common site of this sarcoma The child does not experience pain at the primary tumor site. Limping, if a weight-bearing limb is affected, is a clinical manifestation. The symptoms of the disease in the early stage are almost always attributed to normal growing pains

Correct answer The child does not experience pain at the primary tumor site. Feedback Osteosarcoma is the most common bone cancer in children. Cancer usually is found in the metaphysis of long bones, especially in the lower extremities, with most tumors occurring in the femur. Osteosarcoma is manifested clinically by progressive, insidious, and intermittent pain at the tumor site. By the time these children receive medical attention, they may be in considerable pain from the tumor. Options 1, 3, and 4 are accurate regarding osteosarcoma.

A child has an area of inflammation on the neck that began after wearing a hand-knot woolen sweater. On examination, the skin appears chafed with mild erythematous patches. The lesions are not pruritic. What is an appropriate initial treatment? 0/1 Application of a lanolin-based emollient Burow solution soaks and cool compresses Oral antihistamines given 4 times daily Topical corticosteroids applied 2 to 3 times daily

Correct answer Topical corticosteroids applied 2 to 3 times daily

An infant is brought to clinic with bright erythema in the neck and flexural folds after recent treatment with antibiotics for otitis media. What is the treatment for this condition? 0/1 Oral fluconazole 6 mg/kg on day 1, then 3 mg/kg/dose for 14 days 1% hydrocortisone cream to affected areas for 1 to 2 days Topical keratolytics and topical antibiotics for 7 to 10 days Topical nystatin cream under an occlusive dressing for 3-4 weeks

Correct answer Topical nystatin cream under an occlusive dressing for 3-4 weeks

A school-age child has several annular lesions on the abdomen characterized by central clearing with scaly, red borders. What is the first step in managing this condition? 0/1 Fluoresce the lesions with a Wood's lamp. Obtain fungal cultures of the lesions. Perform KOH-treated scrapings of the lesion borders. Treat empirically with antifungal cream.

Correct answer Treat empirically with antifungal cream.

What syndromes are associated with celiac disease? (select all that apply) 0/1 Type 1 diabetes cerebral palsy turner syndrome williams syndrome downs syndrome IGA selective deficiency

Correct answer Type 1 diabetes turner syndrome williams syndrome downs syndrome IGA selective deficiency

a six-year-old complains that his legs hurt. His mother states that he has complained for the past two weeks, and she thought it was from just playing outside too much. When asked to identify the painful areas, the child points to the mid shaft of the femurs. He grimaces slightly when asked to walk. What should be part of the differential diagnosis? 0/1 Osgood-Schlatter disease Growing Pains acute lymphocytic leukemia (ALL) psychogenic pain

Correct answer acute lymphocytic leukemia (ALL) Feedback bone pain is common in children, especially adolescents. However, a six-year-old with complaints pain should be evaluated for acute lymphocytic leukemia. Osgood-Schlatter produces pain in the knees. Growing pain usually occurs at night time. There is no information from history to suggest psychogenic pain, but ALL must be considered since it is the most common malignancy in children. The child should be assessed for Lymphadenopathy since it's a company's bone pain in acute lymphocytic leukemia at least 50% of the time.

18 year old taking Isoniazid (INH) prophylactically for exposure to TB complains of headache, palpitations, rash, and diarrhea. What would the management be based on? avoidance of foods containing tyramine and histamine addition of Pyridoxine to the diet changing therapy from INH to Rifampin evaluation for hepatic impairment

Correct answer addition of Pyridoxine to the diet Feedback Pyridoxine (vitamin B6) is added to prevent peripheral neuropathy. Foods containing tyramine and histamine (tuna, aged cheese, yeast, vitamin supplements) cause interaction with monoamine oxidase Inhibitors. Anorexia, jaundice, malaise, and fatigue would be signs of hepatic involvement

A preschool-age child undergoing chemotherapy experiences nausea and vomiting. Which of the following would be the best intervention to include in the child's plan of care? 0/1 administer tube feedings small frequent meals fluids only between meals allow child to eat what he/she chooses

Correct answer allow child to eat what he/she chooses Feedback While all options can be done to encourage nutrition, allowing the preschooler choices meets two issues: nutrition and developmental tasks

A 9 year old has been diagnosed with Lyme disease. Which drug should be use to treat him? 0/1 doxycycline amoxicillin azithromycin cephalexin

Correct answer amoxicillin Feedback amoxicillin, doxycycline, and cefuroxime have all been shown to have equivalent efficacy for treatment of Lyme disease. However, because this patient is 6 years old, he should not be given doxycycline as a first-line treatment unless the other regiments are contraindicated. Macrolides like azithromycin should not be used for first line because they are poorly effective at eradicating infection. They may be used in patients who are intolerant of penicillins and cephalosporins and who cannot take doxycycline. First generation cephalosporin is like cephalexin or poorly effective and should not be used.

an 18 month old child is diagnosed with bronchiolitis.his respiratory rate is 28 breaths per minute. Which choice below is most appropriate for patient management? 0/1 antipyretics nebulized bronchodilators inhaled steroids antibiotics

Correct answer antipyretics Feedback bronchiolitis is a viral infection and antibiotics would be inappropriate for management. Since fever commonly accompanies this disorder antipyretic such as acetaminophen and ibuprofen are commonly used. Bronchiolitis is characterized by wheezing. A normal respiratory rate for an 18 month old is 2230 breaths per minute. Bronchodilators, especially nebulized and inhaled oral steroids are commonly employed to decrease respiratory effort. However, these interventions lack proven benefits, increase the cost of care, and have many side effects

a healthy 7 year old child is diagnosed with atypical pneumonia. He is febrile but not in distress. What is the preferred treatment for him? 0/1 supportive measures, it is probably viral amoxicillin, doses 80-90 mg/kg/d azithromycin doxycycline

Correct answer azithromycin Feedback streptococcus pneumoniae is the most common bacterial pathogen in all age groups. Atypical pathogens such as mycoplasma pneumonia and chlamydia pneumonia or common in children less than 5 years of a 2% with abrupt onset of symptoms including malaise, cough, sore throat and headache, but are not ill enough for hospitalization. The initial, preferred antibiotic to treat a typical community-acquired pneumonia is a macrolide antibiotic such as azithromycin. Alternative options include levofloxacin and doxycycline if the patient cannot take a macrolide. Amoxicillin would be an appropriate choice for the patient with pneumonia related to ask pneumonia

a child has a history of several episodes of acute pyelonephritis and has known renal scarring. What should be performed on follow-up visits? 0/1 blood pressure measurement streptozyme test urine dipstick for leukocyte esterase urine culture and sensitivity

Correct answer blood pressure measurement Feedback if a child suffered several episodes of acute pyelonephritis or has known retinal scarring, measure blood pressure at follow-up visits and also consider a urine specimen for proteinuria. Leukocyte esterase is a screening test used to test for white blood cells in the urine, which may indicate a urinary tract infection. A urine culture and sensitivity would be done if a urinary tract infection was suspected. A streptozyme test it a test for B hemolytic streptococcus, which is associated with glomerulonephritis not pyelonephritis.

a four month old patient presents to the office with a history of several days of rhinorrhea, cough, low-grade fever, increased respiratory rate, mild sub costal retractions, wheezes, and crackles. What is the most likely diagnosis? 0/1 croup epiglottitis tracheitis bronchiolitis

Correct answer bronchiolitis Feedback croup, epiglottitis, and tracheitis are all middle respiratory tract infections with a rapid onset. Bronchiolitis is a lower respiratory tract infection that has a more gradual onset it and no barking sound to the cough. Diagnosis can be made on clinical picture or with a viral panel. Treatment is supportive therapies such as nasal infection, fluid, and antipyretics. If oxygen saturations are low or infant cannot feed, the child may require hospitalization

a definitive diagnosis of sickle cell anemia can be made: 0/1 by a CBC by a hemoglobin electrophoresis a pathologist with visual examination of red cells with a serum ferritin, our level, and hemoglobin

Correct answer by a hemoglobin electrophoresis Feedback sickle cell anemia is initially suspected on visual exam of the red cells. They have a sickled shape, hence the name sickle cell anemia. These cells can be identified as early as 3 months of age, when a positive screen is identified, it is repeated using either hemoglobin electrophoresis or a DNA analysis

on exam of a child, the family nurse practitioner notes weak femoral pulses. This finding is associated with what condition? 0/1 Patent ductus arteriosus coarctation of the aorta tetralogy of fallot pulmonary stenosis

Correct answer coarctation of the aorta Feedback weak or absent pulses are associated with coarctation of the aorta and are not indicative of the other cardiovascular diseases. Patent ductus arteriosus may present with a widened pulse pressure and a consistent murmur. Pulmonary stenosis presents with fatigue, tachycardia, cyanosis or syncope

High School athlete presents to the clinic with concerns regarding her menstrual period. She states she has not had a period In the last two months. She has been in training and running about 3 miles a day for the last 3 months. She has lost approximately 15 lb. Her height is about 63 in, and she currently weighs 100 lb. What is the best response by the family nurse practitioner? 0/1 determine the patient's percentage of body fat and body mass obtain follicle-stimulating hormone serum levels determine serum levels of human chorionic gonadotropin (HCG) order thyroid function tests

Correct answer determine serum levels of human chorionic gonadotropin (HCG) Feedback pregnancy is the most common cause of amenorrhea in young women. It's important to rule out pregnancy by obtaining a serum HCG level in a patient with a problem of amenorrhea, Even if she is very athletic. interviewing the patient regarding her sexual practices is or unreliable. The presence or absence of pregnancy should be determined before other diagnostic studies.

many factors can contribute to the risk of congenital heart disease. Which maternal disease carries a higher risk of transposition of the great vessels (TGA), ventricular septal defect (VSD), and hypertrophic cardiomyopathy? 0/1 hypothyroidism diabetes lupus hypertension

Correct answer diabetes Feedback maternal diabetes increases the risk of the above name diseases. Some genital infections like cytomegalovirus, herpes virus, rubella, or Coxsackie virus can increase the risk of cardiac structural abnormalities. TORCH is an acronym used to describe five maternal infections that are deleterious to the fetus. The acronym TORCH stands for toxoplasmosis, other (syphilis), rubella, cytomegalovirus (CMV), herpes simplex virus (HSV)

Which of the following is most common in attention deficit disorder (ADD)? 0/1 this is more common in girls less than nine years of age family history does not play a role in this disorder hyperactivity must be present for this diagnosis dsm-v is used to diagnose a child with ADD

Correct answer dsm-v is used to diagnose a child with ADD Feedback ADD and ADD with hyperactivity or two separate diagnosis. This disorder is more common in boys with a 5 to 1 ratio and symptoms must be present by age 12 for at least six months before diagnosis can be made. Dsm-v should be used to diagnose children. Parents and or teacher should establish specific elements. Examples include fidgeting, difficulty remaining in seat, excessive talking, and patience when asked to wait their turn, blurting out answers before time, and interrupting conversations. These must be established in more than one environment.

The viral gastroenteritis seen in older children and adults has a short incubation of 18 to 72 hours and a short duration of 24 to 48 hours, is characterized by abrupt onset of nausea and abdominal cramps, followed by vomiting and diarrhea, and is often accompanied by headache in myalgia. What causes this disorder? 0/1 enteric adenovirus enteric calicivirus rotavirus cytomegalovirus

Correct answer enteric calicivirus Feedback gastroenteritis is a common cause of abdominal pain in children. Symptoms vary depending on the type of viral infection. Rotavirus mainly affect Infants 3 to 15 months of age in the winter months, causing voluminous watery diarrhea without leukocytes. enteric adenoviruses are the second most common viral infection in infants, the symptoms similar except the duration of the illness maybe longer. Enteric Calicivirus (Norwalk ) mainly causes vomiting but also diarrhea in older children and adults. Duration of symptoms is short, usually 24 to 48 hours. Cytomegalovirus rarely causes diarrhea in his comment after bone marrow transplant and in late stages of HIV infection.

an infant with congenital hypothyroidism is being discharged home. What would the family nurse practitioner instruct the parents to do? 0/1 watch for constipation and slow pulse as signs of toxicity reduce the medication as symptoms decrease give the medication as a single dose in the early morning on an empty stomach expect weight loss until a child adjust to the dose of medication

Correct answer give the medication as a single dose in the early morning on an empty stomach Feedback thyroid replacement is lifelong maintenance therapy and should be given as one dose in the morning and absorbs best one hour before or two hours after a meal. Weight loss, diarrhea, and tachycardia are signs of over medication

children with an inguinal hernia: 0/1 have a history of intermittent bulge in the groin are usually asymptomatic have a mass that is always present on exam are usually irritable and often constipated

Correct answer have a history of intermittent bulge in the groin Feedback hernias are usually asymptomatic in absent on exam but can sometimes be elicited by increasing intra-abdominal pressure such as occurred with straining or crying. The silk sign is infrequently appreciated but represents a silky thickening of the cord. If it is able to be palpated, it is done by placing a single finger next to the inguinal Canal at the level of the pubic tubercle and gently moving the finger from side to side. Children with an incarcerated mass are often irritable but not constipated

indications for antibiotic use in a child with bronchiolitis would include: 0/1 history of two episodes of bronchiolitis in 4 months rhinitis and a productive cough for 3 days high fever and increased crackles after patient had been improving for several days low grade fever, increased nasal secretions, and retractions while febrile

Correct answer high fever and increased crackles after patient had been improving for several days Feedback Rhinitis and a productive cough are common and are not cause for concern unless the mucus begins to change and accompanied by a high fever. The wheezes are also common, if the mucus is loose. A high fever and crackles after initially improving indicate further deterioration and possible bacterial infection (secondary pneumonia)

the test of choice to confirm and assess developmental dysplasia of the hip or DHH in a three month old is: 0/1 frog leg x-rays plain hip x-rays ultrasound of the hip CT of the hip

Correct answer ultrasound of the hip Feedback the primary Imaging technique for assessing developmental hip dysplasia in the first three to four months of life is ultrasound. The biggest limitation is the experience of the ultrasonography. After three to four months of age, AP X-rays of the hip or more valuable. The frog leg view of the pelvis provides the lateral view of the femoral head. During treatment or to confirm reduction postoperatively, a cat-scan may be valuable

what physical findings would lead the family nurse practitioner to suspect Hirschsprung's disease in a six-month-old infant? 0/1 rectal bleeding, diarrhea, and prolonged jaundice at Birth history of constipation and current abdominal distension irritability, vomiting and dehydration history of colic, bloody diarrhea, and nausea

Correct answer history of constipation and current abdominal distension Feedback classic signs and symptoms of Hirschsprung's disease in later infancy include alternating diarrhea and constipation and abdominal distension. The stools are expensive and ribbon like, the abdomen is enlarged, and the veins are prominent. The other listed physical findings are not usually found in Hirschsprung's disease but are prevalent and other gastrointestinal disorders.

a mother presents her school-age child to the family nurse practitioner and expresses her concern that her son is the shortest child in his class and asked to something is wrong with him. What would the initial differential of short stature in this patient by the family nurse practitioner include? 0/1 history with familial height patterns, physical exam with Tanner stage, in radiography to assess skeletal maturation, as indicated history with familial height patterns, physical exam, and trial treatment with growth hormone immediate referral to an endocrinologist physical exam and complete blood count, thyroid function panel, your analysis, karyotyping, chemistry profile, and insulin sensitivity tests

Correct answer history with familial height patterns, physical exam with Tanner stage, in radiography to assess skeletal maturation, as indicated this is the best choice for the initial exam because the history may reveal normal variation in pattern of growth related to race, heredity, size of other family members, and psychosocial status. Skeletal maturation or bone age can be accessed through radiography if child is in 5th growth percentile or lower. Other choices are inappropriate without history and physical exam, and growth hormone would not be administered without laboratory evaluation.

which of the following statements is true regarding tonsils? 0/1 children with large tonsils are more prone to tonsillitis than those with small tonsils tonsils and large as the child grows older hypertrophy of tonsils in children usually represent a normal finding most cases of tonsillitis are caused by B hemolytic streptococcus infection

Correct answer hypertrophy of tonsils in children usually represent a normal finding Feedback enlarged tonsils are common in young children. As the child grows older, the tonsils recede in size. Only about 25 to 30 percent of tonsillitis is caused by B hemolytic streptococcus. Having large tonsils does not make a child more prone to tonsillitis.

a mother reports that her toddler awoke this morning with eyelid redness and swelling. The child is afrebile and the eyelid is non-tender and uniformly swollen. What is the most likely diagnosis? 0/1 blepharitis hordeolum insect bite dacrocystitis

Correct answer insect bite Feedback generalized, diffuse swelling and erythmea of the eyelid is associated with an insect bite. Blepharitis is a chronic inflammatory condition characterized by erythema and scaling of the lid margins. hordeolum, or stye, is an acute, purulent inflammation of the sebaceous glands, usually the glands of Zeis or the meibomian glands of the eyelids and usually does not involve the entire lid. It may be painful, especially over the gland. Dacrocystitis is an inflammation of the lacrimal Sac that is characterized by erythema and swelling over the lacrimal duct.

what are the most consistent clinical findings in children with acute appendicitis? 0/1 an elevated white blood count and pyuria how favor and tenderness around the umbilicus low grade fever and Periumbilical abdominal pain nausea vomiting and diarrhea

Correct answer low grade fever and Periumbilical abdominal pain Feedback appendicitis usually begins as Periumbilical abdominal pain accompanied by anorexia and nausea but not necessarily vomiting or diarrhea. Within hours the patient may develop a low-grade fever. Most laboratory tests are normal. A white blood count greater than 15000/mm3 it's often noted but neither confirms nor excludes the diagnosis of appendicitis. Pyuria is not indicative of appendicitis and suggests renal disease

for the child with congenital heart disease (CHD) and a permanent pacemaker, electrical safety precautions include avoidance of: 0/1 cellular phones microwave ovens household electrical appliances metal detectors

Correct answer metal detectors Feedback metal detectors and electric fences have an electromagnetic field that could alter the Pacemakers function temporarily. In addition, the alarm will be set off as a result of the metal in the pacemaker. For the child with a pacemaker, and electric shock May irreparably damaged the pacemaker, and immediate surgical replacement would be necessary. There is no risk of electromagnetic interference between the permanent Pacemaker and household items such as electrical appliances, radios, electronic equipment, cellular phones, or microwave ovens. Most electrical appliances have filtering systems that prevent interference with the Pacemakers functions, and pacemakers utilize shielding, filters, and bipolar leads to mitigate electromagnetic interference.

a five-year-old child requires daily use of short-acting beta 2 Agonist for wheezing and the child symptoms exacerbate at least twice a week during physical exercise. The parent reports that the child has a dry cough at night, which occurred several times a week. The child's asthma is classified as: 0/1 mild persistent moderate intermittent moderate persistent severe persistent

Correct answer moderate persistent Feedback moderate persistent asthma in the child between 5 and 11 years is defined by daily symptoms, nighttime Awakenings more than once per week, daily SABA used, and some limitation in normal activity.any child taking a SABA daily is classified as having persistent asthma. Because the child symptoms happen on a daily basis, the severity is greater than mild. Additionally, because the child has some limitation in normal activity but it's not extremely limited the classification would not rank as severe

the family nurse practitioner understands that common causes of recurrent abdominal pain in children are: 0/1 Intussusception, gastroenteritis, and right lower lobe pneumonia psychogenic pain, trauma, and urinary tract infection parasitic infestation, lactose intolerance, and chronic stool retention incarcerated hernia, appendicitis, and inflammatory bowel disease

Correct answer parasitic infestation, lactose intolerance, and chronic stool retention Feedback parasitic infestation cause recurrent episodes of abdominal pain, often with diarrhea, nausea, and vomiting, depending on the top of infestation. Lactose intolerant can cause recurrent abdominal pain in the lower abdomen with cramping and distention. chronic stool retention occurs in a child with a history of ineffective toilet training. There is often a family history of constipation. All other conditions present with acute symptoms, except for psychogenic pain, which is recurrent and is a diagnosis of exclusion.

during a visit to the clinic for routine care, the mother reports that her infant was diagnosed with bronchopulmonary dysplasia (BPD). in addition, the infant has been vomiting after each gastrostomy feeding. The family nurse practitioner notes that the infant's weight gain is adequate and would recommend: referral to the pediatrician for follow up reduction in amount of formula for the gastrostomy feeding addition of three ounces of Pedialyte for the next two feedings positioning of the infant after feedings with head and trunk elevated

Correct answer positioning of the infant after feedings with head and trunk elevated Feedback maintaining a high caloric intake is important in the care of infants with BPD to promote growth and nourishment of developing lung tissue. It fits may have gastroesophageal reflux after eating. Initially, positioning should be tried, and then feedings may be thickened or given in smaller amounts more frequently. If the problem persists or weight gain is inadequate, referral to a pediatrician is indicated.

the most common cause of acute pharyngitis in children is: 0/1 S. pyogenes H. influenzae M. pneumonia respiratory viruses

Correct answer respiratory viruses Feedback the most common cause of acute pharyngitis is infection with viral agents. The most common viruses are adenoviruses, Coxsackie A virus, and parainfluenza virus. The most frequent bacterial cause of acute pharyngitis is streptococcus pyogenes. The most prevalent time of year for streptococcus outbreaks is winter

which reflex may be present at 9 months of age during sleep? 0/1 Moro reflex rooting stepping tonic neck

Correct answer rooting Feedback the rooting reflex disappears about two to three months of age but may be present during sleep up to 12 months. The moro reflex disappear somewhere between three and six months, usually at 4 to 5 months. The tonic neck or fencing response generally disappears at about 6 months of age. The stepping response disappears about 2 months and reappears at 8 to 12 months. Interestingly, this is usually when infants are beginning to take the first steps with assistance

what are the clinical manifestations of heart failure in an infant? 0/1 easily fatigued, central cyanosis, tachycardia, tachynpnea, and hepatomegaly coughing, diaphoresis, peripheral edema, and hepatomegaly tachycardia, tachypnea, easily fatigued, pallor, hepatomegaly peripheral edema, coughing, splenomegaly, hepatomegaly, and tachycardia

Correct answer tachycardia, tachypnea, easily fatigued, pallor, hepatomegaly Feedback most cases of heart failure in infants result from congenital heart disease during the first 12 months of Life. Symptoms result from the decreased cardiac output and the infant's compensatory mechanisms. Symptoms include tachypnea, dyspnea, tachycardia, pallor, and easy fatigability. Additional symptoms include periorbital edema, hepatomegaly, difficult feeding, and persistent cough. Diaphoresis, central cyanosis, and peripheral edema are not necessarily associated with heart failure that may be manifestations of the underlying congenital heart defect.

a 12 month old was screened for iron deficiency anemia and found to be anemic. The nurse practitioner ordered oral iron. In one month, the child's hemoglobin was reassessed. It increased greatly. Which answer my account for this? 0/1 the mother administer it with his bottle of milk the child is drinking apple juice the bone marrow is responding appropriately the child experienced a growth spurt

Correct answer the bone marrow is responding appropriately Feedback this child hemoglobin should have increased if he is receiving the iron as prescribed and there is no other disease process occurring. If the child's hemoglobin is not increasing, the instructions for administering the iron should be reviewed with the caregiver in the dosage should be recalculated. If the hemoglobin does not improve despite this, further assessment is needed

the family nurse practitioner is examining a 6 year old child and identifies 8 to 10 patches of coffee colored areas on the trunk. The areas are non-tender, the border is irregular, and most areas are larger than 1.5 cm and nonpalpable. What is the best recommendation to the child's parents? 0/1 the child should be referred to a neurologist these areas are normal pigmentation and will disappear a dermatologist should be consulted for removal of the lesions emollient should be applied to keep skin moist, and sunlight on the area should be avoided

Correct answer the child should be referred to a neurologist Feedback in pre-pubertal children, Cafe Au Lait spots larger than 1 cm and presented in more than five areas are a concern and may be associated with neurofibromatosis. The child should be referred to a pediatric neurologist for further evaluation

what is the usual recommendation about administering MMR and varicella immunizations? 0/1 they should not be given on the same day under any circumstances they should be given on the same day or at least 1 month apart they cannot be given with flu vaccine they can be given only with live viruses

Correct answer they should be given on the same day or at least 1 month apart Feedback MMR is an attenuated virus. Varicella is a live virus. They should be given on the same day or at least one month must separate the two. The reason for this is that higher titers are achieved if they are given together as opposed to being given separately

infective endocarditis prophylaxis may be required for children with congenital heart defects in which of the following procedures? 0/1 dental procedures such a simple adjustment of orthodontic appliances cardiac catheterization tonsillectomy and/or adenoidectomy insertion of tympanostomy tubes regurgitation.

Correct answer tonsillectomy and/or adenoidectomy Feedback procedures for which endocarditis prophylaxis is recommended include dental procedures known to induce gingival bleeding. It is also recommended for surgical procedures that involve the respiratory mucosa such as tonsillectomy and adenoidectomy. Endocarditis prophylaxis is not recommended for insertion of tympanostomy tubes, cardiac catheterization, Simple dental procedures, or endotracheal intubation. Cardiac catheterizations are done under sterile conditions, and prophylactic treatment is not recommended because of a very low incidence of infection. Endocarditis prophylaxis is for patients with prosthetic valves, prosthetic Rings or chords used in cardiac repair, previous infective endocarditis, unrepaired critical congenital heart disease or repaired congenital heart disease with shunts, valvular regurgitation or prosthetic devices, and cardiac transplant with valve

an adolescent male patient presenting with recent onset nocturia, polydipsia, polyphagia, weight loss, and blurred vision is most likely experiencing the symptoms of: 0/1 type 1 diabetes type 2 diabetes urinary tract infection mononucleosis

Correct answer type 1 diabetes Feedback type 1 diabetes usually but not always appears before age 30 and is heralded by the three p's including polydipsia polyuria and polyphagia

which clinical finding is not associated with acute glomerulonephritis in a child? 0/1 urine culture Escherichia coli greater than or equal to 50,000 Colony forming units per milliliter hypertension proteinuria tea colored urine

Correct answer urine culture Escherichia coli greater than or equal to 50,000 Colony forming units per milliliter Feedback there is usually evidence of a prior B hemolytic streptococcus infection. And infection with E coli is associated with acute pyelonephritis. Macroscopic hematuria which is tea colored urine, hypertension, proteinuria, decreased urine output, edema, dyspnea, fatigue, lethargy, and headache or common clinical findings

An adolescent has grouped vesicles on oral mucosa. To determine whether these are caused by HSV-1 or HSV-2, the Family Nurse practitioner will order which test? 0/1 Direct fluorescent antibody test Enzyme-linked immunosorbent assay Tzanck smear viral culture

Correct answer viral culture

You are preparing to perform a painful procedure on a four-month-old infant and will provide local anesthesia. Which other pain control method provides analgesic effects? Singing or music Sucrose solution Providing toys Swaddling or cuddling

Correct! A 12% sucrose solution given two minutes prior to a procedure will have analgesic effects for five to 10 minutes and is useful for infants. The other methods are distraction techniques, which are not analgesic but help small children cope with or reduce their fear response to pain. These are generally more useful in older children and not infants.

An advanced practice registered nurse (APRN) is seeing a 21-year-old patient to administer an HPV vaccination. The patient states that he self-identifies as homosexual but is having problems dealing with his sexual identity. Which intervention should the APRN provide for this patient? Check for status of hepatitis A and B vaccination Complete a depression screening Arrange for a psychiatric evaluation Test for hepatitis C

Correct! A depression screening should be done, as depression and suicide are more common in LGBTQIA individuals.

You are performing an examination on a two-year-old child who has been placed in emergency foster care with a grandparent after the child's mother was arrested. The child has a history of asthma with frequent exacerbations because of parental smoking. What should be your priority during this visit? Referring the child to a social worker to help the child deal with the emotional conflict related to separation from the parent Providing the grandparent with a list of websites and community-based support groups for grandparents parenting grandchildren Evaluating the child's financial resources, medical insurance, and access to healthcare and medications Teaching the grandparent about the need for consistent routines and discipline for the child

Correct! A high percentage of grandparents who parent grandchildren have financial difficulties, and most cannot claim grandchildren as dependents for healthcare. This child has a chronic disease and will need medication and possibly hospitalization, so you should assess the child's resources and access to care.

True or False A milk bleb is a tiny amount of visible milk under the skin due to a blocked nipple pore. True False

Correct! A milk bleb is a blister of visible milk under the skin that occurs when a milk duct opening is covered by a little bit of skin, causing milk to back up behind it

A 30-year-old male patient who has a history of epididymitis is being seen in a clinic by an advanced practice registered nurse (APRN). The patient is concerned that this condition is reoccurring. What should the APRN consider as a sign of this condition? A negative Murphy's sign A positive McBurney's sign A negative Prehn's sign A positive cremasteric sign

Correct! A positive cremasteric reflex can be considered a sign of epididymitis. The cremasteric reflex is a superficial reflex found in human males that is elicited when the inner part of the thigh is stroked.

A two-month-old infant has a staccato cough and fever. Which aspect of the infant's health history is most important to your diagnosis? Medication history Day care attendance Past medical history Immunization history

Correct! A staccato cough may be present with pertussis, which is a vaccine-preventable disease. Careful assessment of immunization history is important when this is suspected. Day care attendance is an important aspect of determining exposure and may be considered after the immunization history is reviewed. Medication and past medical history are probably not relevant in this case because it is unlikely that a two-month-old infant has been taking medications or has a chronic or recurrent illness.

A toddler who was born prematurely refuses most solid foods and has poor weight gain. A barium swallow study reveals a normal esophagus. What should you next consider in order to manage this child's nutritional needs? Consultation with a dietician Magnetic resonance imaging (MRI) Videofluoroscopy swallowing study Fiberoptic endoscopy evaluation

Correct! A videofluoroscopy swallowing study will evaluate other structural defects that may interfere with swallowing and will be relatively noninvasive. A dietician consult may be a part of the plan, but the toddler first needs a thorough evaluation of all potential problems. A fiberoptic endoscopy is invasive. Magnetic resonance imaging (MRI) may be performed if the videofluoroscopy is inconclusive, but it is an expensive test.

ou are examining a two-week-old infant and auscultate a wide splitting of S2 during expiration. Which condition may this finding represent? Atrial septal defect Ventricular septal defect Patent ductus arteriosus Coarctation of the aorta

Correct! A wide splitting of S2 that does not become a single sound on expiration may indicate increased pulmonary flow, typical of atrial septal defect. Coarctation of the aorta may cause a systolic ejection murmur. A patent ductus arteriosus has a machinery-like murmur. A ventricular septal defect has a harsh, high-pitched, grade II to IV/VI holosystolic murmur.

The parent of a four-month-old infant is concerned that the infant cannot hear. Which test should you order to evaluate potential hearing loss? Acoustic reflectometry Auditory brainstem response (ABR) Audiometry Evoked otoacoustic emission (EOAE) testing

Correct! ABR is not a direct measure of hearing, but it allows clinicians to make inferences about hearing thresholds and is useful for identifying hearing loss in a young infant. Although sedation is occasionally required, this test is useful in infants and young children who are unable to complete evoked otoacoustic emission (EOAE) testing or audiometry. Acoustic reflectometry is used to detect middle ear effusion. Audiometry requires that the child is able to cooperate. EOAE testing is used for universal screening in newborns. The American Academy of Pediatrics (AAP) Bright Futures guidelines (AAP, 2014) recommends pure tone audiometry at 3, 4, 5, 6, 8, 10, 12, 15, and 18 years of age.

You are examining a three-year-old child who speaks loudly in monotone, does not make eye contact, and prefers to sit on the examination room floor, moving a toy truck back and forth in a repetitive manner. Which disorder might this child have? Attention deficit hyperactivity disorder (ADHD) Sensory processing disorder Executive function disorder Autism spectrum disorder (ASD)

Correct! ASD manifests in toddlers by alterations in socialization and speech, along with repetitive behaviors.

Some members of the LGBTQIAP+ community have misconceptions regarding the need for preventative healthcare. True or False Pap smear screenings are unnecessary for lesbian women. True False

Correct! All women, including those who are lesbian, gay, or bisexual, need to have cancer screenings with a Pap smear test.

What should you recommend to a high school basketball player who becomes short of breath only when exercising? Treatment for exercise-induced asthma (EIA) with a bronchodilator Evaluation for underlying cardiac causes of this symptom Permanent discontinuation of all strenuous and aerobic activities Enrollment in a conditioning program to improve performance

Correct! Although shortness of breath may indicate several benign causes, athletes who exhibit this symptom should first be evaluated for underlying cardiac causes to prevent sudden cardiac death. Once this possibility is ruled out, other causes may be considered, such as exercise-induced asthma (EIA) or poor conditioning.

True or False Risk factors for erectile dysfunction include hypertension, diabetes, and depression. True False

Correct! Although the cause is not well understood, there are many risk factors for erectile dysfunction, including cardiovascular disease, endocrine conditions, and mood disorders, among others.

An advanced practice registered nurse (APRN) is seeing an adolescent patient for a sore throat, fever, and right earache. The APRN makes a diagnosis of acute otitis media and streptococcal pharyngitis and prescribes an oral antibiotic, but the patient's parent says the family is unable to afford the medication. Which action should the APRN take to assist this patient? Administer an antibiotic intramuscular injection Provide the mother with a list of local ear, nose, and throat (ENT) specialists Refer the patient to a local emergency department Report the mother to children's protective services (CPS)

Correct! An intramuscular injection would be appropriate for treating this bacteria. For group A beta-hemolytic streptococcus (GABHS), benzathine penicillin G (BPG) would be recommended.

Some of the major drug classes associated with erectile dysfunction include antihypertensives, antidepressants, and tranquilizers. True False

Correct! Antihypertensives, antidepressants, and tranquilizers have all been linked to the development of erectile dysfunction.

True or False Depression screening is suggested for all pregnant and postpartum women. True False

Correct! As many as one in every seven women (14 percent) suffers from postpartum depression. Screening is recommended for all pregnant and postpartum women.

An advanced practice registered nurse (APRN) is meeting a transgender patient for the first time. Which action should the APRN take to ensure that psychosocial considerations are addressed? Inquire about prior sexual orientation best practices Inquire about sexually transmitted diseases Ask if they identify as female Ask which pronoun they prefer

Correct! Asking patients which pronoun or terminology they prefer is the recommended approach. All patients should be encouraged to define themselves or self-identify.

An advanced practice registered nurse (APRN) is providing care for a patient who self-identifies as part of the LGBTQIA population. How should the APRN address the patient in this situation? Ask the patient to provide information about themselves and see where it leads Address the patient as "sir" and see if the patient provides a correction Address the patient in accordance to the sex they were assigned at birth Ask the patient if they have a girlfriend or boyfriend

Correct! Asking the patient to provide information about themselves is a good place to start on the first encounter. This gives the patient the opportunity to tell their story and allows them to define themselves or self-identify.

A 37-week gestation woman presents to your office for back pain with sciatica. She denies any injuries or falls, and her exam is essentially normal. What is the most appropriate treatment plan? Order an Xray of the lumbar spine and physical therapy Instruct her to take OTC acetaminophen and apply topical menthol Instruct her to take OTC ibuprofen Prescribe methocarbamol and naproxen

Correct! Back pain with sciatica is common in pregnancy, and since there is no mechanism of injury or abnormalities on exam, over-the-counter acetaminophen and topical menthol are appropriate.

A multidisciplinary approach to the treatment of male sexual dysfunction may include a psychologist, marriage counselor, sex therapist, and urologist. True False

Correct! Because male sexual dysfunction can be multifactorial, stemming from psychogenic, relational, or organic issues (often all three are involved), a multidisciplinary approach may include a psychologist, marriage counselor, sex therapist, and urologist.

What counsel should you give a new parent about ways to reduce the risk of sudden infant death syndrome (SIDS)? Bed-sharing with infants greatly increases the risk of SIDS. There is no difference in SIDS rates in immunized versus nonimmunized infants. Breastfeeding does not appear to have any influence on SIDS risk. Infants who attend day care have a higher than usual incidence of SIDS.

Correct! Bed-sharing with infants has been shown to increase the incidence of SIDS by five times, even with non-drug using and non-smoking parents; smoking, alcohol, and drug use by the parents increase this risk even further. Breastfeeding is recommended and is associated with a reduced risk of SIDS. Day care is not mentioned as increasing SIDS risk. Infants who are immunized have a 50% reduction in SIDS risk, according to research evidence.

Question 3 This is not a form; we suggest that you use the browse mode and read all parts of the question carefully. You are performing a well-child examination on a four-year-old child. The parent reports that the child frequently snores, often wakens during the night, and seems cranky during the day. What should you tell this parent? Sleep disorders at this age can have long-term effects on learning. Most sleep disorders are benign and will be outgrown. Longer daytime naps can compensate for lost sleep. Sleep disorders are symptomatic of underlying behavioral problems.

Correct! Behavioral sleep disorders and sleep-disordered breathing before age five can result in increased special education needs in children by age eight. Previousquestion

You meet a parent who encourages competitiveness in a child who excels at a single sport but not in other sports. What may this behavior also encourage? Worthiness Insecurity Competence Significance

Correct! Children who gain praise for external measures such as performance of a sport may end up unduly comparing themselves with others. They may feel insecure, inferior, and inadequate, even as they continue to excel in the sport.

A school-age child enjoys playing basketball but does not make the school team. Which response by the child shows a growth mindset? "I didn't play well on the day of the tryouts." "I'm probably too short to be really good at basketball." "I'll need to work more on my outside shot." "I'll just have to find another sport I'm good at."

Correct! Children who have a growth mindset have been taught to believe that hard work is the key to success and that effort and practice contribute to growth. By not being discouraged and identifying something to work on, the child is exhibiting a growth mindset.

You are evaluating a child who has short stature. Although their bone age studies reveal a delay in bone age, the child's growth is consistent with their bone age. Which diagnosis is most likely? Klinefelter syndrome Idiopathic short stature Growth hormone deficiency Constitutional growth delay

Correct! Children with constitutional growth delay have a delay in bone age but growth patterns consistent with bone age. Children with a growth hormone deficiency have a delay in both bone age and growth. Children with an idiopathic short stature will have a bone age consistent with their chronological age. Klinefelter syndrome is an overgrowth syndrome.

What should you tell a parent who is struggling to manage a child's difficult temperament? Children with difficult temperaments need strict adherence to rules. A difficult temperament limits intelligence and emotional maturity. A difficult temperament is a risk factor for maladjustment disorders. It is important for the parent to learn to manage criticism and power struggles.

Correct! Children with difficult temperaments tend to engender parental criticism and irritability, power struggles, and restrictive parenting. This dynamic leads to difficulties with psychosocial adjustment. Parents of these children must see the behavioral manifestations as temperament expressions. Doing so will help parents reframe their own responses to these behaviors. Restrictive parenting only increases power struggles and maladaptive behaviors

Which condition are children with limited mobility at a higher risk for? Asthma Obesity Fractures Liver disease

Correct! Children with limited mobility are at a higher risk for obesity and the secondary health conditions caused by obesity.

A school-age child has frequent nosebleeds. Nasal visualization reveals fresh clots and excoriated nasal mucosa, but no visible site of bleeding. The coagulation studies are normal. Despite symptomatic measures, the child continues to have nosebleeds. What should be the next course of action? Refer the child to an otolaryngologist for further evaluation Order a topical vasoconstrictor medication Cauterize the mucosa with silver nitrate sticks Prescribe a barrier agent such as petrolatum jelly

Correct! Children with persistent epistaxis should be referred for evaluation and treatment after usual symptomatic measures are ineffective. Cautery works well for exposed vessels, but the site must be easily accessible, visible, and not bleeding briskly. In this scenario, there is no visible site of bleeding. Topical vasoconstrictors are occasionally used. Petrolatum jelly has not been shown to be effective.

You are selecting a medication for a 12-year-old child who is newly diagnosed with ADHD. The child is overweight, has a history of an atrial septal defect at birth, and reports mild shortness of breath during exercise. What should you prescribe? Cardiovascular pre-screening A low-dose stimulant medication Behavioral therapy only A non-stimulant medication

Correct! Children with potential heart problems and symptoms such as previously detected cardiac abnormalities and shortness of breath with exercise should have a cardiovascular evaluation by a cardiologist prior to initiating ADHD treatment. If the screening and assessment are normal, a stimulant medication may be prescribed.

Which recommendation is most important to prevent a 14-year-old child from getting injured during basketball tryouts? Protective knee braces Stretching before practices Proper footwear Preseason conditioning

Correct! Conditioning in the preseason is one of the most important things children can do to build muscle strength, to prevent sports injuries, and to learn how to make twisting, jumping, and landing movements safely. Proper footwear is also recommended but is not the most important recommendation. Protective knee braces may be worn but do not prevent injury. Stretching should be done after warming up to maintain flexibility.

You are examining a 17-year-old male who is on his high school swim team. The adolescent is concerned about lumps on his chest. You note a marked increase in weight since the last visit, along with worsening of the adolescent's acne. Given this set of symptoms, you are concerned about the use of a performance-enhancing substance. Which substance should you ask him about? Ephedra Creatine Growth hormone Dehydroepiandrosterone (DHEA)

Correct! Dehydroepiandrosterone (DHEA) is a prohormone that is converted to either testosterone or estrogen. It causes adverse changes similar to anabolic steroids, such as weight gain, gynecomastia, and acne.

An adolescent exhibits mild depressive symptoms and tells you that he is most concerned about difficulty falling and staying asleep. The adolescent does not want to take medication to treat the depressive symptoms. What should you recommend to this patient? A sedative-narcotic drug to help both sleep and depression A program of sleep hygiene and gradual sleep extension An antidepressant to improve sleep patterns Cognitive therapy to help the adolescent to sleep better

Correct! Depression is linked to sleep problems, which predict and are predicted by a diagnostic cluster that includes depression, oppositional defiant disorder (ODD), and generalized anxiety disorder (GAD). One study found that gradual sleep extension combined with sleep extension advice had a beneficial effect on depressive symptoms of adolescents with chronic sleep reduction. Sedatives will not affect depression. Cognitive therapy is useful for insomnia related to anxiety. Antidepressants do not necessarily treat insomnia.

vA 40-year-old gay man presents to a clinic with complaints of dysuria, frequency, and urgency. He tells an advanced practice registered nurse (APRN) that he has had unprotected anal intercourse. The APRN diagnoses the patient with a urinary tract infection and recommends a treatment that will eliminate the bacteria causing the problem. Which bacteria is the APRN treating? Streptococcus pyogenes Borrelia burgdorferi Pseudomonas aeruginosa Escherichia coli

Correct! Escherichia coli (E. coli), a type of bacteria commonly found in the gastrointestinal tract, causes urinary tract infections.

An advanced practice registered nurse (APRN) is examining a 15-year-old male refugee for a sports physical for school. Which cultural patient-centered accommodation should this APRN provide? A video that illustrates the examination in the patient's language Pictures showing the steps of the physical examination Same-gender examiners when conducting examinations of genitalia An interpreter of the same age as the patient

Correct! Having examiners of the same gender would be a patient-centered accommodation for any patient at this age, especially males who must have a genital exam as part of the evaluation.

A patient presents to an advanced practice registered nurse (APRN) for a prenatal checkup for a second pregnancy. The patient developed gestational diabetes with the first pregnancy. Which lab test should the APRN perform for this patient? Corticotropin-releasing hormone Human chorionic gonadotropin Prolactin level Human placental lactogen

Correct! Human placental lactogen antagonizes the cellular action of insulin and decreases maternal glucose utilization, which increases glucose availability to the fetus. This may play a role in the pathogenesis of gestational diabetes.

A patient presents with pain and a lump in the testicle. True or False An ultrasound of the scrotum is recommended if the patient has no concerning or emergent clinical symptoms. True False

Correct! In the absence of clinical symptoms indicating emergent workup, an ultrasound of the scrotum is the preferred diagnostic test for testicular masses.

What is a contraindication for the rotavirus vaccine? Recent receipt of antibody-containing blood products Pregnancy History of intussusception Asthma

Correct! Infants with a history of intussusception should not be given the rotavirus vaccine. The maximum age for the last dose is eight months old

What is a critical component of screening for endocrine and metabolic disorders? Using the Centers for Disease Control (CDC) growth chart for children under age two years Obtaining serial measurements to assess patterns over time Using the World Health Organization (WHO) growth chart for children over age two years Measuring supine length in children over the age of two years

Correct! Measuring stature is an important part of a physical examination to screen for endocrine and metabolic disorders. Serial measurements are critical for assessing growth patterns over time. Supine length is measured in children under the age of two. CDC growth charts should be used for children older than two years, and WHO charts should be used for children under two years old.

A patient who is 36 weeks pregnant is seeing an advanced practice registered nurse (APRN) for a follow-up visit associated with a recent diagnosis of hypertension. The patient is taking atenolol and plans to breastfeed. Which change in medication should the APRN make for this patient? Acebutolol Methyldopa Furosemide Nadolol

Correct! Methyldopa is the drug of choice for the treatment of hypertension during pregnancy. Methyldopa is also considered compatible with breastfeeding.

A patient is admitted at 14 weeks gestation for hyperemesis gravidarum. An advanced practice registered nurse (APRN) prescribes 0.9% normal saline IV, vitamin B6, diphenhydramine, and metoclopramide. The vomiting is controlled; however, the patient begins to develop involuntary facial tics and eye blinking. Which action should the APRN take for this patient? Discontinue metoclopramide Insert an NG tube Increase the IV rate Decrease diphenhydramine

Correct! Metoclopramide is a phenothiazine and should be stopped immediately if there are any signs of tardive dyskinesia (involuntary movements, usually of the face). Early detection and discontinuation of the metoclopramide are important for the prevention of permanent tardive dyskinesia.

You are evaluating a 10-year-old child who is describing shooting pain in both legs associated with aching, tingling, and burning. The child is unable to pinpoint specific locations for this pain. Which type of pain may this be? Visceral Chronic Somatic Neuropathic

Correct! Neuropathic pain is poorly localized and characterized by shooting or stabbing pain against a backdrop of aching and burning and can be associated with paresthesia and dysesthesia. Chronic pain is rarely associated with sympathetic nervous system arousal. Somatic pain is well localized in skin and subcutaneous tissues and is generally dull or aching. Visceral pain involves the internal organs and is usually a continual aching sensation or a deep cramping or squeezing pain.

The mother of a three-month-old male infant tells you that she occasionally notices he has a penile erection just after nursing. What should you tell the mother? This is a normal, reflexive behavior at this age. Infants should be prevented from masturbating. This is a form of infantile priapism. The infant is conscious of the pleasure associated with nursing.

Correct! Newborn infants are reflexive beings, and sexual reflexes, which are present prenatally, are easily stimulated. A penile erection may occur while nursing. Infants explore with their hands and may touch their own genitalia for pleasure and comfort, and this is normal. A penile erection at this young age is reflexive and not conscious or intentional. It is not a form of priapism.

An advanced practice registered nurse (APRN) is examining a preschooler for sleep problems that stem from a fear that elephants live under the bed. The child's parent mentions that the child is also having problems at school, and the parent is using time-out in the child's bedroom as a consequence. The parent asks the APRN for recommendations to reduce the frequency of the fear-related sleep problem. Which recommendation should this APRN make? Tell the parent to avoid paying too much attention to the fear so it is not reinforced Recommend taking the child to a zoo to show the location where elephants live Encourage the parent to avoid using the room as a time-out space Refer the child to a mental health specialist to help work through the fear

Correct! Parents must be cautioned against using a bedroom as a time-out space. The child will be upset during that time, and the emotional response can carry over to when the bedroom is used for other purposes, such as sleeping.

During a well-child examination, you learn that a five-year-old child has had several episodes of walking out of the bedroom after falling asleep, looking dazed, with open eyes, and saying things that do not make sense. What should you recommend that the parent do? Awaken the child when these occur and ask about nightmares. Go to a sleep disorder clinic for evaluation of a parasomnia. Establish a graduated extinction program and good sleep hygiene. Make sure the stairs are blocked and doors are locked.

Correct! Parents of children with sleepwalking should be assured that this is relatively benign, but they should secure the house so the child will not cause self-harm. When sleepwalking, the child should be guided back to bed without being awakened. A referral to a sleep disorder clinic may be warranted if the child has an episode of leaving the house or some other dangerous activity.

An advanced practice registered nurse (APRN) is examining a five-year-old patient who is experiencing pain and burning during urination. The APRN asks the patient about the pain location, and the patient's mother interrupts and says, "Between her legs. She does not know anything about medical terms or reproduction. She is too young." Which response made by this APRN is accurate? Children at this age should be aware that sexuality is important at all ages. Children at this age should use correct words for all sexual body parts. Children at this age should be informed about human reproduction. Children at this age should be aware that all creatures grow and reproduce.

Correct! Patients in the two-to-five age range should be taught the appropriate names for body parts and functions, even though they may also be using slang words. This enables children to better comprehend discussions on the topic.

A patient with Down syndrome has sleep-disordered breathing with obstructive sleep apnea. He continues to have symptoms despite a tonsillectomy and adenoidectomy, as well as ongoing treatment with a leukotriene receptor antagonist medication and a nasal steroid spray. You refer the child to a sleep disorder clinic. Which therapy may the clinic consider? Oral appliances Craniofacial surgery Supplemental oxygen Positive airway pressure therapy

Correct! Positive airway pressure therapy can be used to treat sleep-disordered breathing in children who have failed other therapies. Even developmentally delayed children show improvement in behaviors after this therapy.

What is post-exposure prophylaxis (PEP)? Medications that prevent a patient from contracting HIV Vaccines to prevent the spread of disease Condoms and other barrier methods Antiretroviral drugs

Correct! Post-exposure prophylaxis (PEP) is antiretroviral drugs given to a person who has been exposed to HIV to reduce the risk of contracting the disease.

True or False The leading cause of infant mortality in the United States is birth occurring before 37 weeks gestation. True False

Correct! Premature births occur in about 12 percent of births but are responsible for 45 percent of infant mortality.

The parent of a school-age child reports that the child does not like being alone in rooms because of a fear of monsters hiding in closets. What should you tell the parent? "Your child is expressing normal fears for a school-age child." "Fear of imaginary creatures does not usually occur at this age." "Your child may be watching too much violence on television." "I may need to refer your child to a pediatric mental health specialist."

Correct! Preschoolers tend to fear imaginary creatures, not school-age children. You do not have enough information about the severity of the fear or its effects on behavior, so you should evaluate the situation further before doing anything else.

A 72-year-old male patient is speaking to an advanced practice registered nurse (APRN) about urinary hesitancy, urgency, nocturia, and frequency. Urinalysis confirms hematuria and elevated prostate specific antigen (PSA). During a digital rectal exam, the APRN notes an asymmetric prostate with a firm nodule. Which diagnosis should the APRN make for this patient? Prostatitis Hydrocele Benign prostatic hypertrophy Prostate cancer

Correct! Prostate cancer presents with urinary tract abnormalities, hematuria, elevated prostate specific antigen (PSA), and an asymmetric prostate with a firm nodule.

An advanced practice registered nurse (APRN) is providing care for a female patient who self-identifies as bisexual. The APRN is familiar with the top health issues identified for the LGBTQIA population in the Substance Abuse and Mental Health Services Administration (SAMHSA) report. Based on the SAMHSA report, which action should the APRN take for this patient? Encourage routine mammogram screening Collect BMI data and provide obesity counseling Assess for intimate partner violence Educate regarding the importance to start engaging in physical activity

Correct! Providers should routinely assess all clients for any history of domestic violence and/or victimization. Data show that bisexual adults are significantly more likely to report experiencing intimate partner violence than heterosexual adults.

Which populations may have higher rates of under immunization than others, according to recent research? Those with lower rates of poverty Those with a higher percentage of graduate degrees Those with a higher percentage of Asians Those with a lower number of primary providers

Correct! Rates of under immunization were increased in populations with an increased percentage of graduate degrees in a study of geographic clusters of under immunized communities in northern California. Populations having a higher percentage of Asians have increased immunization rates. Higher levels of poverty are associated with under immunization. The study did not look at the effect of the number of primary providers.

RhoGAM [Rho(D) Immune Globulin (Human)] is clinically indicated for an Rh-negative pregnant woman in which of the following situations? At 12 weeks gestation When pregnancy is confirmed Any time laboratory blood work is done on the mother Prophylactically at 28 weeks of pregnancy

Correct! RhoGAM or a similar Rho immune globulin should be administered prophylactically at 26 to 28 weeks gestation. Other indications for Rho immune globulin include abdominal trauma, ectopic pregnancy, spontaneous abortion, or any significant bleeding.

A dipstick urinalysis is positive for leukocyte esterase and nitrites in a school-age child who has dysuria and foul-smelling urine but no fever. The child has not had previous urinary tract infections. A culture is pending. What should you do to treat this child? Reassure the child's parents that this is likely an asymptomatic bacteriuria Wait for urine culture results to determine the correct course of treatment Prescribe trimethoprim-sulfamethoxazole (TMP) twice daily for three to five days Prescribe extended-release ciprofloxacin once daily for three days if the culture is positive

Correct! Short-term antibiotics of three to five days may be as effective as more intense medications for treating a urinary tract infection (UTI) in nonfebrile bladder infections. TMP is generally a first-line drug in children without history of UTIs. Ciprofloxacin is used in adolescents older than 18 years. This child is symptomatic with positive leukocyte esterase and nitrites and will need treatment. Asymptomatic bacteriuria occurs when bacteria are in the urine of a child who is asymptomatic.

What is the most emergent cause of testicular pain in a young male? Testicular torsion Epididymitis Varicocele Tumor

Correct! Testicular torsion is a surgical emergency. If surgery is performed within 12 hours after the onset of symptoms, the testis can be saved in about 90 percent of cases.

Men on testosterone replacement therapy should have a testosterone blood test Choose...24 hours12 hours1 week2 hours

Correct! Testosterone gel should be rubbed in two hours before drawing labs to monitor blood levels.

What lab tests should be done when monitoring testosterone supplement injections? Testosterone level Testosterone level and a comprehensive metabolic profile Testosterone and lipid level Hematocrit, prostate specific antigen, testosterone level

Correct! Testosterone has an erythropoietic stimulating effect that can cause polycythemia. Though testosterone therapy has not been shown to cause malignancy, testosterone can stimulate the growth of preexisting prostate cancer, and prostate specific antigen should be monitored. An elevated hematocrit is the most commonly reported adverse event from testosterone supplementation. Monitoring testosterone levels is recommended, with the goal of achieving levels in the mid-normal range.

You are counseling the parent of an overweight school-age child about improving the child's overall fitness. What should you tell the parent? School-age children need one hour of moderate exercise each day. Exercise will help lower total cholesterol and low-density lipoproteins. Encourage the child to begin by engaging in swimming or cycling. Strength training exercises are not safe for school-age children.

Correct! The American Academy of Pediatrics (AAP) suggests that overweight children should initially participate in activities that place less stress on weight-bearing joints, such as swimming or cycling.

A parent is concerned about adverse reactions to vaccines. What should you tell the parent, based on an Institute of Medicine (IOM) report? Administering multiple vaccines may trigger the development of type 1 diabetes. There is some risk of central nervous system (CNS) disorders associated with the hepatitis B vaccine. Vaccines containing thimerosal are linked to pervasive developmental disorders. The measles-mumps-rubella (MMR) vaccine may be linked to febrile seizures in immunocompromised children.

Correct! The IOM report found that febrile seizures and measles inclusion body encephalitis can occur in immunocompromised children. The IOM found no substantiated evidence that multiple vaccines trigger type 1 diabetes, that hepatitis B vaccine is associated with increased risk of central nervous system disorders (CNS), or that thimerosal-containing vaccines are linked to pervasive developmental disorders.

A developmentally and cognitively disabled 10-year-old child who is unable to communicate must undergo a series of surgeries, and the child's parent asks how to know if the child is in pain and when to give pain medication. What should you teach this parent to use to assess this child's pain? Non-Communicating Children's Pain Checklist-Revised (NCCPC-R) Face-legs-activity-cry-consolability (FLACC) scale Comfort scale Nonverbal observations

Correct! The Non-Communicating Children's Pain Checklist-Revised (NCCPC-R) is designed for use with children three to 18 years of age who have cognitive disabilities or physical impairments and are unable to communicate through speech. The comfort scale is used for mechanically ventilated children with normal cognition. The face-legs-activity-cry-consolability (FLACC) scale is generally used in children under the age of three years. Nonverbal observations are useful in infants and toddlers with normal development.

A nine-month-old infant has developed two teeth since the six-month checkup. The local water supply contains fluoride. What should you do to promote healthy dentition? Teach the parents how to brush the infant's teeth with fluoride toothpaste Prescribe oral fluoride supplementation Encourage the parents to make an initial dental appointment Apply sodium fluoride varnish to the infant's teeth

Correct! The U.S. Preventive Services Task Force has issued two recommendations for preventing caries in children. One recommendation is that primary providers apply a sodium fluoride varnish to the primary teeth of all infants and children beginning at the onset of the first tooth. The American Academy of Pediatric Dentistry recommends that children visit a dentist by age 12 months or six months after the eruption of the first primary tooth. Oral fluoride supplementation is given when local water supplies are fluoride deficient. Fluoride toothpaste is not recommended in infancy unless the child's risk for caries is high.

The mother of two school-age children tells you that she and the children's father are divorcing and asks for advice to help the children cope with the situation. What should you counsel her to do? Use a social support network Maintain her own social life Allow visitation only on weekends Notify the children's teachers

Correct! The availability of a social support system is a key factor in coping with divorce.

How is the estimated date of confinement calculated from the first day of the last menstrual period (FDLMP)? Add nine months to the FDLMP. Count back three months from the FDLMP, add one year, and subtract seven days. Add nine months and seven days to the FDLMP. Add nine months and subtract seven days from the FDLMP.

Correct! The estimated date of confinement (EDC) or "due date" may be determined by adding nine months and seven days (or 40 weeks) to the first day of the last menstrual period

A three-year-old child whose immunizations are up to date has been exposed to measles because of a localized outbreak among unvaccinated children. The parent reports that contact with infected children occurred within the last two days at a birthday party. What is the best course of action for you to take? Give antiviral medications at the first sign of symptoms. Reassure the parent that most exposed children will not get measles. Administer the measles-mumps-rubella (MMR) vaccine to help prevent disease. Give the child a measles-mumps-rubella (MMR) vaccine and a dose of immune globulin to provide both temporary and long-term protection.

Correct! The measles vaccine can be given to those exposed if given within 72 hours of exposure. Immune globulin (IG) may be given in those without a prior measles vaccine. Antiviral medications are not effective. Nine of 10 children who are unimmunized or under immunized will contract the disease if exposed.

You are performing a well-child examination on a fussy toddler who has red hair. The child's parent tells the toddler to stop being fussy and remarks to you that "Red hair gives him such a temper." Which type of common error is this? Stereotyping and typecasting Negating the child's feelings Expecting too much Dwelling on negatives

Correct! The parent is typecasting the child by associating a temper with the trait of red hair, which can limit his sense of possibilities.

A 30-year-old patient is 12 hours postpartum. The patient reports extreme fatigue due to prolonged labor, a feeling of fullness in the suprapubic region, and heavy blood loss >500 mL. The attending advanced practice registered nurse (APRN) assesses the patient and determines the patient needs postpartum obstetric hemorrhage care. Which set of steps should the APRN take to assist this patient? Increase the infusion rate of oxytocin, give methergine, and provide fundal massage Move patient to labor and delivery, give carboprost, and complete labs Repeat labs, transfuse with PRBCs and platelets, and consider ligation Infuse oxytocin, assess for vaginal lacerations, and prepare two units of PRBCs

Correct! The patient is at stage 1 of postpartum obstetric hemorrhage care and requires these actions. Stage 1 reflects blood loss >500 mL (vaginal delivery) or >1000 mL (cesarean delivery).

An advanced practice registered nurse (APRN) is conducting a precollege physical for a 17-year-old patient who plans to live in a dormitory. The patient is up to date on all immunizations. Which booster should be recommended? Meningococcal B Inactivated poliovirus DTaP HepB

Correct! The patient should have a meningococcal conjugate vaccine, as this helps protect against bacterial meningitis. College students living in dormitories are an at-risk group.

A patient presents with a low-grade fever, a heavy sensation in the testicles, and a sudden onset of severe pain that may be partially relieved by elevating the scrotum. What is the most likely diagnosis? Epididymitis Hydrocele Inguinal hernia Varicocele

Correct! The symptoms of epididymitis include low-grade fever, a heavy sensation in the testicles, and a sudden onset of severe pain that may be partially relieved by elevating the scrotum.

When discussing sexual health with a male who has sex with men, what is an important component of wellness education? Cancer screenings Maintaining a healthy weight Influenza vaccine recommendations Condom use

Correct! The use of condoms is important to prevent the spread of sexually transmitted diseases including HIV.

True or False A lactating mother should dump milk for eight hours after consuming ibuprofen. True False

Correct! There is no need to dump milk after taking ibuprofen. Ibuprofen is considered safe and compatible with breastfeeding.

An advanced practice registered nurse (APRN) is creating a plan of care for a 15-year-old patient, which includes family intervention recommendations that will provide age-appropriate support for values and beliefs. Which recommendation should be made by this APRN? Establish regular tasks for the individual in family activities Demonstrate a gentle approach in communication Allow for appropriate experimentation in dress Remove temptations from the environment .

Correct! This allows the individual to develop a sense of self and is identified as an appropriate intervention for an adolescent

You are seeing a 10-year-old child whose parents describe him as a class clown. The child denies having problems at school but acknowledges poor grades by saying, "I'm not very smart, I guess." Which strategy should you suggest to the parent to help the child deal with this self-perception issue? Help the child identify skills and activities that he is good at Empower the child to make decisions and assume more responsibilities Work with the teacher to set appropriate limits on school behavior Spend time each evening helping the child with homework to improve grades

Correct! This child exhibits problems with personal identity and copes by joking around to avoid dealing with inferiority problems regarding his school performance. The parent should work with the child to find his areas of strength and help the child become accomplished in those areas to improve his self-esteem.

A three-year-old child is brought to the clinic by a parent who reports that the child refuses to use his right arm after he was swung by both arms while playing. The child is sitting with the right arm held slightly flexed and close to the body. There is no swelling or ecchymosis present. What should you do? Obtain a radiograph of the child's right arm and elbow Consider maltreatment as a possible cause of injury Immobilize the arm with a sling and refer to orthopedics Gently attempt a supination and flexion technique

Correct! This is most likely an annular ligament displacement injury, or nursemaid's elbow. You can attempt to reduce the elbow using either a supination and flexion or a pronation technique. If this fails after three attempts, immobilization and referral are indicated. Consider maltreatment if the child has recurrent dislocations or other symptoms or signs are present. Radiologic studies are rarely necessary.

You are treating a toddler who has a lower respiratory tract illness with a low-grade fever. The child is eating and taking fluids well, and in the clinic, you determine that he has normal oxygen saturations. You suspect that the patient has a viral pneumonia. What should you do? Order an antiviral medication and schedule a follow-up appointment. Write a prescription for an antibiotic to be given if the child's condition worsens. Prescribe a broad-spectrum antibiotic until the lab results are received. Teach the parents symptomatic care and order labs to help with the diagnosis.

Correct! To decrease antibiotic overuse and resistance, you should order an antibiotic only if laboratory data confirm a bacterial infection. This child is mildly ill and can be treated symptomatically. It is unnecessary to treat with an antiviral medication. A broad-spectrum antibiotic will only increase the risk of antibiotic resistance. Writing a prescription for the parents to fill if needed is not recommended; parents may give an antibiotic believing that it is indicated when it is not.

The parent of a school-age child with a chronic pain condition tells you that during the past few months the child has frequently requested to stay home from school. The child's examination does not reveal any significant change in pathology, and a review of the child's medications indicates appropriate dosing of analgesic medications. What should you recommend to the parent? Require the child to go to school even during pain episodes Assess the child's pain every day to determine changes Ensure the child stays quiet in bed with videos when in pain Have the child do homework when staying home from school

Correct! To promote optimal coping with chronic pain in children, parents should not give excessive attention or special privileges when the child complains of pain. If allowing the child to stay home, parents should require quiet, low-key activities and not video games or television, which may reinforce the child not wanting to go to school. Parents who focus on the child's pain only reinforce the child's behaviors. Parents should encourage normal activities but not necessarily require them.

In a younger man without a sexually transmitted infection (STI), the presence of a urinary tract infection (UTI) is often associated with anatomic abnormality. True False

Correct! Urinary tract infections in males are uncommon, and many experts consider any UTI in a male to be complicated. Anatomic abnormality should be considered for a UTI in a younger man without an STI.

The mother of a 15-year-old female expresses concerns that her daughter may be sexually active because she has had a steady boyfriend for more than a year. You learn that the family is Catholic and that the mother had an abortion when she was 16 years old. What should you do initially? Recommend that the mother discuss this concern with her daughter. Offer to prescribe contraception to prevent pregnancy. Explore the mother's feelings about her own past experience. Suggest that the mother talk with a priest about her daughter.

Correct! When assisting families in managing ethical and behavioral issues, you should first assist parents and children in values clarification. Before offering other suggestions, you should help the mother to clarify her own feelings. The other options may be necessary after values are clarified.

The parent of a school-age child who has asthma tells you that the child often comes home from school with severe wheezing after gym class and needs to use his metered-dose inhaler right away. What should you do? Suggest asking the school to excuse the child from gym class. Write the prescription for two metered-dose inhalers with spacers. Recommend that the child go to the school nurse when symptoms start. Review the child's asthma action plan and possibly increase his steroid dose.

Correct! When children take medication at school or day care, you should dispense two units of the medication so that one can remain at school and one at home to avoid missed doses. The school nurse will not be able to order a medication that the child does not have available. The child is missing his rescue medication and just needs access to his inhaler. It is not necessary to excuse the child from gym class if his symptoms can be controlled.

Which symptoms will be present in a respiratory disorder that causes a check valve obstruction? Narrowing of the lumen with increased air flow resistance Obstruction of air entry with unimpeded expiratory air flow Complete obstruction on inspiration and expiration Air entry on inspiration with expiratory occlusion

Correct! With check valve or ball valve obstruction, air entry is possible, but the lumen is completely occluded during expiration, so air escape is impossible. A complete obstruction does not allow inspiratory or expiratory air flow. A bypass valve obstruction allows air flow but does so with increased resistance. There is no obstruction that impedes inspiration while allowing expiratory air flow.

When speaking to a transsexual, you should use the pronoun that corresponds to how they are dressed. True False

Correct! You should ask any LGBTQIAP+ patient what pronouns they prefer.

The parent of a two-month-old infant is reluctant to have the baby vaccinated. What should be your initial step in responding to this parent's concerns? Remind the parent that the infant is exposed to thousands of germs each day. Provide vaccine information statements (VISs) for the parent to review. Question the parent's reasons for concern about immunizations. Inform the parent that all vaccines may be given without thimerosal.

Correct! You should question and listen carefully to parents' concerns about vaccines. After concerns are identified and understood, you can address them. Common concerns are the presence of thimerosal in vaccines as well as the potential to overwhelm a child's immune system. These concerns should be addressed if identified. Providing a vaccine information statement (VIS) should be done as part of the discussion to provide information to the parent.

During a well-child examination of a six-year-old, you note that the child becomes embarrassed and resists taking off her underwear for the examination. What might you infer from this observation? The child is exhibiting regressive behavior. The child is feeling violated by the examiner. The child has been sexually molested. This is a normal reaction in a child of this age.

Correct! Young school-age children can be extremely modest and embarrassed and resist taking off their clothes for an examination. Because this is normal behavior, it does not indicate a history of sexual abuse unless other signs are present.

9 An advanced practice registered nurse (APRN) examines a six-month-old infant, and a piece of assessment data causes concern about the child's development. Which finding caused this response by this APRN? Failure to attempt feeding self with spoon Inability to pull self to standing position Inability to pick up toy with one hand Failure to double birth weight

D

A 13-year-old child has exhibited symptoms of mild depression for several weeks. The parent reports feeling relieved that the symptoms have passed but concerned that the child now seems to have boundless energy and an inability to sit still. What will the Family nurse practitioner do? 1/1 Administer an ADHD diagnostic scale and consider an Attention-deficit/hyperactivity disorder (ADHD) medication. Consult with a child psychiatrist to prescribe an antidepressant medication. Reassure the parent that this behavior is common after mild depressive symptoms Refer the child to a child psychiatrist for evaluation of bipolar disorder.

D

A 13-year-old is being seen by an advanced practice registered nurse (APRN) for a routine exam, and the parents mention the child does little to no exercise. The APRN notes that the child has gained 20 pounds in the last year and is concerned about the dramatic gain. Which action should the APRN take to assist this patient? Suggest joining the cross-country team Suggest entering local running contests Encourage enrollment in a baseball league Encourage participation in enjoyed activities

D

A 16-year-old adolescent female whose body mass index (BMI) is at the 90th percentile reports irregular periods. The Family nurse practitioner notes widespread acne on her face and back and an abnormal distribution of facial hair. The nurse practitioner will evaluate her further based on a suspicion of which diagnosis? 0/1 Nonalcoholic steatohepatitis dyslipididemia hypothyroidism polycystic ovarian syndrome

D

A 3-month-old infant who was previously healthy now has a persistent cough, bilateral lung crackles, and poor appetite. The Family nurse practitioner auscultates a grade III/VI, low-pitched, holosystolic murmur over the left lower sternal border and palpates the liver at one centimeter below the ribs. What diagnosis is likely? 0/1 Atrial septal defect (ASD) Coarctation of the aorta (COA) Patent ductus arteriosis (PDA) Ventricular septal defect (VSD)

D

A 3-month-old infant who was previously healthy now has a persistent cough, bilateral lung crackles, and poor appetite. The Family nurse practitioner auscultates a grade III/VI, low-pitched, holosystolic murmur over the left lower sternal border and palpates the liver at one centimeter below the ribs. What diagnosis is likely? 0/1 Atrial septal defect (ASD) Coarctation of the aorta (COA) Patent ductus arteriosis (PDA) Ventricular septal defect (VSD)D

D

A 30-month-old girl who has been toilet trained for 6 months has daytime enuresis and dysuria and a low-grade fever. A dipstick urinalysis is negative for leukocyte esterase and nitrites. What is the next step? 1/1 Begin empiric treatment with trimethoprim-sulfamethoxazole. Discuss behavioral interventions for toilet training. Reassure the child's parents that the child does not have a urinary tract infection. Send the urine to the lab for culture.

D

A 4-year-old child who has previously met developmental milestones is not toiled trained. The Family nurse practitioner notes decreased reflexes in the lower extremities and observe a dimple above the gluteal cleft. Which diagnosis may be considered for this child? 1/1 Arnold-Chiari malformation Reye syndrome Spina bifida cystica Tethered cord

D

A 67-year-old woman asks the advanced practice registered nurse (APRN) when her next Pap smear should be performed. The APRN notes that the client has had three consecutive negative cytology results. Which schedule should the APRN recommend for this patient? You Selected Repeat cervical cytology in three years Repeat cervical cytology in one year Repeat cervical cytology in five years Ongoing cervical cytology is not necessary

D

A 72-year-old patient comes to the clinic for routine follow-up care with an advanced practice registered nurse (APRN). The APRN performs a comprehensive physical exam, which is negative for findings, and then completes advanced care planning services. Why did the APRN complete this planning for this patient? It increases billing services for the practice. It is required by CMS for a patient of this age. It is indicated for this patient's disease process. It enhances self-determination and quality of care.

D

A child diagnosed with Group A beta-hemolytic streptococci (GABHS) 2 weeks prior is in the clinic with periorbital edema, dyspnea, and elevated blood pressure. A urinalysis reveals tea-colored urine with hematuria and mild proteinuria. What will the Family nurse practitioner do to manage this condition? 1/1 Prescribe a 10- to 14-day course of high-dose amoxicillin. Prescribe high-dose steroids in consultation with a nephrologist. Reassure the parents that this condition will resolve spontaneously. Refer the child to a pediatric nephrologist for hospitalization

D

A child has an acute infection causing lower airway obstruction. Which initial symptom is expected in this child? 1/1 Atelectasis Barrel chest Over-inflation wheezing

D

A child is in the clinic for evaluation of an asthma action plan. The Family nurse practitioner (PNP) notes that the child's last visit was for a pre-kindergarten physical and observes that the child is extremely anxious. What will the nurse practitioner do initially? 1/1 Ask the child's parent why the child is so anxious. Perform a physical assessment to rule out shortness of breath. Reassure the child that there is nothing to be afraid of. Review the purpose of this visit and any anticipated procedures.

D

A parent is concerned that a 12-month-old child is "bow-legged." A physical examination reveals internal tibial torsion bilaterally. A radiograph reveals asymmetric bowing of the legs with an angle greater than 15 degrees. What is the correct action for the Family nurse practitioner? 1/1 Observe the child's condition over time to assess progression. Order physical therapy to prevent progression of symptoms. Reassure the parent that the child will outgrow this deformity. Refer to a pediatric orthopedic specialist for treatment

D

A parent is concerned that a 12-month-old child is "bow-legged." A physical examination reveals internal tibial torsion bilaterally. A radiograph reveals asymmetric bowing of the legs with an angle greater than 15 degrees. What is the correct action for the Family nurse practitioner? 1/1 Observe the child's condition over time to assess progression. Order physical therapy to prevent progression of symptoms. Reassure the parent that the child will outgrow this deformity. Refer to a pediatric orthopedic specialist for treatment.

D

A three-week-old patient presents to an advanced practice registered nurse (APRN) with periorbital swelling and a fever. The patient's parent reports that a mark was noticed a week ago originally appearing to resemble a mosquito bite, and a fever began only the day before. On exam, the baby is febrile and has reduced intake, wet diapers, and periorbital swelling to the right eye. Which action should the APRN take for this patient? Consult an optometrist to treat conjunctivitis Consult a dermatologist to treat dermatitis Transfer to an emergency room to treat bullous impetigo Transfer to an emergency room to treat cellulitis

D

An 82-year-old patient comes to a clinic for routine care. The patient lives alone and has no family living close by. Which topic should the advanced practice registered nurse (APRN) discuss as part of the plan of care for this patient? Completing the Beers criteria Invoking durable power of attorney for healthcare Moving into a nursing home Reviewing advance directives

D

An adolescent female has a history of repaired tetralogy of Fallot (TOF). Which long-term complication is a concern for this patient? 0/1 Aortic stenosis Chronic cyanosis Mitral valve prolapse Ventricular failure

D

An adolescent male comes to the clinic reporting unilateral scrotal pain, nausea, and vomiting that began that morning. The Family nurse practitioner palpates a painful, swollen testis and elicits increased pain with slight elevation of the testis (a negative Phren's sign). What will the nurse practitioner do? 1/1 Administer IM ceftriaxone and prescribe doxycycline twice daily for 10 days. Encourage bed rest, scrotal support, and ice packs to the scrotum as tolerated. Prescribe NSAIDs, limited activities, and warm compresses to the scrotum. Refer the adolescent immediately to a pediatric urologist or surgeon.

D

An advanced practice registered nurse (APRN) is developing an educational program that provides information on the U.S. dietary guidelines for the acceptable macronutrient distribution range (AMDR) for school-age children. Which AMDR guideline should the APRN share in this education program? Protein consumption should be between 20-45% AMDR. Saturated fat consumption should be between 20-25% AMDR. Fat-soluble vitamin consumption should be between 20-50% AMDR. Carbohydrate consumption should be between 45-65% AMDR.

D

An advanced practice registered nurse (APRN) is evaluating a 16-month-old foster child. The foster parent is concerned because the child has been showing extreme fear anytime one of the foster parents leaves the child. Which action should the APRN take? Consult with a psychologist about the behavior Refer to a play therapist Instruct parent on behavior management skills Assure that this is normal behavior

D

An advanced practice registered nurse (APRN) is evaluating a 96-year-old male for possible pneumonia. The APRN identifies that the patient has pneumonia and requires antibiotics. The APRN is planning multimodal treatments for this patient. Which system should the APRN carefully evaluate first before recommending a wide-spectrum antibiotic? Dermatological Endocrine Neurologic Renal

D

An advanced practice registered nurse (APRN) is evaluating a patient in the clinic. The patient complains about having diarrhea for the last five days, a fever that started the day before, and abdominal bloating. The patient mentions trying to keep hydrated but is finding that goal difficult to achieve. What is an appropriate question for the APRN to ask during the history and physical? "Have you been exposed to new cats or dogs?" "Have you eaten any fried or overcooked food within the past week?" "Have you exposed anyone to your infectious stool?" "Have you traveled abroad recently?"

D

An advanced practice registered nurse (APRN) is examining a 70-year-old patient for complaints of fatigue, constipation, and dentures that are suddenly not fitting well. The APRN suspects dehydration. Which diagnostic tool will confirm this diagnosis? Urine culture and sensitivity Abdominal X-ray Lipid profile Complete metabolic panel

D

An advanced practice registered nurse (APRN) is listening to the heart sounds of a five-year-old patient and, upon auscultation, hears a fixed split S2 heart sound. Which action should the APRN take for this patient? Recheck in two months Consult with a pediatric pulmonologist Reassure the patient's parents this is a normal finding Refer to a pediatric cardiologist

D

An advanced practice registered nurse (APRN) is performing a well-child examination on a five-year-old boy and notes a common indicator that may signal sexual abuse is occurring. Which observation should this APRN recognize as a warning sign for this problem? The parent reports walking in on the son while he was masturbating. The boy tells the APRN that sex is how babies are made. The parent reports that the boy was caught playing naked with peers. The boy tells the APRN details about oral sex.

D

An advanced practice registered nurse (APRN) is speaking with the parent of a six-year-old child. The parent is concerned that the child does not get enough sleep each day. The APRN educates the parent about the amount of sleep this age child needs each day. How many hours of sleep daily should the APRN recommend for this child? 12-14 10-12 14-16 9-12

D

An advanced practice registered nurse (APRN) is treating a 96-year-old patient who is taking six different medications for chronic diseases. The APRN wants to improve compliance with the medications while avoiding pill burden. Which action should the APRN take to meet these goals? Reduce the frequency of all medications to once a day Encourage the patient to grind up all pills together Change the medications to liquid form Use combination medications to minimize prescriptions

D

An established two year old patient presents to the clinic today for a well-child visit and it's noted that his body mass index is below the 3rd percentile. The family nurse practitioner learns that the patient does not have regular mealtimes and it's allowed to carry a bottle of juice around at all times. Which diagnosis will the FNP use for this problem? 1/1 failure to thrive Home Care Resources inadequate nutrition altered - less than required parenting alteration

D

During a well child examination of a 2-year-old child, the Family nurse practitioner palpates a unilateral, smooth, firm abdominal mass which does not cross the midline. What is the next course of action that? 0/1 Order a CT scan of the chest, abdomen, and pelvis. Perform urinalysis, CBC, and renal function tests. Reevaluate the mass in 1 to 2 weeks. Refer the child to an oncologist immediately.

D

The Family nurse practitioner is offering anticipatory guidance to the parents of a 12-month-old child. The parents are bilingual in Spanish and English and have many Spanish-speaking relatives nearby. They are resisting exposing the child to Spanish out of concern that the child will not learn English well. What will the pediatric nurse practitioner tell the parents? 1/1 Children who learn two languages simultaneously often confuse them in conversation. Children with multi-language proficiency do not understand that others cannot do this. Learning two languages at an early age prevents children from developing a dominant language. Most bilingual children are able to shift from one language to another when appropriate.

D

The Family nurse practitioner is performing a well child examination on a 3-year-old. The child's parent reports that the child has recently begun masturbating. What will the nurse practitioner counsel this parent? 1/1 To allow the behavior whenever it occurs, since it is normal To discuss sexuality with the child To explore whether the child is being abused To teach the child about privacy and hand hygiene

D

The Family nurse practitioner is providing anticipatory guidance to the parent of a school-age boy. The parent expresses concerns that the child prefers to play with dolls, is worried that the child will be a homosexual, and asks what can be done to prevent this from happening. What will the nurse practitioner tell this parent? 1/1 Homosexual identity formation cannot be predicted by early childhood behavior. Masculinizing boys from an early age helps to determine heterosexual orientation. Sexual orientation identification begins late in adolescence and not in childhood. The development of sexual orientation is generally a multifaceted process

D

The Family nurse practitioner performs a physical examination on a 9-month-old infant and notes two central incisors on the lower gums. The parent states that the infant nurses, takes solid foods three times daily, and occasionally takes water from a cup. What will the pediatric nurse practitioner counsel the parent to promote optimum dental health? 1/1 To begin brushing the infant's teeth with toothpaste To consider weaning the infant from breastfeeding To discontinue giving fluoride supplements To make an appointment for an initial dental examination

D

The Family nurse practitioner performs a well baby exam on a term 4-month-old infant and observes flattening of the left occiput, bossing of the right occiput, and anterior displacement of the left ear. The parents report performing various positioning maneuvers, but say that the baby's head shape has worsened. What will the nurse practitioner recommend to correct this finding? 0/1 Allow the infant to sleep on the tummy when the parents are in the room. Lay the infant in the "back to sleep" position, alternating the left and right occiput. Order a head CT to evaluate the infant for craniosynostosis. Refer the infant for orthotic cranial molding helmet therapy.

D

The Family nurse practitioner provides primary care for a 5-year-old child diagnosed with cerebral palsy who exhibits athetosis and poor weight gain in spite of receiving high-calorie formula to supplement intake. The child has had several episodes of pneumonia in the past year. Which specialty consultation is a priority for this child? 0/1 Feeding clinic to manage caloric intake Neurology to assess medication needs Pulmonology for possible tracheotomy Surgery for possible fundoplication and gastrostomy

D

The advanced practice registered nurse (APRN) is performing a bimanual pelvic examination on a 30-year-old female patient. The APRN will be checking the patient's cervix for cervical motion tenderness and explains this to the patient. Which condition is the APRN evaluating in this patient? Adnexal cysts or masses Lesions of the vaginal walls Enlargement of the uterus Pelvic infection or inflammation

D

The clean catch urine specimen of a child with dysuria, frequency, and fever has a colony count between 50,000 and 100,000 of E. coli. What is the treatment for this child? 1/1 Obtain a complete blood count and C-reactive protein. Perform sensitivity testing before treating with antibiotics. Repeat the culture if symptoms persist or worsen. Treat with antibiotics for urinary tract infection.D

D

The mother of a 16-year-old male was recently divorced after several years of an abusive relationship and tells the Family nurse practitioner that the adolescent has begun skipping school and hanging out with friends at the local shopping mall. When she confronts her child, he responds by saying that he hates her. What will the nurse practitioner tell this mother? 1/1 Adolescence is marked by an inability to comprehend complex situations. Adolescence is typically marked by tempestuous and transient episodes. Adolescents normally have extreme, disruptive conflicts with parents. Adolescents often need counseling to help them cope with life events

D

The mother of a 3-month-old male infant tells the Family nurse practitioner that she occasionally notices he has a penile erection just after nursing. What will the nurse practitioner tell the mother? 1/1 Infants should be prevented from masturbating. The infant is conscious of the pleasure associated with nursing. This is a form of infantile priapism. This is a normal, reflexive behavior at this age

D

The parents of a 12-year-old child are concerned that some of the child's older classmates may be a bad influence on their child. What will the Family nurse practitioner tell the parent? 1/1 Allowing the child to make poor choices and accept consequences is important for learning values Children at this age have a high regard for authority and social norms, so this is not likely to happen Moral values instilled in the early school-age period will persist throughout childhood The pressures from outside influences may supersede parental teachings and should be confronted

D

To evaluate brain tissue disorders in infants, which test is useful? 0/1 Computerized tomography Head radiographs Magnetic resonance imaging Ultrasonography

D

Which statement is not supported by the ACIP general vaccination guidelines? 1/1 If two live virus parenteral vaccines are given less than 28 days apart, the vaccine given second should be disregarded; repeat this second vaccine at least 4 weeks later. When multiple vaccines are given on the same extremity, the sites of injection should be at least 1 inch apart; the anterolateral aspect of the thigh is preferred. In some circumstances (e.g., imminent travel, country epidemics, delayed immunizations) an accelerated schedule is available from the ACIP. While written, dated records are preferred, parent or guardian recollection of a child's immunization status may be accepted if source is considered reliabl

D

a two year old patient presents today with a diffuse, non-pruritic, discreet, rose-colored maculopapular rash located on the trunk. The lesions measure 2-3 mm in diameter and Fades on pressure. The provider learns that for the last three days the patient had a temperature of 101 degrees Fahrenheit and when the fever broke the rash appeared. What condition does this patient most likely have? 0/1 meningoencephalitis scarlet fever meningitis roseola infantum

D

the first sign that a male child is experiencing sexual maturation is: 1/1 increase in testicular size enlargement of the scrotum increase in length of the penis scrotal and penile changes

Feedback a male in Tanner stage II will have an increase in testicular volume from 1.5 ml or less up to 6 ml. The skin on the scrotum will begin to thin, and enlarge. The penile length will remain the same.

n advanced practice registered nurse (APRN) is performing a wellness exam for an adolescent and would like to evaluate a concern mentioned by the patient's parent about the patient's level of risk-taking behaviors. Which assessment tool should the APRN use to evaluate this concern? Massachusetts Youth Screening Instrument (MAYSI, MAYSI-2) Ages and Stages Questionnaires: Social-Emotional (ASQ:SE) Patient Health Questionnaire-9 (PHQ-9) Home, Education and Employment, Eating, Activities, Drugs, Sexuality, Suicide and Depression, and Safety (HEEADSSS)

D

n advanced practice registered nurse (APRN) is seeing a toddler for a physical exam prior to the child being enrolled in daycare. The APRN notes multiple cigarette burns on the child's buttocks. Which action should the APRN take first to assist this patient? Postpone the examination and speak to the parent about suspected abuse Treat the burns and schedule a return visit in one week to verify the child's condition Advise parent to return to the waiting room until the examination is complete Continue the examination and have the office staff notify child protective services

D

in evaluating a 16 year old female patient, which symptom would indicate anorexia nervosa? 1/1 refuses to discuss questions pertaining to food reflects a positive body image State she is eating very well but has episodes of vomiting the family states she refuses to stop her severe dieting

D Feedback adolescents with anorexia nervosa will severely reduce their nutritional intake by dieting constantly on high fiber and low calories. They usually have an inappropriate body image and may admit to occasional episodes of vomiting

while interviewing an adolescent female presenting with her mother for birth control counseling and exam, you detect signs of family tension. During the physical exam, while the mother is out of the room, the daughter admit to frequent marijuana use and occasional drinking. What assessment information would most confirm the presence of active or potential violence in the home? 1/1 signs of General neglect injuries at different stages of healing patience response to a direct question admitted fear of Mother's boyfriend

D Feedback although all answers May contribute to a family nurse practitioner suspicion of Family Violence, the admission of genuine fear of a household member is considered an excellent indicator of actual or potential violence in the level of danger in a home. The FNP should involve Social Services to assist the Adolescent female in this situation

a toddler is brought to the clinic with a history of an insect bite last evening. What presenting symptoms would be associated with the bite of a brown recluse spider? 0/1 paresthesias in all extremities edematous, erythematous areas with coalescing macules tissue sloughing in the Bay Area within 8 to 10 hours development of a central black eschar of "sinking infarct" within 12-24 hours

D Feedback brown recluse spiders produce sharp pain at the instant of the bite, with subsequent minor swelling and irritation immediately. Tissue necrosis may occur within the next 24 to 96 hours. A blue gray to Black macular Halo may surround the bite, with eventual widening and sinking of the center, leading to a sinking infarct. This leaves a deep ulcer that requires weeks or months to heal

what does the family nurse practitioner expect a preschool child with attention deficit hyperactivity disorder (ADHD) to have? 1/1 delayed growth and development, especially language skills negativism, over activity, and active curiosity diminished fine motor skills and frequent mood swings easy distractibility, impulsiveness, and fidgeting

D Feedback in a preschool child, it may be difficult to distinguish ADHD because problems of overactivity, inattention, and negativism are common. Problems with language skill development, along with fine motor skills, or more likely found with learning disabilities. Active curiosity and negativism are normal behaviors for preschoolers

The family nurse practitioner treats a two year old patient who has acute otitis media (AOM). The parents report a concern for speech delay. Review of the medical record reveals two additional AOM diagnosis in the past 6 months. The patient's plan of care includes: 1/1 a follow-up visit with the pediatrician tomorrow deter for ongoing ear infections a hearing evaluation in the primary care office for speech concerns a referral to the local school district for speech evaluation an ear nose and throat consultation for recurrent otitis media

D Feedback the American Academy of Pediatrics guidelines State you can offer PE tubes If the child has had three or more episodes of AOM in a 6-month.. This child has concern for speech delay as well which is another indication that he or she might be having hearing issues secondary to recurrent AOM or otitis with effusion

An advanced practice registered nurse (APRN) is treating a female patient with bacterial vaginitis. Which action should be included in the plan of care? Metronidazole 500 mg orally twice a day for seven days Clotrimazole 1% cream 5 g intravaginally for 7 to 14 days Return in one week if symptoms persist Treatment of the sexual partner is recommended within seven days

Metronidazole 500 mg orally twice a day for seven days

Types of Insulin Analogues Rapid acting

Duration: 3-5 h • Aspart (NovoLog) • Lispro (Humalog) • Glulisine (Apidra)

An advanced practice registered nurse (APRN) is seeing a patient with preeclampsia who has recently developed grand mal seizures. Which diagnosis should the APRN make for this presenting condition? Chronic hypertension with superimposed preeclampsia Gestational diabetes Eclampsia Preeclampsia with gestational diabetes

Eclampsia

a child is having a workup for rheumatic fever. His physical findings are temperature of 103.6 degrees Fahrenheit, migratory joint pain, and increased erythrocyte sedimentation rate (ESR). according to the Jones criteria, what is essential for the diagnosis of rheumatic fever? 1/1 history of group A B-streptococcal throat infection carditis Sydenham's chorea history of erythema marginatum for the last 3 days

Feedback According to the Jones Criteria, in addition to two major manifestations (carditis, polyarthritis, sydenham's chorea, erythema marginatum, and subcutaneous nodules) or one major and two minor manifestations (arthralgia, fever, elevated ESR, C-reactive protein, and prolonged PR interval), a diagnosis of rheumatic fever is highly likely if there is evidence of a previous group A B-hemolytic streptococcal infection

hich is an accurate statement about vegetarian diets during childhood? 1/1 a child who eats a lacto vegetarian diet is unlikely to consume adequate amounts of protein and vitamins for proper growth and development algae is a good source of bioavailable vitamin B12 for a child who eats a vegetarian diet bioavailable vitamin B12 is present in both animal and plant based foods vitamin B12 and zinc deficiencies place a child at risk for developmental delays

Feedback B12 and zinc ore most commonly found in meat and egg products. Sources of these nutrients in a vegan diet would include fortified soy milk, fortified we based meat substitutes, nutritional yeast, fortified cereals, almonds, brown rice, wheat germ, tofu, pecans, and spinach. Other deficiencies with vegan or variations of vegetarian diets can include vitamin D, riboflavin, calcium, and iron.

A 12 year old boy seen in the clinic, and a diagnosis of Hodgkin's disease is suspected . Which diagnostic test results confirm the diagnosis of Hodgkin's disease? 0/1 Elevated vanillylmandelic acid urinary level. The presence of blast cells in the bone marrow The presence of Epsetin-Barr virus in the blood. The presence of Reed-Sternberg cells in the lymph nodes Correct answer The presence of Reed-Sternberg cells in the lymph nodes

Feedback Hodgkin's disease is a neoplasm of lymphatic tissue. The presence of gaint multinucleated cells ( Reed- Sternbergs cells) is the hallmark of this disease. The presence of blast cells in the bone marrow indicates leukemia. The Epstein-Barr virus is associated with infectious mononucleosis . Elevated levels of vanillylmandelic acid in the urine may be found in children with neroblastoma.

a three-year-old child with up-to-date immunizations is brought to the office by his mother with a fairly rapid onset of stridor and a high-pitched weeds. In view of this information, what would be the least important conditions considered for this differential diagnosis? 1/1 croup foreign body aspiration epiglottitis bacterial tracheitis

Feedback If the child is up-to-date on immunizations, he should have had his vaccine for haemophilus influenzae type B or Hib, the most common cause of epiglottitis. Considering the age of this child in the suddenness of the onset, the most likely diagnosis is foreign body aspiration.

children with legg-calve perthes disease should: 1/1 maintain a diet high in protein, vitamins and minerals sleep on a firm mattress to prevent contractures avoid weight bearing on the affected extremity are you allowed to play basketball

Feedback Legg-Calve Perthes disease is a childhood hip disorder initiated by disruption of blood flow to the head of the femur. Due to the lack of blood flow, the bone dies (osteonecrosis or avascular necrosis) and stops growing. Over time, healing occurs by new blood vessels infiltrating the dead bone and removing the necrotic bone which leads to a loss of bone mass and a weakening of the femoral head. The bone loss leads to some degree of collapse and deformity of the femoral head and sometimes secondary changes to the shape of the hip socket. The goals of treatment or to decrease pain, reduce the loss of hip motion, and prevent or minimize permanent femoral head deformity so that the risk of developing a severe degenerative arthritis as an adult can be reduced. Diet and sleeping on a firm mattress do not alter the course of the disease. However, avoiding High Impact Sports such as basketball during treatment is essential is increase weight on the hip will cause further damage

David, age 15 months, is recovering from surgery to remove Wilms' tumor. Which findings best indicates that the child is free from pain? 1/1 decreased appetite increased heart rate decreased urine output increased interest in play

Feedback One of the most valuable clues to pain is a behavior change: A child who's pain-free likes to play. A child in pain is less likely to consume food or fluids. An increased heart rate may indicate increased pain; decreased urine output may signify dehydration."

an 18 month old child is brought to the clinic by her mother and is complaining of abrupt onset of vomiting, followed by more than 10 liquid stools with mucus for the last 48 hours. The temperature is 100° Fahrenheit orally. The stool smear obtained by the family nurse practitioner is negative for white blood cells. What is the most likely etiologic pathogen for this young child gastroenteritis? 1/1 rotavirus Shigella dysenteriae campylobacter jejuni salmonella

Feedback Rotavirus is the most frequent cause of gastroenteritis in children six months to two years of age. the Norwalk virus is more predominant in school age children. Up to 58% diarrhea in children results from viral infections, causing vomiting and then diarrhea. The stool smear is negative for white blood cells and viral causes in positive in bacterial causes. And viral diarrhea, the fever is mild, and diarrhea is watery and non bloody.

a 1 week old infant has a bilateral mucopurulent discharge. What explains the etiology of the discharge? 1/1 the mother probably has an STD the infant likely has a plugged tear duct this is bacterial conjunctivitis this is viral conjunctivitis

Feedback a bilateral mucopurulent eye discharge in a 1 week old infant is likely genitourinary in origin. The mother may have chlamydia. If the infant is delivered vaginally and the mother is infected with chlamydia, symptoms will appear one to two weeks Post delivery in the infant. The discharge is mucopurulent and would not be confused with the discharge associated with a plugged tear duct. Other STDs like gonorrhea could reduce symptoms two to four days post vaginal delivery

a seven year old has a complaint of ear pain. If he has otitis externa, which complaint is most likely? 1/1 he has tragal pain he has difficulty hearing the TV he has fever he has a concurrent upper respiratory infection

Feedback a patient with otitis externa has swimmer's ear, an infection of the external Canal. The classic complaint is tragal pain or even pinnae pain. if there is significant edema in the external Canal, hearing may be impaired, but the most common complaint is tragal pain. systemic complaints do not accompany swimmer's ear unless the second diagnosis is present simultaneously. Fever and upper respiratory infection or not likely

An adolescent Twisted his knee while skateboarding and comes to the clinic complaining of knee pain. He also stated in the past few weeks his knee has locked up a couple of times. On exam a positive McMurray test is elicited. What diagnosis is this consistent with? 1/1 anterior cruciate ligament tear dislocated patella medial meniscus tear chondromalacia patella

Feedback a positive McMurray's test (palpable click in pain when rotating the foot laterally and extending the leg) along with the symptoms is indicative of a medial meniscus tear. The drawer test evaluates for ACL tears (knee Flex with foot on table, sit on foot and grasp both sides of tibia at knee, pull tibia forward, abnormal if movement of tibia away from the joint).

a 6 month old child comes into the clinic for immunizations. Which item below allows a delay in getting immunizations today? 1/1 the child is on antibiotics the child has otitis media with a temperature of 103 the Mom is pregnant the child has a family member on chemotherapy

Feedback a six month old with acute otitis media and a temperature of 103 is considered moderately ill. Patients with moderate or severe illnesses, with or without fever, can be vaccinated as soon as they are recovering and not considered acutely ill. Antibiotics would not be considered a contraindication for any routine immunizations today. Pregnancy in the mother would not contraindicate any immunizations today. Live vaccines would be contraindicated if a family member on chemotherapy, however, a six month old will not be receiving any live immunizations.

according to the Tanner stages of development (sexual maturity rating) breast development in stage 3 in females includes the: 1/1 elevation of the breasts and nipple enlargement and elevation of breast in areola with no separation of their Contours projection of areola and nipple to form a secondary Mound above the level of the breast protection of the nipple only

Feedback according to the Tanner stages of development, breast development in stage 3 in females includes the enlargement and elevation of breast in areola with no separation of their Contours

Hirschsprung's Disease is characterized by: 1/1 intermittent constipation in the first year of life inability to absorb carbohydrates failure to pass meconium in the first 48 Hours of life chronic fecal incontinence

Feedback another name for this condition is aganglionic megacolon. the affected segment of colon is absence of ganglia that are responsible for producing peristalsis. The diagnosis cannot be made in the first 48 Hours of life, but a suggestion of a diagnosis can be made because meconium is not passed in a timely manner. Consequently, most infants are diagnosed in the neonatal period. patients usually present with emesis, abdominal distention, and failure to pass stool. And abdominal x-ray will demonstrate distal intestinal obstruction with dilated bowel Loops. The patient should be referred for surgery to remove the diseased segment of colon

a positive trendelenburg's test could be used to identify a child with 1/1 scoliosis Osgood-Schlatter disease nursemaids elbow slipped capital femoral epiphysis

Feedback asking a child with a complaint of hip pain to stand on the affected side is how the Trendelenburg test is assessed. A positive Trendelenburg test occurs when standing on the effective leg causes a pelvic tilt, such the unaffected hip is lower. This can be assessed and observed in children with slipped capital femoral epiphysis, legg-calve-perthes disease, or developmental dysplasia of the hip. Nursemaids elbow is a common ligamentous injury in young children. The Radial head becomes subluxed. displacement is usually easy to reduce. The Adams forward bend test is used to assess for scoliosis. Osgood-Schlatter disease is assessed by assessing for pain with palpation of the tibial tubercle

in young children, a complaint of hip pain without a history of trauma suggest several differential diagnosis. Which diagnosis is considered a true Orthopedic Emergency? 1/1 toxic synovitis of the hip Legg calve Perthes disease increased femoral anteversion avascular necrosis of the femoral head

Feedback avascular necrosis results in death of the femoral head with revascularization. Toxic synovitis and increased funeral anteversion, while causing pain, is not life threatening. Legg-calve-perthes disease results in necrosis of the proximal femoral epiphysis, however, there is later revascularization

how often should blood pressure be measured in a child who is 3 years old? 1/1 blood pressure measurement should begin at age 6 years, then every other year it should be measured every other year it should be measured annually it should be measured only at well-child visits

Feedback beginning at 3 years of age, blood pressure is measured annually. It continues to be measured annually through adulthood. Prior to Age 3 years, blood pressure is not routinely measured. Blood pressure is assessed in all four extremities at Birth to assess for coarctation of the aorta, Patent ductus arteriosus or PDA, other cardiovascular abnormalities, or significant murmurs. If blood pressure is only assessed at well-child visits for older children, it would not occur annually.

a two year old with sickle cell anemia should receive which immunization? 1/1 all routine childhood immunizations at an accelerated rate all routine childhood immunizations at a diesel rated rate all routine childhood immunizations at the usual time immunization should be limited in this group

Feedback children with sickle cell anemia should receive all the routine childhood immunizations at the usual time for administration, including annual flu immunization. Sickle Cell crisis arise when children become ill. Unfortunately, children with sickle cell anemia are more prone to Sickle Cell crisis when illness occurs, so, decreasing the likelihood of illness by immunization decreases the likelihood of sickle cell crisis

a six-year-old patient with a history of asthma has never used a peak flow meter. What quick tool can be used to assess the severity of the child's distress? 1/1 arterial blood gases child inability to complete a sentence chest x-ray presence of a runny nose

Feedback in children too young to use a peak flow meter properly, the inability to cry or complete a sentence May indicate an acute asthma attack. Obtaining an ABG is usually not performed in office settings, and a chest x-ray will most likely agitate the child and take too long to process

the sexual development of a 14 year old girl is notably delayed. She is very short, slightly obese, and complains of constipation and fatigue. Her skin is sallow and coarse in texture, and her hair is brittle. Which of the following tests should be obtained? 1/1 chromosome analysis and serum thyroxine serum thyroxine and thyroid stimulating hormone serum luteinizing hormone and chromosome analysis serum luteinizing hormone and thyroid stimulating hormone

Feedback classic symptoms of growth failure, goiter, delayed or arrested puberty, delayed dentition, weight gain, fatigue, dry skin, hyperlipidemia, decline in school performance, mennorhagia, and fatigue. It would be most appropriate to test for hypothyroidism first in this patient. If these are normal then you may want to consider the other test for reasons for her growth delay etc.

cryptorchidism it's most commonly found in the: 1/1 infant with history of 25 week prematurity infant with a history of 43 week post Post maturity term infant toddler who is potty training

Feedback cryptorchidism is the result of undescended testes, either bilateral, or more often, affecting the right testis. Normally, descent is in the 7th to 8th month of gestation, therefore it more often affects premature infants.

which of the following increases the risk of Cryptorchidism? 1/1 family history of hearing problems premature birth maternal iron deficiency anemia constipation

Feedback cryptorchidism refers to a testis that is not within the scrotum and has not descended by 4 months of age. the undescended testis maybe in the abdomen, the priests crotalaria, or maybe absent. If testis does not descend, it may be due to a short spermatic artery or poor blood supply. An absent testis may be due to a agenesis or atrophy. During normal prenatal development, the testes develop in the abdominal cavity and descend through the girl went issue, forming a scrotal Sac. The incidents and premature birth is about 30% compared to full-term male infants, and whom the right is about 3%

A "sweat test" or newborn screening may be used to detect; 1/1 cystic fibrosis adrenal insufficiency Graves disease hypothyroidism

Feedback cystic fibrosis is the most common inherited fatal disease of children and young adults in the United States. Cystic fibrosis is usually diagnosed by the time and affect the child is 3 years old. Often, the only signs are persistent cough, a large appetite but poor weight gain, and extremely salty taste to the skin, and large, foul-smelling bowel movements. A simple sweat test is currently the standard diagnostic test. The test measures the amount of salt in the sweat and abnormally high levels are the Hallmark of this disorder

which of the following signs and symptoms are associated with a diagnosis of childhood acute lymphocytic leukemia (ALL) ? 0/1 splenomegaly, facial rash and cough expiratory wheezing, bleeding and hepatomegaly bleeding, fever and pain bone pain, fever and night sweats Correct answer bleeding, fever and pain .

Feedback early presenting symptoms of ALL include pallor, bleeding, pain, and fever associated with infiltration of the bone marrow of proliferative lymphocytic cells, which ultimately leads to bone marrow failure. The symptoms splenomegaly, facial rash, cough, expert or wheezing, bleeding. bone pain, fever, and not sweats are often noted in CML

the family nurse practitioner understands that growth retardation that appears after age 12 in boys is usually caused by: 1/1 chromosomal abnormalities hyperthyroidism hyperpituitarism hypogonadism

Feedback hypogonadism, associated with delayed sexual development, sexual infantilism, and small testes, is most often the contributing factor to growth retardation after age 10 years for girls and 12 years for boys. Other causes of deceleration in growth or short stature include hypothyroidism, diabetes, and hypopituitarism. Chromosomal abnormalities would be noted at an earlier age.

a mother brings her preschool child to see the family nurse practitioner because of sores on his arms and legs. On exam, the family nurse practitioner notes several honey-colored crusted lesions within a erythematous base on the arms and legs. There's a history of exposure to mosquitoes. The rest of the exam is essentially negative. What is the most likely diagnosis? 1/1 scabies impetigo pityriasis rosea varicella

Feedback impetigo presents with honey-colored crusted lesions with a erythematous base. Staphylococci and group a streptococci are important pathogens in this disease. Scabies presents with linear Burrows about the wrists, ankles, finger webs, anterior axillary fold, genitalia, or face (in infants).pityriasis rosea present with erythematous papules that coalesce to form oval plaques preceded by a large oval platter with Central clearing and a scaly border (the herald patch). varicella or chickenpox presents as crop of red macules that rapidly become tiny vesicles with surrounding erythema that forms pustules. The pustules become crusted, and then scabs form. The rash appears predominantly on the trunk and face.

what test is ordered for all newborns to screen for Cystic Fibrosis? 1/1 serum amylase immunoreactive trypsinogen (IRT) sweat chloride test DNA analysis

Feedback in all states cystic fibrosis newborn screening starts with evaluating for an elevated serum IRT, which is biomarker for trypsinogen. When there is blockage of the pancreatic exocrine Ducts, which occurs in cystic fibrosis, this prevents the release of trypsinogen in the small intestine. The increase circulating trypsinogen is because of blockage caused by decreased cystic fibrosis transmembrane conductance regulator activity, which is a protein that acts on the chloride Channel. If the infants serum IRT is elevated, a second step is required, which is DNA analysis looking for common mutations. If one or mutations are found, the infant is referred to cystic fibrosis Foundation accredited Care Center for sweat chloride confirmation testing. The sweat chloride test has been the primary way to confirm a diagnosis of cystic fibrosis. a sweat chloride test measures the amount of chloride in the infant's sweat. A chloride value of 60 mmol per liter or greater has been considered positive for a diagnosis of CF.

an 8 year old has a painful lip. He reports that his knee hurts medially. On exam he has pain with internal rotation of the hip. How should the nurse practitioner manage this situation? 1/1 he should be immediately referred to Orthopedics the nurse practitioner should order a hip X-ray, complete blood count, and ESR the nurse practitioner should order a hip and knee x-ray he should be referred for synovial fluid aspiration

Feedback in an eight-year-old, there are several diagnosis in the differential. One must consider legg-calve Perthes, transient synovitis of the hip, a slipped capital epiphysis (SCFE), and a septic hip. this could be as benign as transient synovitis that does not require a referral. All of the others mentioned would need urgent Orthopedic referral. Once the diagnostics were completed, the nurse practitioner would have a better idea about whether Orthopedic referral was essential

the most common cause of pneumonia in an otherwise healthy three-year-old child is: 1/1 S pneumonia s aureus mycoplasma a viral infection

Feedback in children who are six months to five years of age, the most common cause of pneumonia is a viral pathogen.rarely are studies performed to identify viral pathogens, however one of the most common viral pathogens is respiratory syncytial virus or RSV. S. pneumoniae is a common cause of pneumonia in very young children, it is also implicated in older adults is the causative agent in pneumonia. And young and middle adults, mycoplasma is a common pathogen

which infant feeding behavior is least likely related to congenital heart disease (CHD)? 1/1 limited intake of volume of milk beatings that take a long time to complete feedings that are interrupted by sleeping infants that burp frequently when feeding

Feedback infants who burp frequently probably are swallowing too much air with feeding. This is likely not related to congenital heart disease. Some red flags associated with feeding that I should probably examiner to assess for congenital heart disease include feedings interrupted by choking, gagging, or vomiting. Some infants have rapid breathing with feeding or persistent cough or wheeze. They should be assessed and congenital heart disease should be considered

which patient below is most likely to experience stranger anxiety during a physical exam? 0/1 a 6 month old male a 12 month old female a 3 year old male a 4 year old female Correct answer a 12 month old female

Feedback infants younger than 6 months and children older than three years typically did not exhibit stranger anxiety when the examiner enters the room. there is no known difference between stranger anxiety and male and female children. The most specific time for stranger anxiety to develop is about nine months old.

what is the usual age for vision screening in young children? 1/1 2 years 3 years 4 years 5 years

Feedback initial vision screening should take place at 3 years of age. If the child is not Cooperative, screening should be attempted six months later. If the child is still not cooperative at three and a half years, it should be attempted it for years. Generally, children of Cooperative at four years of age. The usual vision of a 3-year old is 20 / 50

during a clinic visit, the family nurse practitioner notes in an 11 month old infant is pale. The physical exam reveals a pulse of 170 beats per minute, height at the 25th percentile, and weigh at the 95th percentile. The nurse questions the mother about the infant's diet. The mother states that the infant eat mostly pureed fruit and whole milk. Which diagnostic finding does the family nurse practitioner expect? 1/1 normal hemoglobin elevated mean corpuscular volume (MCV) low serum ferritin level microcytic, hypochromic anemia

Feedback iron deficiency anemia is often found in this age group, especially in infants who do not eat a balanced diet that includes foods rich in iron such as iron fortified cereals Etc. This is a microcytic, hypochromic anemia with a decrease in serum iron or ferritin. MCV would be decreased.

a four-year-old has been diagnosed with measles. The nurse practitioner identifies koplik's spots. These are: 1/1 spots on the skin that are pathognomonic for measles red rings found on the time that have a white granular area inside the ring found on the inside of the cheek and are granular blanchable areas on the trunk and extremities

Feedback koplik's spots are found in the oral cavity, especially on the buccal mucosa opposite the first and second molars. The spots are white and granular and are circled by erythematous ring. the spots are pathognomonic for measles. the exanthem associated with measles is typically described as cranial to caudal to progression. deletions become confluent and last approximately four days before fading begins

18 year old boys experiencing problems with wheezing, coughing, and shortness of breath for about 4 hours after basketball practice. He has normal respirations and experiences problems only after exercise. He is experiencing no other respiratory problems, and the physical findings are within normal limits. What is the treatment of choice for this child? 1/1 albuterol two Puffs metered dose inhaler 20 to 30 minutes before exercise cromolyn sodium two Puffs each morning Theophylline 100 mg PO bid beclomethasone two Puffs three to four times daily

Feedback medication should be taken only when the child is planning to exercise and anticipates respiratory difficulty. Cromolyn and beclomethasone or steroids and do not provide immediate relief. The other one should be avoided unless symptoms progressively worsen and cannot be controlled with inhalation therapy

heart failure is a common clinical presentation occurring in a child with congenital heart disease (CHD). what is another common clinical presentation with CHD? 1/1 hypoglycemia hypertension peripheral edema cyanosis

Feedback most cases of heart failure in children result from congenital heart disease, and most occur during the first year of Life. Although the clinical presentation of a child with HF will vary with the specific defect, the clinical manifestations usually relate to the degree of heart failure or cyanosis

which of the following is the most accurate statement about juvenile rheumatoid arthritis (JRA)? 1/1 symptoms present in much greater severity than an adult RA complete remission occurs in 3/4 of patients more than 90% of cases progressed to severe joint destruction cytotoxic drug should be initiated as early as possible in the treatment regimen

Feedback most do not have disease persistent into adulthood. JRA symptoms present very similar to adult arthritis, most disease activity diminishes with age. Although some do have residual joint damage, the percentage is not this High. Nonsteroidal anti-inflammatory drugs (previously aspirin) are the treatment of choice, cytotoxic drugs are reserved for patients in whom other therapies have failed.

on exam the family nurse practitioner notes that a 2 week old infants left eye is watering and crusted material is on the eyelids. No edema or erythema is noted. What is the most likely diagnosis? 1/1 nasolacrimal duct obstruction conjunctivitis congenital dacrocystocele corneal abrasion

Feedback nasolacrimal duct obstruction occurs in up to 6% of infants. Signs and symptoms include a wet eye and mucoid discharge. Irritated skin and conjunctivitis Also may be associated with this condition. There is no redness, which would suggest conjunctivitis. Congenital dacrocystocele Presents at Birth as a bluish subcutaneous Mass. There would be more signs of irritation and pain with a corneal abrasion.

a toddler has ingested some of his grandfather's pills. The toddler is vomiting, feels weak, and has a first degree atrioventricular (AV) block pattern on the electrocardiogram. The grandfather brings in for medication bottles. Which medication did the toddler most likely ingest? 1/1 amitriptyline digoxin Furosemide aspirin

Feedback nausea, vomiting, and anorexia are common side effects of many medications. The first degree AV block confirms the ingestion of digoxin in this situation. Tricyclic antidepressants toxicity is characterized by agitation and anticholinergic symptoms. Furosemide toxicity is characterized by hypokalemia, weakness, and cardiac dysrhythmias. And aspirin toxicity is characterized by tinnitus, confusion, gastrointestinal symptoms, and Rapid, deep respirations.

a four-year-old child with otitis media with effusion: 1/1 needs an antibiotic probably has a viral infection probably has just had acute otitis media has cloudy fluid in the middle ear

Feedback otitis media with effusion (OME) frequently precedes or follows an episode of acute otitis media. This condition should not be treated with an antibiotic since the middle ear fluid is not infected. However, the fluid acts as a medium for bacterial growth. This fluid may be present for months after an episode of acute otitis media

a six-year-old child is seen by the family nurse practitioner for ear pain. The child is afebrile. The left ear canal is extremely edematous and moderately inflamed, with thick yellowish drainage at the external meatus. The child denies putting anything in the ear canal, but the FNP finds the child swims frequently. What is the most likely diagnosis? 1/1 acute otitis media serous otitis media sinusitis Otis externa

Feedback patients with Otis externa or swimmer's ear, or inflammation of the external auditory canal presents with ear pain. The most common clinical findings include redness and swelling of the external ear canal, pain with manipulation of movement of the auricle, no swelling, or pain over mastoid, and usually no involvement at the tympanic membrane. The tympanic membrane is involved in acute otitis media. No evidence indicated that the child placed a foreign body in the ear. The most significant distinction between OME (otitis media with Effusion) and AOM is that clinical findings of acute infection such as fever and otalgia are lacking in OME. Clinical findings of OME, the most common cause of hearing loss in children include a relatively asymptomatic, decreased mobility, and bulging, opaque tympanic membrane with no visible landmarks

pharmacologic treatment for children who have hypertension should be initiated for: 1/1 those who are obese stage 1 hypertension diabetics with hypertension asymptomatic stage 1 or stage 2 hypertension

Feedback pharmacologic treatment should be initiated for children who have both hypertension and diabetes, symptomatic hypertension, hypertension above the 95th percentile, when in Oregon damage is present. Obesity is a risk factor but it's not a soul indicator of treatment

a pregnant mother in her first trimester has a five-year-old who has fifth disease. What implications does this have for the mother? 1/1 she does not have to worry about transmission to the fetus she may get a mild case of Fifths Disease there is a risk of fetal death if she becomes infected the mother should have a fetal ultrasound today

Feedback pregnant mother should avoid exposure to patients with known fifth disease. However, the risk of transmission is very low. She should avoid exposure to aplastic patients who are infected because they are highly contagious. Infection during pregnancy is associated with a 10% risk of fetal death. There is no need for an ultrasound today. This pregnant patient does not have evidence of the disease. She should be monitored for a rash which could indicate infection

infective endocarditis prophylaxis may be required for children with congenital heart defects in which of the following procedures? 1/1 dental procedures such a simple adjustment of orthodontic appliances cardiac catheterization tonsillectomy and/or adenoidectomy insertion of tympanostomy tubes

Feedback procedures for which endocarditis prophylaxis is recommended include dental procedures known to induce gingival bleeding. It is also recommended for surgical procedures that involve the respiratory mucosa such as tonsillectomy and adenoidectomy. Endocarditis prophylaxis is not recommended for insertion of tympanostomy tubes, cardiac catheterization, Simple dental procedures, or endotracheal intubation. Cardiac catheterizations are done under sterile conditions, and prophylactic treatment is not recommended because of a very low incidence of infection. Endocarditis prophylaxis is for patients with prosthetic valves, prosthetic Rings or chords used in cardiac repair, previous infective endocarditis, unrepaired critical congenital heart disease or repaired congenital heart disease with shunts, valvular regurgitation or prosthetic devices, and cardiac transplant with valve regurgitation.

which of the following findings with the family nurse practitioner expect to find in a child with pubertal gynecomastia? 1/1 Tanner stage two with testes less than or equal to 4 cm in length breasts and nipples non-tender in equal in size breast tissue enlargement mainly glandular, movable and non-adherent to skin or underlying tissue. lymphadenopathy, goiter, asymmetric testes, and repaired hypospadias

Feedback pubertal or physiologic gynecomastia is a visible or palpable glandular enlargement of the male breast that can occur in healthy adolescents. Typically, the breasts are unequal in size and may be Tender, nipples are often irritated from rubbing against clothing, and Tanner stage is 2 through 4 of pubertal development are noted. The symptoms of Lymphadenopathy, goiter, asymmetric testes, and repaired hypospadias are associated with pathologic gynecomastia.

the family nurse practitioner is counseling a parent who has a child with sickle cell disease. The parent asks "if my child has sickle cell disease, does that mean that I'm at an increased risk for developing the same problems?" the family nurse practitioners response will be based on Which principle of sickle cell disease? 1/1 the mother is at an increased risk because the condition is inherited and she probably has the condition and has not had an active episode the mother is not at risk for developing the condition because males are carriers of this trait the mother has a 25% chance of being affected by the disease, especially during times of stress both parents are either carriers of the tray or half a disease, and each child has a 25% chance of having the condition

Feedback sickle cell disease is an autosomal recessive inherited disease transmitted by both parents, who are carriers of the sickle cell trait or have the disease. Each pregnancy carries a 25% chance of sickle cell disease, a 25% chance of an unaffected child, and a 50% chance of the child carrying the trait. Depending on the country and state in which the parents were born and the parents age, a hemoglobin electrophoresis would diagnose sickle cell disease and or the trait on a newborn screening

an infant is brought to the nurse practitioner because his gaze is asymmetrical. Which finding indicates a need for a referral to ophthalmology? 1/1 he is two months of age he is 3 months of age he has persistent strabismus his red reflex is normal

Feedback strabismus maybe completely normal in the very first few months of Life. Persistent strabismus at any age likely indicate a muscle weakness, cranial nerve abnormalities or a number of other pediatric eye diseases. The infant with a normal red reflex probably does not have retinoblastoma are congenital cataracts, but both of these conditions can result in strabismus.the infant should be referred to an ophthalmologist for evaluation if he has persistent strabismus

the family nurse practitioner understands that the blood glucose level in diabetic children 7 to 12 years of age who can recognize the symptoms of hypoglycemia should Target which range before meals? 1/1 60-75 mg / DL 100-175 mg / DL 90 - 130 mg / DL greater than 180 mg / DL

Feedback the American Diabetes Association recommends most children have a premier target range for blood glucose between 90 to 130 mg / DL if they can recognize the symptoms of hypoglycemia. A blood glucose of 60-75 mg / DL is too low and may predispose the child to hyperglycemia. The other options are too high.

what Tanner stage corresponds to an average 8 year old male? 1/1 stage 0 stage 1 stage 2 stage 3

Feedback the Tanner scale indicates the stages of sexual development of males and females. The development of sexual characteristics is described based on the stage. An average 8 year old would be expected to be pre-pubertal. This is characterized as Tanner stage 1. Pubertal changes can occur as early as 9 to 10 years in males or females. If changes occur prior to this, precocious puberty should be considered.

a young child has an audible murmur. The nurse practitioner describes it as a grade 4 murmur. How should this be managed? 1/1 the child should be monitored for other abnormal findings child should be referred to Cardiology and EKGs should be performed the nurse practitioner should palpate to see if a thrill is present

Feedback the child should be referred to pediatric cardiology. A grade four designation for a murmur indicates that a murmur is loud and has a thrill associated with it. The finding Of A Thrill which is characteristic of a grade four murmur is always an abnormal finding and requires referral. the thrill can be palpated with the examiner's hand over the anterior chest in the area of the point of maximal impulse or PMI

a three-year-old child is seen in clinic for chronic, relapsing diarrhea. A stool for OVA and parasites is obtained and it's positive for giardia. What is the most appropriate pharmacologic intervention? 1/1 ampicillin (Omnipen) erythromycin (E-mycin) metronidazole (Flagyl) tetracycline (Achromycin)

Feedback the drug of choice for treating Giardia is metronidazole 500 mg per kilogram up to 250 mg tid for 5 days. this treatment is 80 to 95% effective. this drug is well tolerated and children. It has a disulfram-like effect and should not be used in children or adolescents receiving ethanol containing medications. All other drugs are ineffective for giardia. Tetracycline should not be prescribed to children under age 8 for any reason.

on physical exam of a 14 year old girl complaining of amenorrhea, the family nurse practitioner notes blood pressure 138/90, pulse of 98, broad chest with widely spaced nipples, Tanner stage one, webbing of the neck, low hairline, and prominent anomalous ears. What does the family nurse practitioner suspect? 1/1 Klinefelter syndrome Marfan syndrome fragile X syndrome Turner syndrome

Feedback the following findings are consistent with turner syndrome: short stature, gonadal dysgenesis, lymphadema (usually appearing in infancy)left-sided heart or aortic abnormalities. Primary amenorrhea, and delayed onset of puberty. Fragile X syndrome is an inherited condition usually affecting males and characterized by a long, narrow face and prominent ears, mild to profound mental retardation, hyperactivity and poor attention span, and autistic type of behavior. Marfan's syndrome is a connective tissue disorder of tall and thin adolescents that is characterized by long limbs, narrow hands, long, slender fingers, and nearsightedness. Klinefelter syndrome is characterized by small testes, sterility, gynecomastia, and long legs.

a mother brings her school-age child in for an exam. She reports that the child frequently scratches and that she seems to itch worse at night. On exam, the family nurse practitioner notes lesions on the sides of the fingers and inner aspects of the elbows. These lesions are short, your regular and run approximately 2-3 mm long and the width of a hair. The FNP tells the mother she suspects: 1/1 scabies hives fleas ticks

Feedback the location and appearance of the lesions are typical of Scabies. the rash causes little bumps that often form a line that has approximately two to three mm long in the width of a hair. The inflammatory lesions are a erythemotous and pruritic papules most commonly located in the finger web, Flexor Surfaces of the wrist, elbows, axillae, buttocks, genitalia, feet, and ankles. The older adult might itch more severely with fewer cutaneous lesions and is at risk for extensive infestations, probably related to a decline in cell mediated immunity. In addition, there may be back involvement in those who are bedridden.

what is the major symptom of reflux in infants? 1/1 vomiting or regurgitation, especially after feeding poor weight gain hyper irritability and refusal of feeding fever and diarrhea

Feedback the major symptom of reflux is vomiting or regurgitation. It may occur during sleep, and frequently after feeding. Poor weight gain, hyper irritability, and refuse your feeding baby songs in some infants but are not the major symptoms. Fever or diarrhea may be present in patients with acute otitis media, gastroenteritis, or urinary tract infections

the family nurse practitioner knows that an infant who is exclusively breastfed is at risk for developing iron deficiency anemia after what age? 1/1 one month 2 months 4 months 6 months

Feedback the normal, full-term infant is born with sufficient iron stores to prevent iron deficiency for the first 6 months of life.

a four year old is being examined today by the nurse practitioner in the clinic. He appears shy and does not make eye contact with the examiner. The mother does not make eye contact with the examiner either. The patient lacks animation and does not smile. What likely possibility must be considered? 1/1 the patient is depressed sexual abuse is very likely neglect is possible a child probably does not have medical insurance

Feedback the possibility of neglect should always be given when a young patient and caregiver make poor eye contact with the examiner. Other Clues to neglect or a lack of animation and no social smile. Additionally, the child should be observed for nutritional status, behavior, attitude, and physical appearance, sexual abuse should also be considered, but there is nothing in the stem of this question to suggest this or a lack of medical insurance

the nurse practitioner sees a child who reports fatigue and presents with purpura on his lower extremities. His temperature is normal. The differential includes: 1/1 anemia abuse acute leukemia kawasaki syndrome

Feedback the presence of purpura must make the nurse practitioner consider platelet problems. Fatigue should prompt consideration of anemia. Both are seen in patients with leukemia. Acute leukemia must be considered because of a combination of fatigue and purpura. Kawasaki disease is an autoimmune disease that produces vasculitis of the midsize arteries. henoch-schonlein purpura could also be considered as part of the nurse practitioners differential, but this was not a choice here

the family nurse practitioner is interpreting the notation of "string sign" on an upper gastrointestinal series performed on an infant. What diagnosis is this associated with? 1/1 intussusception Hirschsprung's disease pyloric stenosis gastroesophageal reflux

Feedback the string sign is indicative of a narrow pyloric Channel and its associated with pyloric stenosis. Intussusception would be evaluated by a barium enema. hirschsprung's disease or congenital aganglionic megacolon, can be diagnosed with a Wagensteen-Rice series (air rises in the inflated colon). gastroesophageal reflux is usually diagnosed by clinical findings and infants. Barium swallow will reveal free regurgitation of barium from stomach to esophagus. And upper GI series would be performed to rule out causes of vomiting

cystic fibrosis (CF) is the preliminary diagnosis for young girl he was brought to the clinic for evaluation. What is the test used to rule out CF? 1/1 hemocult test sweat chloride test sputum culture and sensitivity glucose tolerance test

Feedback the sweat chloride test is positive for Cystic Fibrosis because of the abnormal amount of sodium chloride in the sweat. Hemoccult is a test for blood in the stool. Sputum culture and sensitivity help determine which medication is effective against an organism. Glucose tolerance test is performed to diagnose diabetes. Genetic testing can be used. There are more than 1,700 mutation of the CFTR gene, with likely more gene mutations that have not yet been discovered. chloride sweat test Remains the gold standard

which of the following would be pertinent in the past medical history of a child who is being evaluated for cardiovascular disease? 1/1 Kawasaki disease hypothyroidism osteogenic sarcoma Tourette syndrome

Feedback the two major conditions known to play a causative role in the development of cardiovascular disease in children are untreated streptococcal infections including group A B-hemolytic streptococci (leads to cardiac valve dysfunction) and kawasaki disease (leads to coronary artery aneurysm)

a 3 day-old full-term infant has a bilirubin level of 16 mg / DL. How should this be managed? 1/1 monitor only increase fluids and stop breastfeeding increase breastfeeding order phototherapy for the infant

Feedback there are several ways to determine how elevated bilirubin level is. One measure is to use the bhutani nomogram.it predicts bilirubin levels based on postnatal age. A level of 16 is considered high intermediate. Since the bilirubin probably will rise a little more, phototherapy is probably appropriate

a mother brings her six-year-old daughter to the nurse practitioner for evaluation of breast and axillary hair growth. The patient has grown 1 inch during the last three months. The nurse practitioners action is to: 1/1 follow the Child Development every 6 months inform the mother that this condition is normal initiate therapy with Ketoconazole refer the patient to a pediatric endocrinologist

Feedback this child has evidence of precocious puberty in a child less than eight years of age. Isolated axillary hair or single symptoms can be monitored. when the child has two or more symptoms they should be referred to Endocrinology for workup of precocious puberty. The Ketoconazole is an antifungal medication and will not improve symptoms and this is not a normal finding

a four year old female is brought into the clinic by her mother, who reports that she is constantly scratching "her private part". the patient states that itches. An exam, the vagina is red and irritated. How should the NP proceed? 1/1 call child protection for suspected sexual abuse prescribed a cortisone cream collected vaginal swab of the external vagina for microscopic evaluation prescribed a topical antifungal

Feedback this child has vaginitis. There are many diagnosis in the differential, including pinworms, yeast, contact irritants from soap or bubble bath, Etc. Since the diagnosis is not clear, some evaluation must occur in order to determine the diagnosis so proper treatment can be initiated. Since the description of the problem does not indicate what the diagnosis is, it is inappropriate to treat with cortisone cream or topical antifungal.

a three year old female had a fever of 102 for the past 3 days. Today she woke up from a nap and is afebrile. She has a maculopapular rash. Which statement is true? 1/1 this child probably has measles the rash will Blanch this is a streptococcal rash this could be Kawasaki disease

Feedback this describes a patient with roseola or exanthem Subitum. this is a common viral exanthem found in young children caused by human herpesvirus 6. It is characterized by high fever for 3 Days followed by the abrupt cessation of fever and the appearance of a macular papular rash. it usually resolves in a few days. The child May return to school or daycare when he has been fever free for 24 hours

A mother presents with her one month old infant. She reports that he cries inconsolably every evening after his first evening feeding. She asked for help. What should be done? 1/1 order stool specimens order ranitidine for the infant's bottle try a different formula provide education, parental reassurance, and encouragement

Feedback this describes colic. Colic is a symptom complex characterized by episodes of inconsolable crying accompanied by apparent abdominal pain. It typically occurs between 1 and 3 months of age and usually in a very predictable pattern, typically in the evening after feeding. Many different approaches are tried, but medications like Ranitidine is not indicated nor is changing formulas. Parents need education regarding colic, Comfort measures like rhythmic rocking or frequent burping, much reassurance, and encouragement

a 15 year old male patient presents with complaints of severe scrotal pain for the last 2 hours. The scrotum is swollen and extremely tender, palpation of the epididymis it's not possible. What does the family nurse practitioner recognized as the immediate treatment? 1/1 narcotic analgesics and bed rest warm packs and scrotal support antibiotics, ice packs, and analgesics referral to a surgeon for exploration

Feedback this involves the differential diagnosis between epididymitis and testicular torsion. Irreversible damage will be done to the testicles if torsion is not released in three to four hours. Time should not be wasted with other treatments if torsion is strongly suspected.

a 6 week old male infant is brought to the nurse practitioner because of vomiting. The mother describes vomiting after feeding and feeling a knot in his abdomen especially after he vomits. The child appears adequately nourished. What is the likely etiology? 1/1 gastroesophageal reflux (GERD) pyloric stenosis constipation Munchausen Syndrome by proxy

Feedback this scenario is typical of an infant with pyloric stenosis. It is more common in males at 88% and usually is diagnosed before the child is 12 weeks old. The classic presentation is an infant who vomits immediately after eating. The knot in the abdomen is the typical Olive shaped Mass palpable at the lateral edge of the rectus abdominis muscle in the right upper quadrant of the abdomen. The mass is best palpated immediately after vomiting. The differential should include gastroesophageal reflux, but no mass is palpable. Munchausen Syndrome by proxy is the fabrication or induction of an illness in a child in order for the caregiver to receive attention.

the nurse practitioner observes multiple oval macules, both hyperpigmented and hypopigmented, on a child's upper trunk. A fine scale around the borders of the lesions. The diagnosis is: 0/1 atopic dermatitis keratosis pilaris pityriasis rosea tinea versicolor

Feedback tinea versicolor is a common dermatosis characterized by multiple small, oval, scaly patches measuring 1-3 cm in diameter, usually located in a raindrop pattern on the upper chest, back, and proximal portions of the upper extremities. In children, atopic dermatitis lesions are typically dry, papular, and intensely pruritic. Circumcised scaly patches are distributed on the wrists, ankles, and antecubital and popliteal fossa. Keratosis pilaris is typically distributed on the posterior upper portion of the arms and lesions are dry and papular.With Pityriasis rosea, a prodrome of malaise, headache, and mild constitutional symptoms occasionally precedes the rash but it's not crucial for diagnosis. The typical eruption begins with the appearance of one or multiple "Herald patches" which are large, isolated, oval lesions, usually pink in color and slightly scaly. They may occur anywhere on the body.From 5 to 10 days later, other smaller oval lesions appear on the body, frequently concentrated over the truck but also seen on the proximal extremities, especially the thighs

which of the following is an appropriate initial intervention for gastroesophageal reflux disease (GERD) in an eight week old? 1/1 small, frequent thickened feedings cimetidine every 6 hours change formula to soy-based place infant on left side after eating

Feedback two strategies should be tried and initially. first, avoidance of overfeeding is recommended. Hence, small, frequent feedings. second, milk thickening agents appear to improve symptoms in infants who experience gastroesophageal reflux disease. Thickened feedings significantly decreased frequency of reflux in most infants. Also, caloric content is increased in this may be helpful for patients who are underweight because of persistent gastro esophageal reflux disease. Generally, when medications are used, proton pump inhibitors are preferred over H2 blockers like cimetidine. Changing formula generally does not help, however, a milk-free diet may help since 40% of infants with gastroesophageal reflux disease are sensitive to cow's milk protein. Lateral positioning seems to be an effective in relieving symptoms in infants

what would be age appropriate anticipatory guidance for the parents of a nine month old infant? 1/1 keep syrup of ipecac in case of accidental poisoning stranger anxiety will develop by a year of age your baby should be able to say 10 words before age 1 year discuss weaning from a bottle

Feedback weaning from the baby's bottle takes place at about one year of age. This should be discussed at the visit prior to the time this will take place. Nine months is an appropriate time to discuss this. Syrup of ipecac is no longer recommended. Parents and caregivers should be instructed not to use this. Stranger anxiety develops at about nine months of age. The average 12 month old says about 2-5 words

the family nurse practitioner is concerned about the development of which complication in a young child with a diagnosis of iron deficiency anemia? 1/1 Crohn's disease pernicious anemia impaired cognitive development hepatic and spleen dysfunction

Feedback with iron deficiency anemia, the amount of oxygen carrying hemoglobin is reduced. Long-term oxygen deprivation can lead to impaired cognitive and motor development. Pernicious anemia is associated with lack of intrinsic factor. Crohn's disease has a familial incidence and leads to diarrhea related problems. The liver and spleen are both involved in red blood cell production

An advanced practice registered nurse (APRN) is seeing a patient who states they are transgender and are having serious issues related to stress at work. The APRN decides to refer the patient to a psychologist and needs to determine the correct medical diagnostic terminology recommended by the Diagnostic and Statistical Manual of Mental Disorders (5th edition) to use during the process. Which medical diagnostic terminology should be used for this patient? Gender identity disorder Gender dysphoria Gender role conformity Gender expression

Gender dysphoria

A female patient presents to the primary care clinic complaining of lower abdominal pain, abnormal menses, and difficulty urinating. The advanced practice nurse (APRN) performed a pelvic examination that revealed cervical motion tenderness (CMT) with purulent discharge. A diagnosis of gonorrhea is confirmed. The APRN prescribes ceftriaxone 250 mg IM; however, the medication is unavailable. Which alternative treatment regimen should the APRN use for this patient? Amoxicillin 3 g plus probenecid 500 mg orally Metronidazole 2 g orally once Gentamicin 240 mg IM plus azithromycin 2 g orally once Erythromycin 500 mg orally qid

Gentamicin 240 mg IM plus azithromycin 2 g orally once

An advanced practice registered nurse (APRN) notices a six-month-old infant's weight has doubled since birth, and the child cries when picked up. The infant's head lags when the child is pulled to a sitting position and the rooting reflex is not present. Which developmental issue should be of concern to the APRN? Birth weight has doubled Cries when picked up Rooting reflex is not present Head lags when pulled to sitting

Head lags when pulled to sitting

Lumbar lordosis, or hyperlordosis,

If the lordosis persists in the forward bending position, this indicates a fixed structural deformity and needs referral to an orthopedist.

An advanced practice registered nurse (APRN) is caring for a patient who is transgender and has been hospitalized for cancer treatment. The patient is using Medicaid coverage. The APRN asked the patient to complete paperwork to identify a medical decision-maker, and a best friend who is also transgender was selected. The patient mentions being concerned about the choice since the chosen person lives an alternative lifestyle and is not a family member. Which piece of information regarding this choice should the APRN share with this patient? The court will need to appoint an advocate to work with any non-family member who is selected to be a medical decision-maker. The patient will need to marry the friend, and then the new spouse will be able to be named the medical decision-maker. Medicaid regulations allow patients to choose their own medical decision-makers regardless of their legal relationship to the patient. Medicaid will not allow non-family members to be the medical decision-maker, so the patient should apply for Medicare instead.

Medicaid regulations allow patients to choose their own medical decision-makers regardless of their legal relationship to the patient.

A lactating mother presents to your office with acute bronchitis. She has been up all night coughing and is exhausted. She is requesting codeine cough syrup that has helped her in the past. What is an appropriate response to the mother's request? Prescribe benzonatate instead. Explain that she can take codeine, but she must "dump" milk for at least four hours. Provide an alternative prescription for dextromethorphan and explain that codeine is not safe for lactating mothers.

Prescribe the codeine as requested. Correct! Codeine is metabolized into morphine in the liver, and in rapid metabolizers unsafe levels can cross into the breastmilk causing CNS depression in infants.

An advanced practice registered nurse (APRN) is examining a two-month-old and notes a well developed infant who has the ability to turn the head when laid on the stomach and a positive rooting reflex. The parent is concerned because the infant has been very fussy for the last week and will not quiet unless breastfed, even though it is much earlier than the typical feeding schedule. A second dose of the HepB vaccine is due during the current visit. Which action should the APRN take for this patient? Encourage the parent to supplement feedings with cereal Proceed with the recommended vaccine Treat the infant for excessive abdominal gas Advise the parent to return in one month to reassess

Proceed with the recommended vaccine

An advanced practice registered nurse (APRN) is evaluating a two-month-old infant for a well-baby checkup. The APRN assesses the infant's speech and language milestones to determine overall developmental status. Which milestone should this APRN expect? Quiets in response to the mother's voice Responds to angry voices Looks at the mother when name is called Actively seeks sound source

Quiets in response to the mother's voice

A 7-year-old female has recently developed pubic and axillary hair without breast development. Her bone age is consistent with her chronological age, and a pediatric endocrinologist has diagnosed idiopathic premature adrenarche. The Family nurse practitioner will monitor this child for which condition? 1/1 Adrenal tumor Congenital adrenal hyperplasia Polycystic ovary syndrome Type 1 diabetes mellitus

c

an eight month old patient has difficulty sleeping at night without being held by the parents. The family nurse practitioner understands that this is a result of: 1/1 sleep apnea sleep terrors learned behavior insomnia

c

An advanced practice registered nurse (APRN) is seeing a 28-year-old male who is presenting with fever, chills, malaise, myalgia, and perineal pain. The patient reports urgency with urination. Upon examination, the prostate is tender, warm, boggy, and irregular on palpation. The urinalysis indicates WBCs > 10/hpf and positive bacteria. Which condition should be ruled out before the APRN determines a management plan for this patient? Testicular torsion Viral infection of the prostate Sexually transmitted infection Benign prostatic hypertrophy

Sexually transmitted infection

The Family nurse practitioner performs a developmental assessment on a 32-month-old child. The child's parent reports that about 70% of the child's speech is intelligible. The pediatric nurse practitioner observes that the child has difficulty pronouncing "t," "d," "k," and "g" sounds. Which action is correct? 1/1

c Evaluate the child's cognitive abilities. Obtain a hearing evaluation. Reassure the parent that this is normal Refer the child to a speech therapist.

The following are seen in TEC: • Anemia (in which the Hgb content may be as low as 2.5 g/dL or only slightly decreased but generally around 6 to 8 g/dL) • Markedly low reticulocyte count • MCV characteristically normal for age • WBC count usually normal but some degree of neutropenia can occur in up to 20% • Platelet count normal or elevated • High serum iron level reflecting decreased utilization • Bone marrow aspiration indicating erythroid hypoplasia

The following are seen in TEC: • Anemia (in which the Hgb content may be as low as 2.5 g/dL or only slightly decreased but generally around 6 to 8 g/dL) • Markedly low reticulocyte count • MCV characteristically normal for age • WBC count usually normal but some degree of neutropenia can occur in up to 20% • Platelet count normal or elevated • High serum iron level reflecting decreased utilization • Bone marrow aspiration indicating erythroid hypoplasia

scarlet fever

There is abrupt illness with sore throat, vomiting, headache, chills, and malaise. Fever reaches 104°F (40°C). Tonsils are erythematous, edematous, and usually exudative. The rash generally begins on the neck and spreads to the trunk and extremities becoming generalized within 24 hours. The face may be spared (cheeks may be reddened with circumoral pallor), but the rash is denser on the neck, axilla, and groin

Renal Tubular Acidosis

Type 1 - distal (impaired distal tubule acidification) - urine pH > 5.5, hypokalemia Type 2 - proximal (reduced tubular bicarbonate reabsorption) hypokalemia , urine pH <5.5 (intact ability to acidify urine); HCO3 level 16-18 Type 4 - hyperkalemia ; due to hyporeninemic hypoaldosteronism, dRTA has a rapid response to treatment, and normal bicarbonate levels are maintained with little difficulty. lasts a life time pRTA requires higher doses to normalize bicarbonate and is less easily maintained. Type IV RTA requires mineralocorticoid treatment if aldosterone is deficient.

Vesicoureteral Reflux

Vesicoureteral Reflux VUR is retrograde regurgitation of urine from the bladder into the ureters, and potentially the kidney. The major concern with VUR is the exposure of the kidney to infected urine which may cause pyelonephritis. Primary VUR is the most common type and typically involves an abnormally short ureter and ineffective valve. Secondary VUR is due to either functional or structural bladder outlet obstruction. It is graded according to an international classification (Fig 41.2). Grade I does not reach the renal pelvis; grade II extends up to the renal pelvis without dilation; grade III describes reflux to the renal pelvis with mild to moderate dilation of the ureter and the renal pelvis; grades IV and V (high grade) include definite distention of the ureters and renal pelvis and can include hydronephrosis or reflux into the intrarenal collecting system

A 27-year-old female presents to the clinic. The advanced practice registered nurse (APRN) is told about the following symptoms:· intense vulvar and vaginal itching thick, creamy vaginal discharge vulvar erythema Which diagnosis should the APRN make for this patient? Bacterial vaginosis Pelvic inflammatory disease Vulvovaginal candidiasis Trichomoniasis vaginitis

Vulvovaginal candidiasis

42 An advanced practice registered nurse (APRN) is examining a 96-year-old patient who is complaining of an ear infection. The patient reports currently taking four pills every day. The APRN decides to respond to that information by taking a course of action recommended by the goals of geriatric care. Which action should the APRN take for this patient? Avoid any additional medications and wait to see if it resolves Identify the strongest antibiotic to take to prevent a super infection Write the reason for the medication on the patient's prescription Add a medication to avoid a yeast infection

Write the reason for the medication on the patient's prescription

The Family nurse practitioner is counseling the parents of a toddler about appropriate discipline. The parents report that the child is very active and curious, and they are worried about the potential for injury. What will the pediatric nurse practitioner recommend? 1/1

a Allow the child to explore and experiment while providing appropriate limits. Be present while the child plays to continually teach the child what is appropriate Let the child experiment at will and to make mistakes in order to learn. Say "no" whenever the child does something that is not acceptable.

during a well-child visit, a mother expresses concern that her three-year-old has recently started to stutter when excited. The nurse practitioner informs the mother that stuttering in a preschool-age child: 1/1 is a normal characteristic of language development is a sign of abnormal speech pattern development occurs because the child does not know what to say signals the presence of stress in the child's environment Feedback d

a three to four year old children have normal hesitancy in speech or stuttering. They will repeat words especially when stating a full sentence or when excited. It should not include syllable repetition. This usually resolves by 5 years of age and should not cause undue stress to the chil

An 18-month-old child with no previous history of otitis media awoke during the night with right ear pain. The Family nurse practitioner notes an axillary temperature of 100.5°F and an erythematous, bulging tympanic membrane. A tympanogram reveals of peak of +150 mm H2O. What is the recommended treatment for this child? 0/1 Amoxicillin 80 to 90 mg/kg/day in two divided doses An analgesic medication and watchful waiting Ceftriaxone 50 to 75 mg/kg/dose IM given once Ototopical antibiotic drops twice daily for 5 days

b

The Family nurse practitioner performs a Hirschberg test to evaluate what? 1/1 color vision ocular alignment peripheral vision Visual acuity

b

the family nurse practitioner examines a three year old patient who underwent a myringotomy 4 months ago.65% of the child speech remains unintelligible. The FNP's response is to: 1/1 consider the patient's history, but assessing the performance against standards, and recommend no action for 8 months interpret the patient's speech performance against expected standards and recommend a referral to an audiologist listen to the mothers report that the child speech is progressing normally and recommend no further intervention for 12 months recommend that the child is at a pediatric speech therapist because children should be speaking by 3 years of age Feedback

b The FNP should consider the speech development against expected standards and recommend further evaluation of hearing. The three-year-old should have 90% intelligible speech and if delayed it requires further assessment. Delays should be addressed immediately to promote the best outcome for children.

a four year old boy weighing 18 kg is diagnosed with bilateral otitis media. His last year infection was six months ago, and he has no known drug allergies. What would be an appropriate medication to prescribe? 1/1 ampicillin 40-50 mg per kilogram per day tid x 7 days corticosteroid otic solution three gtts both ears x 10 days amoxicillin 75 to 90 mg per kg per day bid x 10 days doxycycline 250 mg one teaspoon po tid x 10 days

c Feedback the recommended treatment for otitis media in a child is amoxicillin 75 to 90 milligrams per kilogram per day, or for this child who is 18kg, 250mg/ml, 2.75 tsp bid for 10 days. The other drugs and doses listed are not appropriate treatment for acute otitis media

what are the clinical manifestations of heart failure in an infant? 0/1 easily fatigued, central cyanosis, tachycardia, tachynpnea, and hepatomegaly coughing, diaphoresis, peripheral edema, and hepatomegaly tachycardia, tachypnea, easily fatigued, pallor, hepatomegaly peripheral edema, coughing, splenomegaly, hepatomegaly, and tachycardia

c-Correct answer tachycardia, tachypnea, easily fatigued, pallor, hepatomegaly Feedback most cases of heart failure in infants result from congenital heart disease during the first 12 months of Life. Symptoms result from the decreased cardiac output and the infant's compensatory mechanisms. Symptoms include tachypnea, dyspnea, tachycardia, pallor, and easy fatigability. Additional symptoms include periorbital edema, hepatomegaly, difficult feeding, and persistent cough. Diaphoresis, central cyanosis, and peripheral edema are not necessarily associated with heart failure that may be manifestations of the underlying congenital heart defect.

an adolescent patient has had yellowish green nasal discharge and a frontal headache for a week. The Adolescents temperature has gone up to 101F on most afternoons, and she has a cough that worsens when she lies down. The physical exam is within normal limits except for the drainage and a slightly erythematous pharynx. She does not have any drug allergies and has not been taking any medications in the last few months. What medication would be the best to prescribe for her? 0/1 diphenhydramine (Benadryl) erythromycin (E-mycin) pseudoephedrine (Sudafed) Amoxicillin/Clavulanate (Augmentin)

cCorrect answer Amoxicillin/Clavulanate (Augmentin) Feedback the patient is experiencing symptoms of moderately severe acute sinusitis based on the symptoms of facial pressure, headache, and post nasal discharge. The first line antibiotic to prescribe is Augmentin for its safety and efficacy. Oral antihistamines, such as Bendadryl should not be used unless the patient has allergies. Oral decongestants such as sudafed are not as effective in patients with sinusitis

he mother of a female infant is concerned that her daughter is developing breasts. The Family nurse practitioner notes mild breast development but no pubic or axillary hair. What is the likely diagnosis? 1/1 Congenital adrenal hyperplasia causing breast development Precocious puberty needing endocrinology management Premature adrenarche which will lead to pubic hair onset Premature thelarche which will resolve over time

d

a five year old male patient is brought to the Outpatient Clinic with a child's mother complaining he has not urinated in the last 24 hours. The child had nausea, diarrhea, abdominal pain, and a low-grade fever for 3 days, which he states started improving yesterday. Physical exam reveals no abnormalities. Laboratory analysis reveals hemoglobin 10.1 G / DL, hematocrit 26%, and platelets 90000 mm 3. Your analysis is unable to be obtained initially. Which of the following should the family nurse practitioner suspect? 1/1 urinary retention viral syndrome nephrotic syndrome hemolytic uremic syndrome (HUS) F

eedback HUS is most often caused by Shiga toxin-producing escherichia coli (STEC) or shigella and less commonly by streptococcus pneumoniae. Urinary symptoms and thrombocytopenia in HUS typically do not manifest until several days after the presentation of nausea, vomiting, and diarrhea which is often bloody. Immediate supportive therapy is indicated, including transfusions and dialysis based on the severity of symptoms

Menometrorrhagia

excessive uterine bleeding at both the usual time of menstrual periods and at other irregular intervals

Epididymitis

inflammation of the epididymis that is frequently caused by the spread of infection from the urethra or the bladder First line: Ceftriaxone (250 mg intramuscularly one time) plus doxycycline (100 mg twice a day for 10 days). • Alternative treatments: Ofloxacin (

Osgood-Schlatter disease

inflammation or irritation of the tibia at its point of attachment with the patellar tendon

macrocytic

macrocytic; commonly include weakness, pallor, and a beefy-red, smooth sore mouth and tongue.Treatment is dietary supplementation and correction of the underlying disorder (e.g., infection) if possible.

Common vaccinations contraindicated or not recommended in pregnancy include

measles, mumps, and rubella (MMR); varicella; herpes zoster; live, attenuated influenza; and human papillomavirus (HPV)

normocytic Anemias

normocytic Anemias that have an RBC size within the normal range are termed normocytic. Normocytic anemias tend to coincide with chronic illness, B12 deficiency, traumatic blood loss, or pregnancy.

o ASD

o ASD is a defect or hole in the atrial septum and accounts for 5% to 10% of all CHD Larger defects require intervention, usually after the child is 1 year old, before school entry, or when the defect is identified in an older child. Most small to moderate ostium secundum ASDs can be closed percutaneously in the cardiac catheterization laboratory if the child weighs more than 15 kgs and there are adequate margins to anchor the device. If the defect is large or unfavorable for device closure, cardiac surgery is indicated

The parent of a school-age child tells you that the child is restless most nights and often complains that bugs are in the bed. The child consults a sleep disorder specialist, and a subsequent evaluation shows a ferritin level of 30. Which medication should you treat this child with? Gabapentin Clonazepam Ferrous sulfate Sertraline

orrect! A ferritin level of less than 50 is associated with periodic limb movements (PLM). The treatment for this is ferrous sulfate 3 mg/kg per day. Clonazepam and gabapentin may be ordered if ferritin levels are normal and other organic causes of PLM have been ruled out. Sertraline may make PLM worse.

thalassemia

thalassemia's; β Thalassemia Major Affected infants usually become symptomatic in the first year of life and have pallor, failure to thrive, hepatosplenomegaly, and a severe anemia with an average Hgb of 6 g/dL and low MCV (60 to 70 fL). RBC morphology reveals significant microcytosis, poikilocytosis, hypochromia, target cells, and nucleated RBCs. Hgb A2 and Hgb F levels are elevate RBC transfusions are usually necessary every 2 to 4 weeks with the goal of maintaining a pretransfusion Hgb level between 9.5 and 10.5 g/dL., Iron chelation is necessary to t

Polymenorrhea

the occurrence of menstrual cycles more frequently than is normal

True or False Pregnant women should receive the influenza nasal spray vaccine. True False

x Correct! Pregnant women should not receive the influenza nasal spray vaccine, because it contains a live, attenuated virus. Pregnant patients should be given a flu shot instead.

Nephrotic syndrome

§ Nephrotic syndrome is due to excessive excretion of protein in urine as a result of alterations in the integrity of the glomerular filtration barrier. § The classic definition of nephrotic syndrome is massive proteinuria (3 to 4+ protein with UA § History of allergy in up to 50% of children with MCNS § • Edema is the cardinal clinical feature, especially periorbital edema, dependent areas (tight shoes or underwear), and lax tissues (puffy eyes) § • Low urine production § • Gastrointestinal symptoms: Anorexia, paleness, listlessness, diarrhea, vomiting, abdominal pain (right upper quadrant) § • Respiratory difficulties secondary to ascites, effusion, pneumonia in advanced disease

pulmonic stenosis

§ Normally the pulmonary valve opens to allow the flow of blood from the right ventricle into the pulmonary artery. In pulmonic stenosis, there is narrowing at the subpulmonic, valvular, or supravalvular area. Right-sided pressure is increased as the ventricle pumps against the obstruction. Right ventricular hypertrophy occurs as a result of this increased load § Balloon valvuloplasty in neonates and older children with stenosis greater than 50 mm Hg are performed. If unsuccessful, surgical valvuloplasty or replacement may be indicated. Stents and balloons are also used for branch stenosis.

· tetralogy of falot

§ Tetralogy of Fallot (TOF; also referred to as TET) is a combination of four anatomic cardiac defects resulting in right ventricular outflow tract (RVOT) obstruction: (1) pulmonary valve stenosis, (2) right ventricular hypertrophy, (3) VSD, and (4) an aorta that overrides the ventricular septum (Fig 38.11). It is the most common cyanotic cardiac lesion §

ITP

· Immune or ITP is the most common of the thrombocytopenic purpuras in childhood and is believed to be an autoimmune response in which circulating platelets are destroyed. It usually occurs after viral illnesses. In many cases the cause is autoimmune. The prognosis for patients with ITP is excellent, with spontaneous recovery in the majority of pediatric cases within the first 6 months. Most cases can be managed on an outpatient basis without any specific therapy. If the platelet count is greater than 20,000/mm3 and no bleeding is observed, children and parents should be advised to avoid contact sports, aspirin and other NSAIDs,

Sickle cell--admission or referral is necessary in the presence of the following:

• Fever (to rule out sepsis) greater than 101°F (38.3°C) • Pneumonia, chest pain, or other pulmonary symptoms (acute chest syndrome) • Sequestration crisis (splenomegaly with decreased Hgb or Hct) • Aplastic crisis (decreased Hct and reticulocyte count) • Severe painful crisis, priapism • Unusual headache, visual disturbances

You conduct a well-baby examination on an infant and note mild gross motor delays but no delays in other areas. What should be your initial course of action? Teach the parents to provide exercises to encourage motor development. Prepare the parents for a potentially serious developmental disorder. Refer the infant to an early intervention program for physical therapy. Consult a developmental specialist for a complete evaluation.

Correct! A child who has mild delays in only one area may be managed initially by having the parent provide appropriate exercises. If this is not effective, or if delays become more severe, referrals for evaluation or early intervention services are warranted. A mild delay does not necessarily signal a serious disorder.

You are counseling the parents of a toddler about appropriate discipline. The parents report that the child is very active and curious, and they are worried about the potential for injury. What should you recommend to the parents? Say "No!" whenever the child does something that is not acceptable. Be present while the child plays to continually teach the child what is appropriate. Allow the child to explore and experiment while providing appropriate limits. Let the child experiment at will and make mistakes in order to learn.

Correct! A child who is securely attached uses parents as a base from which to safely explore the world. Toddlers learn by doing and need to experiment to gain mastery over the environment.

You enter an examination room to find a two-month-old infant in a car seat on the examination table. The infant's mother is playing a game on her smartphone. How might you interpret this behavior? A sign that the mother has postpartum depression A moderate concern for parenting problems A sign of the normal range of behavior in early parenthood An extreme concern for potential parental neglect

Correct! A parent who seems disinterested in a child raises moderate concerns for parenting problems.

You perform a well-child assessment on a six-month-old infant whose mother reports having less breast milk because of stressors associated with pumping and returning to work. You provide resources to promote pumping. What else should you do for the mother? Prescribe the infant a multivitamin containing iron. Suggest offering only breast milk to the infant. Encourage the mother to increase her fluid intake. Discuss adding other foods to the baby's diet.

Correct! After six months, infants should continue to breastfeed while taking other nutrients at least up to one year of age. The mother should be encouraged to continue breastfeeding while adding other nutrients.

You conduct a well-baby examination on an eight-month-old infant and note mild gross motor delays but no delays in other areas. Which initial course of action should you take? Consult a developmental specialist for a complete evaluation. Teach the parents to provide exercises to encourage motor development. Prepare the parents for diagnosis of a potentially serious developmental disorder. Refer the infant to an early intervention program for physical therapy.

Correct! All children progress at different rates. Encouraging play that engages gross motor movement will help with development. Other actions would be considered if the examination showed multiple delays.

You are examining a young child who was brought in by a grandmother for evaluation of a partial thickness burn on one arm. You suspect that this is an intentional injury, but the grandmother states that the parents are "just careless" and that the child is now living with her. What should you do? Report a suspicion of abuse to child protective services. Note this as a concerning incident in the child's record. Refer the child's parents to a parenting resource center. Reassure the grandmother that she is doing the right thing.

Correct! All states have mandatory reporting laws that require healthcare professionals to report suspected or known abuse to the appropriate agencies and provide both civil and criminal immunity to mandated reporters. The other options may be necessary once the case is investigated, but the priority is to report the suspicion of abuse.

The parents of a 12-year-old child are concerned that some of the child's older classmates may be a bad influence on their child, who, they say, has been raised to believe in right and wrong. What should you tell the parents? Children at this age have a high regard for authority and social norms, so it is not likely that the child will be influenced by classmates. The pressures from outside influences may supersede parental teachings and should be confronted. Moral values instilled in the early school-age period will persist throughout childhood. Allow the child to make poor choices and accept the consequences as an important way to learn values.

Correct! Although early school-age children learn values from their parents, these values may be challenged as children learn that others have different values. Parents must confront and negotiate these issues daily with their children. Although children may make poor choices and subsequently learn from the consequences, it is best for parents to actively discuss these issues with their children. Children do have a high regard for authority and social norms, but they may easily transfer this authority to other, less reliable people, such as peers. Moral values may not persist if other sources of authority become prominent.

You are performing a well-child examination on a 12-year-old female who has achieved early sexual maturation. The mother reports that the girl spends more time with her older sister's friends instead of her own classmates. What should you tell this parent? The association with older adolescents will help her daughter gain social maturity. Girls who mature early need to identify with older adolescents to feel a sense of belonging. The desire to spend time with older adolescents indicates a healthy adjustment to her maturing body. Girls who join an older group of peers may become sexually active at an earlier age.

Correct! Although females who mature early may join an older group of peers to feel that they fit in, the ones who do increase their likelihood of engaging in risky behaviors, including sexual activity. Many teens feel awkward when they mature at different rates than their peers. But for teens to join a group of older peers demonstrates a poor adjustment to the changes and does not promote social maturity.

An advanced practice registered nurse (APRN) is performing a wellness check for an adolescent patient. The patient's parents are concerned about exposure to pictures and videos that are not appropriate for the teen's age, and the patient admits to excessive use of social media. Which evidence-based management strategy should the APRN use for this patient? Ask parents to preview and approve the websites and applications used by the teen Advise the parents to use software tracking and remote monitoring to ensure safety Encourage the teen to avoid social media as a means of maintaining relationships Explain that teens have the ability to manage negative interactions online

Correct! An important part of managing social media usage involves implementing parental education and encouraging supervision of social media usage. Ask parents to preview and approve websites and applications being used by the adolescent.

During a well-child examination on an infant who has colic, you learn that the infant's mother is 17 years old and that the father, who is in the military, has been deployed to duty since shortly after the baby was born. What should you ask the mother about to determine the immediate risk of child maltreatment for this infant? Location of extended family members Child-rearing and parenting styles Spiritual beliefs and religious practices Parents' role responsibilities

Correct! Assessment of resources, including the support of extended family members, is a key dimension of family functioning. In this case, the mother is young and alone and may lack the skills needed to cope with an infant with colic. Child-rearing and parenting styles can affect the emotional and physical health of children who misbehave or who are learning how to behave in the world. An assessment of role responsibilities is important when there are disagreements about shared responsibilities. Assessing spiritual beliefs helps to determine the values ascribed to events. While all of these are important assessments, there is an urgent need to determine the level of support available to this mother.

The parent of a 10-year-old boy tells you that the child does not appear to have any interest in girls and spends most of his time with a couple of other boys. The parent is questioning the child's sexual identity. What should you say to this parent? Children at this age who prefer interactions with same-gender peers usually have a homosexual orientation. Children at this age may have an attachment to other same-gender children to learn how to interact with people. Children should be encouraged to have mixed-gender interactions to promote development of sexual values. Children experiment with sexuality at this age as a means of deciding later sexual orientation.

Correct! At age 10, children usually develop an intense same-gender relationship with a peer. This is how children learn to expand their self-identity, share feelings, and learn how other people manage problems. It does not indicate later sexual orientation and is not a characteristic of experimentation with sexuality. It is not necessary to encourage mixed-gender interactions.

Question 1 This is not a form; we suggest that you use the browse mode and read all parts of the question carefully. A child has failed kindergarten screening and is brought to you for evaluation. The child has not had any medical care since the 18-month well visit. On your examination, you note the child cannot balance on one foot, draw a circle, count to three, or name colors. What should you do? Refer the child for occupational therapy. Counsel the parent on the need for regular checkups to detect delays, and refer the child to a developmental specialist. Recommend the child wait a year to take the screening again. Assure the parent that the child will catch up.

Correct! Because of the child's significant developmental delays, you should refer the child to specialists for additional testing. You should also provide parental counseling. Both are an important part of the initial treatment for this child.

The parent of a five-year-old child tells you that the child has been using the toilet to urinate since age three but continues to defecate in training pants. You learn that the child has predictable bowel movements. A physical examination is normal. What should you recommend to the parent? Use polyethylene glycol until the child is able to regularly use the toilet Place the child on the toilet for 5 to 10 minutes at the usual time of defecation Put the child back in diapers and resume toilet training in a few months Provide a reward system to offer incentives when the child uses the toilet

Correct! Because this child has predictable bowel patterns, the parent can put the child on the toilet for 5 to 10 minutes at a time to encourage toilet use. Rewards may be used at some point, but it is not recommended because the child is learning to do what is to be expected. Younger children may be put back in diapers and retrained in a few months. The child is not constipated and does not need medication.

You are performing a well-child examination on an eight-year-old girl and note the presence of breast buds. What should you include when initiating anticipatory guidance for this patient? A discussion about the risks of pregnancy and sexually transmitted diseases Material about the human papillomavirus (HPV) vaccine Information about sexual maturity and menstrual periods Sexual orientation and the nature of sexual relationships

Correct! Because this child is eight years old, it is early to discuss sexual behavior and reproduction given the level of the child's cognition and understanding. However, with these early changes in thelarche marking the onset of puberty, it is wise to discuss menstruation in an age-appropriate manner before it occurs so that the child can be prepared. Because this child is showing signs of early puberty, this information can be included in anticipatory guidance.

A female adolescent patient presents to a clinic with complaints of anxiety. Upon further review of the patient, an advanced practice registered nurse (APRN) notes that the patient is very self-conscious about her appearance, and she is focused on the present. The patient's mother reports her daughter has been requesting to do more things on her own and has been wanting more privacy. Which stage represents this adolescent? Middle adolescence Variable adolescence Late adolescence Early adolescence

Correct! Being self-conscious about appearance, focusing on the present, being independent, and wanting more privacy are social and emotional developmental indications of early adolescence.

An advanced practice registered nurse (APRN) examines a nine-year-old child, and the child's parents express concerns that their child is not participating in enough physical activity. Which recommendation should the APRN share with these parents? Limit screen time to no more than three hours per day Encourage sports specialization Avoid strength training Strive for 60 minutes of physical activity to occur daily

Correct! Children and adolescents should strive for 60 minutes of physical activity each day. The minutes do not need to be continuous.

During a well-child assessment of an 18-month-old child, you observe the child becoming irritable and uncooperative. The parent tells the child to stop fussing. What should you do? Offer the child a book or toy. Allow the parent to put the child in a time-out. Stop the examination because the child is having a meltdown. Ask the parent about usual discipline practices.

Correct! Children of this age require frequent entertainment and interaction. It is normal for a child to become fussy during an assessment that requires the child to remain still for a time, and this is an opportunity for parental education. Educate the parent that providing easy toys for entertainment will be more effective than discipline. The child is not misbehaving but needs a diversion. Providing a diversion through toys or other entertainment will help diffuse the situation.

The parent of a school-age child is concerned that the child is going to be short like both of his parents. The parent worries that the child will have difficulty in school if he cannot participate in a variety of sports. What counsel should you give this parent? Steer the child to other activities at school Point out the accomplishments of other short people Encourage the parents to overlook their feelings about this physical characteristic Encourage the child to engage in regular physical activity

Correct! Children should be encouraged to participate in all activities and to explore their own abilities and limitations without attributing "good" or "bad" to any of these experiences to improve self-esteem. Parents should be encouraged to explore their own feelings about their child's appearance to avoid subliminal messages of low self-worth. Pointing out the characteristic puts an unnecessary focus on the trait. Steering the child to specific activities is limiting.

The father of a three-year-old child presents with concerns about his child's behavior. He expresses concern that after he and his spouse had a minor argument the child began hitting the family dog with a toy and poked at its eyes despite the dog yelping in pain. After he got the dog away from the child, the child began hitting himself. The father states the argument was a minor disagreement that did not become loud or physical. What is the most appropriate action for you to take? Provide the father with guidance on nonviolent discipline methods. Immediately refer the child for psychological evaluation. Make a referral to children's protective services. Reassure the father that children do not understand the consequences of their actions and the child did not intend to hurt the animal.

Correct! Children who display cruelty to animals or engage in self-harm behaviors should be referred for immediate evaluation.

The parent of a 15-year-old male is concerned that he refuses to eat meals with the family and consumes only protein drinks. The adolescent is on the track team at school and spends much of his time training and working out. You note that his weight and BMI have dropped from the 20th percentile to the 3rd percentile in the past year. What does this child most likely have a problem with? Role performance Body image Personal identity Self-esteem

Correct! Children with disturbed body image problems may have concerns related to appearance, body size, function, or potential. Possible behaviors include eating disorders and a preoccupation with the perfect body. Children with body image problems become very concerned with appearance and compare themselves to others. This child is losing weight and working out too often in an effort to change his body.

A seven-year-old female has recently developed pubic and axillary hair without breast development. Her bone age is consistent with her chronological age, and a pediatric endocrinologist has diagnosed idiopathic premature adrenarche. Which condition should you monitor in this child? Type 1 diabetes mellitus An adrenal tumor Polycystic ovary syndrome (PCOS) Congenital adrenal hyperplasia (CAH)

Correct! Children with idiopathic premature adrenarche are at increased risk for PCOS and metabolic syndrome. An adrenal tumor and CAH are both possible causes of premature adrenarche, but these have been ruled out to make a diagnosis of idiopathic premature adrenarche.

You are evaluating a 16-year-old male who is on the high school wrestling team and whose weight fluctuates as much as seven or eight pounds before matches. The child is eager to talk about the various trophies he has won. When he expresses confidence that he will get a wrestling scholarship for college, his father remarks that his grades will never be good enough for college. The child responds that his teachers are to blame for his poor grades. What is a potential problem for you to identify? Role performance Body image Personal identity Self-esteem

Correct! Children with poor self-esteem seek attention, importance, and security and may become self-absorbed with external markers of self-worth, such as their performance in a sport. Another mark of insecurity is defensiveness, which this child exhibits by blaming his teachers for his poor grades.

You are examining a school-age child who complains of frequent stomach pain and headaches. The parent reports that the child misses several days of school each month. The child has a normal examination. What should you initially ask the parent before proceeding with further diagnostic tests? Whether the parent feels a strong need to protect the child from problems How well the child performs in school and extracurricular activities Whether there are any unusual stressors or circumstances at home About the timing of the symptoms each day and during the week

Correct! Children with school refusal or school phobia often have symptoms that gradually improve as the day progresses and often disappear on weekends. You should ask about the frequency and duration of the symptoms to evaluate this pattern. Once management of school phobia has begun, the other options are important questions to ask in order to understand the underlying causes for the reluctance to go to school.

A seven-year-old patient is being evaluated by an advanced practice registered nurse (APRN) for a history of anxiety, a compulsion to rearrange the furniture at home, and the need to check door locks repeatedly during the day. The APRN diagnoses obsessive-compulsive disorder (OCD). Which action should the APRN take first to assist this patient? Prescribe SSRI medications Initiate cognitive behavioral therapy Refer to child psychiatry Teach relaxation techniques

Correct! Cognitive behavioral therapy provides the best long-term effectiveness for this problem and is considered first-line therapy for all children and adolescents with mild to moderate OCD.

You perform a physical examination on a nine-month-old infant and note two central incisors on the lower gums. The parent states that the infant nurses, takes solid foods three times daily, and occasionally takes water from a cup. What should you tell the parent to promote optimal dental health? Discontinue giving fluoride supplements Begin brushing the infant's teeth with toothpaste Make an appointment for an initial dental examination Consider weaning the infant from breastfeeding

Correct! Dental examinations should begin when teeth appear. Toothpaste is not yet required. Breastfeeding cessation is not required and should continue. Fluoride supplements should continue as well.

You are performing a well-child examination on an adolescent who was adopted as a toddler. The parent reports that the child was removed from an abusive home at the age of three years old. What should you evaluate for possible long-term effects of this early situation? Mental health and suicide risk Cognitive and psychosocial development Moral development and conscience formation Spirituality, faith, and religious affiliation

Correct! Early research findings suggest that epigenetic changes may mediate the effect of the prenatal and infant environment on the mental health and disease in older children and adults. Adult suicide victims who were abused as children show decreased levels of a genetic marker of a stress reduction gene, suggesting that child abuse may have an epigenetic effect leading to prolonged stress and mental disorders. Mental health and suicide risk must be monitored.

The parent of a five-year-old child who has just begun kindergarten expresses concern that the child is having difficulty adjusting to the recent birth of a sibling. What should be your main recommendation to the parent? Allow the child opportunities to discuss feelings about the baby Provide reassurance that the sibling will not replace the child Give the child specific baby care tasks to promote sibling bonding Have snack time with the child each day to discuss the school day

Correct! Family routines provide support to children and help them self-regulate, especially during times of change. Routines also serve as a buffer during times of change and transition. This child is experiencing two major changes, so setting aside regular time to spend with the child will help stabilize these changes. The other options may be useful as well, but routines and special activities are most important.

An advanced practice registered nurse (APRN) is examining a two-year-old child for a routine physical examination and notices that the child appears to have strabismus in the right eye. Which initial action should this APRN take? Perform a test to measure the pressure in each eye Have the parent provide high amounts of visual stimulation Recommend that the right eye be patched for six months Refer the patient to an ophthalmologist

Correct! Fixed or intermittent strabismus persisting longer than six months since birth is a warning sign for visual impairment. An ophthalmologist referral would be indicated in this case.

The parent of a school-age child reports that the child is on a gluten-free diet. When questioned about the reason for this diet, the parent states that the child has fewer stomachaches since beginning the diet. The child has never been diagnosed with celiac disease. The parent reports using gluten-free grain products for all family members. What should you tell this parent about gluten-free grain products? They may be deficient in essential nutrients. They are generally low in sugar and fat. They are healthy and help prevent obesity. They provide adequate protein to meet daily needs.

Correct! Gluten-free grain products are often highly processed and not enriched with iron or folate. Many are very low in protein, enough so that they are used for patients with metabolic conditions such as phenylketonuria (PKU) or who have severe protein restrictions. They often have sugar and fat added to them to improve taste, and they do not help prevent obesity.

You are counseling an adolescent with a family history of hyperinsulinemia and type 2 diabetes. Which foods should the patient avoid eating? Baked potato chips High-fiber cereals Canned vegetables Processed breads

Correct! High-glycemic foods such as soft drinks, sweetened juices, and processed breads, pastries, and crackers are more quickly converted to serum glucose. They stimulate a sharp rise in insulin production and a subsequent rapid shift into hypoglycemia. To help prevent this in a child with a family history of this disorder, you should recommend that the child avoid processed breads, pastries, and crackers. High levels of fructose and low fiber intake also contributes to this phenomenon.

You are evaluating a 12-year-old female who reports penile penetration of her vagina by her mother's boyfriend two days ago. You report this to the local child abuse hotline. What should your next action be? Obtain urethral specimens for sexually transmitted infection (STI) testing Perform a colposcopic examination to evaluate for trauma Obtain a history of the abuse from the child Refer the child to the emergency department for forensic specimen collection

Correct! If sexual abuse has occurred within 72 hours, it is required that appropriate forensic specimens be collected. The child abuse team will obtain a history from the child as part of the child abuse evaluation. They will also obtain urethral specimens for STI testing. An expert in sexual abuse will complete a colposcopic examination if trauma is suspected.

It is important for you to assess a family's beliefs on infant sleeping habits. Doing so will help ensure the family heeds your direction regarding a patient's sleep. What are some families' erroneous beliefs about infant sleeping? Using a family bed Providing a safe sleep environment Placing children on their back to sleep Using a new crib

Correct! In some cultures, a family bed is the norm. Ensure parents understand the importance of infants sleeping in their own beds to help prevent injury and death from parents rolling onto or smothering the child during sleep. Providing a safe area for children to sleep and avoiding a family bed entirely improves sleeping safety for children.

The mother of a six-week-old breastfeeding infant tells you that her baby, who previously had bowel movements with each feeding, now has a bowel movement only once every third day. What should you tell her? This change may be related to the baby's dietary intake. The baby should consume more water. The baby is probably constipated. This change may be normal for breastfed babies.

Correct! Infants begin to have fewer bowel movements. These bowel movements may range from once or twice daily to once every other day when breastfeeding. Unless there are other signs, the baby is probably not constipated. The mother does not need to change the baby's intake of foods or water unless constipation is present.qq

An advanced practice registered nurse (APRN) is evaluating a four-month-old infant during a well-child visit, and the infant's parent mentions the infant is waking up and crying several times a night, but after a short time of rocking, the baby will go back to sleep. The parent asks the APRN how to reduce the frequency of this problem. Which recommendation should the APRN suggest to this parent? Instruct the parent to have the child sleep with the parent Allow the child to self-soothe when awaking at night Give the child a bottle to help him return to sleep Provide soothing music to help the child maintain sleep

Correct! Infants need to be allowed to self-soothe when they awaken at night. Infants who are placed in cribs while drowsy but not yet asleep are more likely to go back to sleep without comfort from the parent when they wake.

During an assessment of a four-week-old infant, you learn that the breastfed infant nurses every two hours during the day but is able to sleep for a four-hour period during the night. The infant has gained 20 grams per day since last seen in the clinic. What should you recommend to the parent? Continue to nurse the infant using the current pattern Nurse the infant for longer periods every four hours Supplement with formula at the last nighttime feeding Wake the infant every two hours to nurse during the night

Correct! Infants who are encouraged to breastfeed every two to three hours may have one longer stretch of four hours at night without a feeding. This infant is gaining between 0.5 and one gram per day, which is appropriate. It is not necessary to alter the pattern of nursing or to supplement with formula.

You are performing an assessment on a one-week-old newborn with a slightly elevated bilirubin level. The infant is breastfeeding well and has gained 30 grams in the past 24 hours. The infant is stooling and voiding well. You suspect breast milk jaundice. What should you do? Recheck the serum bilirubin and the infant's weight in 24 hours. Recommend that the mother pump her breast milk for a couple of days. Reassure the mother that the bilirubin levels will drop in a few days. Order home phototherapy and closely monitor the infant's bilirubin levels.

Correct! Infants with breast milk jaundice who are gaining weight and thriving should continue to breastfeed and be monitored for the development of pathologic jaundice. It is not necessary to order phototherapy or discontinue breastfeeding unless pathologic jaundice is present. The bilirubin levels may remain elevated for up to three months.

You perform a developmental assessment on a 32-month-old child. The child's parent reports that approximately 70% of the child's speech is intelligible. You observe that the child has difficulty pronouncing t, c, d, k, and g sounds. What should you do? Reassure the parent that this is normal. Evaluate the child's cognitive abilities. Refer the child to a speech therapist. Obtain a hearing evaluation. .

Correct! Intelligibility of speech reaches approximately 66% between the ages of 24 and 36 months. Tongue-contact sounds are more intelligible by age five. This child exhibits normal speech for the child's age

An advanced practice registered nurse (APRN) is providing safety education strategies to the parents of a six-year-old child. Which strategy should be recommended by this APRN? Move booster chair to front seat to improve visibility Provide structured time for play Limit screen time to three hours per day Wear protective gear when riding a bicycle

Correct! It is important to follow safety guidelines (e.g., wearing a helmet or athletic supporter, baseball pitching and throwing limitations) for all activities.

Seven-month-old twins and their mother are meeting with an advanced practice registered nurse (APRN). The mother is concerned because one of the infants does not babble or startle to noises like the other twin. The APRN verifies the differences in the responses stated by the mother but has no other concerns. Which action should the APRN take next to address this mother's concern? Complete a full assessment during the next visit Refer to an audiologist Start a course of antibiotics Schedule a consultation with a pediatrician

Correct! Lack of speech and startling to noise is an indication of hearing loss. Sending the patient to an audiologist for testing is recommended to identify the underlying cause of hearing loss and implement interventions to prevent further developmental delay.

An advanced practice registered nurse (APRN) examines a three-year-old child, and the child's parent mentions concerns about future sibling rivalry. The family is adopting an infant who will arrive sometime during the next six months. Which recommendation should the APRN offer this parent? Help prepare the child by changing daily routines to desensitize the child to this change Allow the child to help when preparing for the new baby Encourage the child to avoid negative talk about the new baby Enroll the child in multiple activities as a distraction

Correct! Management of sibling rivalry is anticipatory and preventive. Having the older child help prepare for the new baby will reduce the likelihood of a negative response to the event.

During a well-child examination, a 15-year-old female tells you that some of her friends have begun having sex. She has a boyfriend but denies engaging in sex with him. What should you do initially? Find out if she is considering sexual relations. Discuss the risks of sexually transmitted diseases. Give her information about contraception. Ask her how she defines sex.

Correct! Many adolescents do not equate oral or anal intercourse with sex, so it is important to find out how this patient defines sex. The other options also may be considered depending on the situation; however, clarity about the words used in the discussion are most important initially for you to focus the subsequent conversation appropriately.

You are performing a well-child examination on a three-year-old. The child's parent reports that the child has recently begun masturbating. What should you tell this parent? Teach the child about privacy and hand hygiene. Discuss sexuality with the child. Allow the behavior whenever it occurs, since it is normal. Explore whether the child is being abused.

Correct! Masturbation is normal at this age, and children do this because it is pleasurable. Parents should be taught to discuss privacy and hygiene with the child and to encourage the child to limit the activity to a private place. At this age, the behavior is not associated with sexual fantasies, so a discussion of sexuality is not warranted. Masturbation at this age is not usually an indication of abuse.

You are performing a well-child examination on a 15-year-old female who menstruates and consumes a vegan diet. Which nutrients may this adolescent need to supplement? Magnesium, vitamin E, and zinc Calcium, vitamin C, and vitamin A Iron, folic acid, and vitamin B12 Vitamin D, vitamin C, and phosphorus

Correct! Menstruating females are at risk for iron deficiency. Women of childbearing age should take folic acid supplements. Children who eat a vegan diet will need vitamin B12 supplements.

You are offering anticipatory guidance to the parents of a 12-month-old child. The parents are bilingual in Spanish and English and have many Spanish-speaking relatives nearby. They are resisting exposing the child to Spanish out of concern that the child will not learn English well. What should you tell the parents? Children who learn two languages simultaneously often confuse them in conversation. Learning two languages at an early age prevents children from developing a dominant language. Children with multi-language proficiency do not understand that other people are not bilingual. Most bilingual children are able to shift from one language to another when appropriate.

Correct! Most children easily learn two languages and will easily move between the two when learning to talk.

During a newborn wellness visit, the priority is to attend to the concerns of the parent. True False

Correct! Newborns present a challenge to all parents, and understanding how the parents are accepting and caring for their newborn is paramount to the child's growth and development. When you evaluate all aspects of parental understanding regarding care of the newborn, you have the opportunity to educate and intervene as needed. Including the parents in the wellness examination also sets the stage for upcoming wellness visits and ensures an understanding of wellness importance.

An advanced practice registered nurse (APRN) examines a 15-month-old child, and a piece of assessment data causes concern about the child's development. Which finding caused this response by the APRN? Inability to name body parts No attempts at walking Inability to use a spoon Inability to follow two-step commands

Correct! No attempts at walking is a developmental warning sign for a 15-month-old child.

This is not a form; we suggest that you use the browse mode and read all parts of the question carefully. The mother of a 15-month-old toddler tells you that she wishes to continue nursing her child for another year, if possible. What should you recommend to the mother? Offer the breast just prior to meals to maintain milk supply. Clean the toddler's teeth each time after breastfeeding. Breastfeed only at bedtime to establish meal patterns. The toddler should continue to be breastfed on demand.

Correct! One drawback to breastfeeding toddlers is the effect of prolonged contact with lactose on the teeth. Mothers should be cautioned to consult with a dentist and to clean the toddler's teeth after each breastfeeding session. The mother may choose when and how often to breastfeed but should not allow on demand feedings. The breast should be offered after meals.

A child who has been playing in a public park is brought to your clinic with wheezing, vomiting, diarrhea, and drooling. A physical examination reveals a low heart rate and diaphoresis. What could be a cause of these symptoms? Phthalate ingestion Arsenic consumption Organophosphate exposure Lead poisoning

Correct! Organophosphates cause clinical signs of cholinergic excess.

The father of a three-year-old child takes the child to a play group once a week. He expresses concern that the child plays with toys but does not interact with the other toddlers. What counsel should you give the father? Toddlers need more structured play to encourage interaction and socialization with others. The toddler is probably very shy but will outgrow this tendency with repeated exposure to other children. The toddler may have a language delay that interferes with socialization with other children. Toddlers may be interested in other children, but they usually do not engage in interactive play.

Correct! Parallel play is common among toddlers who, although they may be fascinated by other children, generally do not engage with peers in an interactive manner. This does not mean that the child is shy or has a language delay, although in preschool years, the development of symbolic language increases interactive play. Children need both structured and free play, but structured play will not increase interaction during this normally parallel period.

During a well-child examination of a five-year-old child, you assess the child for school readiness. Which finding may be a factor in limiting school readiness for this child? Daily family routines and regular activities The child's ability to recognize four different colors Parental concerns about bullying in the school Two older siblings who attend the same school

Correct! Parental expectation is the strongest predictor of school success in children. Parents who are worried about what may happen in school can transmit this anxiety to the child.

You perform a physical examination on a 12-year-old child and note poor hygiene and inappropriate clothing for the weather. The child's mother appears clean and well dressed. The child reports getting six to seven hours of sleep each night because of texting with friends late into the evening. What should you do to help promote healthy practices for this patient? Reassure the mother that this noncompliance is temporary. Give the child information about sleep and self-care. Discuss setting clear expectations about self-care with the mother. Tell the mother that experimenting with self-care behaviors is normal.

Correct! Parents of school-age children should be advised to set clear limits for their children for cleanliness, healthy exercise, hours of sleep, and other health promotion behaviors. This encourages children to develop responsibility for their health behaviors. Giving the child information can be done along with the parent setting expectations, but, at this age, the parent should still be supervising a child's self-care. Although noncompliance is common for this age and is a means of asserting independence, parents need to discuss this with children to resolve the issue.

While you are discussing anticipatory guidance with the mother of a 12-month-old, the child repeatedly pulls objects out of the mother's purse. Each time, the mother slaps the child's hands as she takes the objects away. What should you recommend for helping the mother manage this child's misbehavior in a developmentally appropriate manner? Place acceptable objects in her purse for the child to find. Keep her purse up high and out of the child's reach. Use time-out each time the child gets into the purse. Say "No!" instead of slapping the child's hands.

Correct! Parents should provide a developmentally appropriate environment to minimize children's misbehavior. Children at this age are naturally curious and will explore the environment, seeking out objects within their reach. It is easier to put the purse out of reach than to repeatedly say, "No." Putting acceptable objects in her purse only reinforces the undesired behavior of getting things out of the mother's purse.

You are discussing sexual maturation with the parents of a 13-year-old female who has Down syndrome. The mother is concerned that starting menstruation will cause her daughter distress and wonders how much sexual information the teen needs at this stage of her life. What should you counsel these parents to do? Use contraceptives to suppress the child's periods, which will lessen the parents' distress Discuss and support healthy sexuality with the child Give the child information about periods but not about sexuality Avoid providing too much information about sexuality so the child does not become confused given the child's cognitive level of understanding

Correct! People with disabilities have the same desires to make decisions and foster fulfilling relationships with others as other people have. Unless healthy sexuality is taught and supported, unhealthy and abusive sexuality is more likely to occur. Parents should give information when it is desired, and they should deliver it in a manner appropriate to the child's level of understanding.

The parents of a four-year-old child are concerned that the child has begun refusing foods the child would normally eat. The child now wants to eat mashed potatoes and chicken strips at every meal and snack. The child's rate of weight has slowed, but the child remains at the same percentile for weight on a growth chart. What should you tell the parents to do? Suggest cutting out snacks to improve the child's appetite at mealtimes Place a variety of nutritious foods on the child's plate at each meal Prepare mashed potatoes and chicken strips for the child at mealtimes Allow the child to choose the foods for meals to improve the child's caloric intake

Correct! Placing different foods on the child's plate offers a variety of choices and the opportunity to develop new tastes and favorite foods. A variety of foods should be offered to encourage improved intake. Allowing the child to pick only certain foods will not meet nutritional guidelines for optimal growth.

The parent of a toddler tells you that the family has adopted a plant-based diet and the child is receiving rice and almond milk instead of cow's milk. What should you counsel the parents about? Excess caloric intake Protein deficiency Excess fat intake Calcium deficiency

Correct! Plant-based milks are low in protein, and young children who consume them are at risk for protein deficiency. These milks are not low in calcium, high in calories, or high in fat.

An advanced practice registered nurse (APRN) examines a 12-month-old patient for a well-child check. The patient's parent mentions that the patient is quieter than her siblings and does not babble or use words. Which action should the APRN take for this patient? Request a psychological evaluation Refer to a neurologist Request an electroencephalography Refer to a speech pathologist

Correct! Primary healthcare providers work with parents to connect them to resources required for potential developmental issues. This behavior pattern must be reviewed by a speech pathologist.

The parents of a three-year-old child ask for help in preventing the child from drawing on walls with crayons. Which discipline recommendation should you provide the parents? Take the crayons away from the child to prevent the behavior. Provide a roll of paper for drawing and teach the child to use it instead. Put the child in time-out each time the child draws on walls. Give the child washable markers so the drawings can be removed easily.

Correct! Providing an appropriate outlet for drawing helps to teach the child where to use the crayons. Discipline involves training or education that molds appropriate behavior and is used to teach the child what is permitted and encouraged.

he parent of a four-year-old child points to a picture and says, "That's your sister." The child responds by saying, "No! It's my baby!" Which type of thinking does this response indicate? Realism Animism Artificialism Egocentrism

Correct! Realism is a concept of thinking that embraces fact and reality. Viewing the sister as the baby embraces the reality of the baby's placement in the family unit and how the parents refer to the sister.

You are examining a child whose parents recently emigrated from a war-torn country. What is a priority assessment when performing the patient history? Determine the parents' English language competency and literacy level Review the child's previous health and illness records Ask about physical, psychological, and emotional trauma Learn about cultural preferences and complementary medicine practices

Correct! Recent history that includes trauma, loss, and refugee camp experience may exacerbate difficulties in adjusting to life in the United States. These experiences can lead to acute and chronic physical and mental health concerns. All the other parts of the patient's history will be necessary, but understanding the patient's trauma should be a priority because the family has escaped a war-torn country.

During a well-child examination on a 13-year-old female, you note that the child is at Tanner stage 3. During the examination, when you initiate a conversation about healthy sexuality education, the parent states that the topic is "off limits." What should you do? Spend private time with the parent to discuss how sexuality education reduces the risk of early sexual intercourse and risky sexual behaviors. Separate the parent from the adolescent to discuss the adolescent's concerns in private. Tell the parent that this information is a routine part of adolescent well-child examinations and must be included. Ask the adolescent whether she wishes to discuss these matters because she is becoming an adult.

Correct! Research has shown that sexuality education leads to a reduction in early onset of sexual intercourse and risky sexual behaviors. It is important for you to be sensitive to the values of the family but also advocate for your patient. The adolescent should be told, especially when she shows an interest in sexual relationships, that she may seek care independently of her parent and that it will remain confidential. When possible, the parent's wishes should be considered, and both the adolescent and the parent should be encouraged to have an open dialogue about these matters.

The parent of an eight-year-old child tells you that the child has begun to ask questions about why a schoolmate has two dads. The parent wonders how to talk to the child about this. What should you recommend that the parent do? Discuss the issue with the child in terms of the parent's religious values and norms Discuss different types of sexual relationships and same-sex partners Explain that not all families are the same and what is most important is that caregivers love and care for their children Tell the child that some adult relationships are complicated and will be better understood when the child is older

Correct! School age is a good time for parents to reinforce the notion that there is diversity in families within which parents and adults love and care for their children. It is not necessary to be explicit, but parents should establish a good history of communication, and explain complex issues to children at the level of the child's understanding. This way the child will know that the parents are accessible and open to discussion of complex and puzzling issues.

During a well-child examination of a school-age child, you learn that the child has been having angry episodes at school. You observe that the child appears withdrawn and sad. Which action should you take? Ask the child and the parent about stressors at home. Suggest that the parent consider moving the child to a different classroom. Provide information about anger management. Make a referral to a child behavioral specialist.

Correct! School-age children are learning to manage emotions and need help in managing their feelings in acceptable ways. A variety of stressors, including parental divorce, substance abuse, bullying in school, and early responsibilities, can cause anxiety in a child. The child may not manage the associated feelings well. Until the underlying cause is better understood, appropriate management options cannot be determined, so referrals to specialists, information about anger management, or moving the child to a different classroom may not be needed.

An advanced practice registered nurse (APRN) is interviewing an adolescent patient who admits to occasionally smoking cigarettes with friends before school. Which action should be taken by this APRN? Present multimedia education to the adolescent's school Use social norm theories and encourage the adolescent to avoid tobacco use Provide education about addiction and emphasize it is easier to stop early Teach skills on how to avoid peer pressure

Correct! Since this patient is already smoking, secondary preventive intervention, with a goal of cessation, is appropriate. Secondary intervention for smoking would include information on the health risks of tobacco use and addiction

An advanced practice registered nurse (APRN) is performing a wellness check for a six-month-old infant with failure to thrive, and the infant's parents ask about the introduction of solids into the infant's diet. Which advice should the APRN provide to these parents? "The use of home-prepared food is preferred." "The use of non-iron-fortified cereals is preferred." "Introduce one food group, wait one week, and then introduce a new food group." "The use of iron-fortified cereals is preferred."

Correct! Six months is an appropriate time to introduce solid foods into the infant's diet. Introduction of solids into an infant's diet should focus on iron-fortified cereals.

The parent of a school-age child who recently had a tonsillectomy is concerned about the child's decreased oral intake. The child's body mass index (BMI) was at the lowest range of normal prior to the surgery, and the parent reports that the child is refusing solid food. What advice should you give the parent to get the child to eat? Puree high-calorie foods such as hamburger and pizza into shakes. Give soda, juice, and other sugary drinks. Encourage calorie-dense foods such as nuts, cheese, and cereal with whole milk. Fortify whole milk by adding peanut butter or instant breakfast mix-ins or use commercially prepared supplement drinks.

Correct! Solid foods may cause too much discomfort, so liquids should be fortified with extra calories while the child is recovering from surgery.

During a well-child assessment of an 18-month-old child, you observe that the child points to a picture of a dog and says, "Want puppy!" What type of speech is this an example of? Telegraphic Receptive Semantic Holographic

Correct! Syntax (the structure of words in sentences or phrases) is developed in stages between the ages of eight months and three and a half years. Telegraphic speech begins at about 18 months of age when children speak in phrases with many words omitted, so that the sentence sounds like a telegram. Holographic speech is the use of a single word to express a complete idea. Receptive speech refers to the ability to understand a word without necessarily being able to use the word. Semantics is the understanding that words have specific meanings.

During a well-child assessment of an 18-month-old child, the child points to a picture of a dog and says, "Want puppy!" What type of speech is this an example of? Semantic Receptive Holophrastic Telegraphic

Correct! Syntax, or the structure of words in sentences or phrases, is developed in stages between the ages of 8 months and 3.5 years. Telegraphic speech begins at about 18 months of age when children speak in phrases with many words omitted so the sentence is short and simple, sounding like a telegram.

The parent of a newborn infant asks you when the parent should intervene to help the infant's future intellectual growth. What should you tell the parent? Preschool is an optimal time to begin general learning. Language and literacy skills begin at birth. Intellectual growth begins when speech develops. Cognitive learning begins during the toddler years.

Correct! Talking to, reading to, playing with, and interacting with a child are some of the most important things a parent can do to enhance and stimulate intellectual growth. Intellectual learning begins long before preschool. Waiting until preschool may produce a delay in the child's development. Children absorb information from birth, and learning occurs before speech. Intellectual development begins at birth and long before toddler years.

The parent of a six-year-old child expresses concern that the child may have attention deficit hyperactivity disorder (ADHD). Which screening tool should you use to evaluate this possibility? Pediatric Symptom Checklist (PSC) Conners Parent and Teacher Rating Scale Behavioral and Emotional Screening System (BESS-2) Behavior Assessment System for Children - Second Edition (BASC-2)

Correct! The Conners Parent and Teacher Rating Scale is used to assess ADHD symptoms in children ages 6 to 18 years old.

An advanced practice registered nurse (APRN) is examining an 11-year-old child for routine health maintenance, and the child's parent mentions being concerned that the child may not be ready to advance in school. The APRN decides to use a screening tool to assess the child's readiness for the next grade. Which screening tool should be used by the APRN for this purpose? CRAFFT SSC HEADSS PEERAMID

Correct! The Pediatric Examination of Educational Readiness at Middle Childhood (PEERAMID) is an educational readiness assessment for neurodevelopmental, behavioral, and health variables.

What has been the result of the Toxic Substances Control Act (TSCA) of 1976? The development of a mechanism to report reactions to toxic chemicals A mandate for corporations to disclose known toxic chemicals The authorization of the Environmental Protection Agency (EPA) to require testing and reporting of some chemicals A requirement that all manufactured chemicals undergo toxicity testing

Correct! The TSCA authorized the EPA to require testing and reporting of some chemicals. However, of the more than 84,000 chemicals registered for use in the United States, only about 200 have been tested and only five have been banned. Corporations have resisted disclosing chemical properties, and the TSCA does not require them to disclose this information.

You are evaluating a five-year-old child who has frequent soiling of stool associated with stomach aches and decreased appetite for the past two months. The parent states that the child has two or fewer formed bowel movements each week and has been using the toilet for approximately two years. Which initial assessment should you make? History of neurogenic conditions Toilet learning history Recent illnesses, fluid intake, and changes in diet Recent adjustments in the family

Correct! The child uses the toilet and has recently developed chronic constipation. The first step is to evaluate recent illnesses or dietary changes that could cause constipation and painful stools, which have resulted in stool withholding. Underlying physiologic or psychological pathology should be explored if simple physiologic causes are not present.

A three-month-old infant is being examined by an advanced practice registered nurse (APRN). The infant was adopted at birth, has gained six ounces since birth, is very fussy, and is inconsolable throughout the day. Which developmental component should be of concern to the APRN for this patient? Physical Fine motor Gross motor Hearing

Correct! The concern is physical development due to lack of weight gain and to fussiness. Poor weight gain is less than 1 lb (0.5 kg) weight gain in one month. Fussiness is also a concern for physical development in a three-month-old.

You are counseling an obese adolescent whose parents both have type 2 diabetes mellitus. Which health behavior prediction model would be useful when discussing lifestyle changes with this patient? Health belief model Health promotion model Transtheoretical model Behavioral change model

Correct! The health belief model explains behavior used to prevent disease rather than to promote health. Patients need to believe that they are vulnerable to the disease, will have negative consequences if they are affected, and can reduce the risk by taking action. An adolescent who believes that there is a risk of developing diabetes may be willing to undertake lifestyle changes if taught that these changes can reduce the risk.

During a well-child assessment of a 13-year-old male, you note small testicles and pubic and axillary hair. You need to further evaluate these findings. What should you ask the patient about? His alcohol and tobacco use His participation in sports Changes in his voice His increase in height and weight

Correct! The initial sign of puberty in males is testicular enlargement. If this does not precede other changes, you should consider whether the child is taking exogenous anabolic steroids, common among those who wish to improve athletic ability. Voice changes and rapid growth may accompany pubic hair development, but the primary concern is anabolic steroid use.

An advanced practice registered nurse (APRN) is evaluating a five-year-old child who is vomiting, bed-wetting, having a poor appetite, crying excessively, and sleeping fewer hours than is typical. Assessment shows a healthy child who has had a two-pound weight loss since the last visit. The father mentions that the mother has been in jail for the last eight weeks, and the child is unable to speak to her. Which differential diagnosis should the APRN make for this child? Preoccupation with death Grief with the loss of a loved one Mourning with persistent anger Persistent sleep problems

Correct! The mother is unavailable to the child, and this is a normal reaction to the loss of a loved one at this age. The problems will go away when the mother returns.

An advanced practice registered nurse (APRN) is seeing a 15-year-old patient, and the patient's parents mention that the patient is lashing out at friends and is withdrawn both at home and in public. The APRN would like to screen the patient for emotional, behavioral, and mental health problems. Which screening tool should the APRN use for this purpose? PSC-Y BITSEA ASQ:SE-2 M-CHAT

Correct! The youth self-report version of the Pediatric Symptom Checklist (PSC-Y) is self-administered or completed by a parent. It is appropriate for children above age 11 and screens for cognitive, emotional, and behavioral problems.

What counsel should you give a new mother who smokes to prevent exposure of smoking-related risks to her nursing newborn? The mother also occasionally smoked throughout her pregnancy. If the mother quits now, her child will not have long-term effects from exposure. Prenatal smoke exposure does not cause respiratory effects after the infant is born. Third-hand smoke exposure risks may last for years even if the mother quits now. Smoking outdoors or near an open window prevents exposure to tobacco smoke.

Correct! Third-hand smoke exposure is contamination that remains after a cigarette is extinguished. It lasts for years because the chemicals from smoke mix and linger with other environmental substances. Prenatal exposure has long-term effects on children, including respiratory effects.

An adolescent has localized bleeding of the gums when brushing his teeth. Your examination of his mouth reveals plaque and calculus on the teeth, but the teeth are not loose. What should you recommend to this patient? Using a xylitol-containing gum after meals Consistently brushing and flossing the teeth twice daily Rinsing the mouth daily with chlorhexidine gluconate Seeing an oral surgeon for treatment

Correct! This adolescent has gingivitis from poor dental hygiene, which is reversible with good hygiene. It is not necessary to refer to an oral surgeon.

A 14-year-old patient presents to an advanced practice registered nurse (APRN) with a concerned parent because the patient has gained a considerable amount of weight since the last checkup and has become withdrawn from family and friends. The patient mentions being embarrassed about the physical change and is unable to lose any weight on her own. Which action should the APRN take to assist this patient? Refer to a mental health specialist Provide a pamphlet on weight loss interventions Prescribe an antidepressant Recommend an Individualized Education Program (IEP)

Correct! This child is presenting with signs of depression, a problem that puts this teenager at risk. Counseling and mental health services are possibly recommended for these cases, hence a referral to this specialist.

An advanced practice registered nurse (APRN) is assessing a 13-year-old child, and the child's parent expresses concerns about the child having emotional problems. Which screening tool should the APRN use to assess this concern? Short Sensory Profile (SSP) Patient Health Questionnaire (PHQ-9) Pediatric Symptom Checklist (PSC) Columbia Impairment Scale (CIS)

Correct! This is a psychosocial screening tool to identify cognitive, emotional, and behavioral problems.

An advanced practice registered nurse (APRN) is caring for a 12-month-old infant brought in by the infant's mother and father for a well-child exam. The parents speak some English but require an interpreter during medical visits. Assessment findings include the following for the infant: • experiences night awakening that persists • does not pick up a toy with one hand • does not look at a caregiver • stands alone momentarily • walks with one hand held by a parent Based on the preliminary data, the APRN acknowledges further assessment is needed. Which standardized screening tool should the APRN use to further assess this child? Short Sensory Profile (SSP) Infant-Toddler and Family Instrument (ITFI) Ages and Stages Questionnaires (ASQ) Temperament and Atypical Behavior Scale (TABS)

Correct! This tool screens and provides surveillance of developmental milestones. It measures overall development and the gross motor, fine motor, and personal-social skills as well as skills in problem-solving and communication.

The parent of a 24-month-old child asks you when toilet learning should begin. What should your response be? "We should assess your child's motor skills." "Begin by reading to your child about toileting." "Tell me about your child's daily habits." "Most children are capable by age two years."

Correct! To assess the parent's understanding of toilet readiness, you should ask the parents about the child's daily habits and routines to see if the child has predictable patterns that can be the basis for toilet learning.

You are performing a well-child examination of a school-age child from a family who recently immigrated from another country. You learn that the child has been involved in many arguments at school. The parents are concerned that their child will never fit in with classmates. How should you address this situation? Recommend counseling to determine underlying causes of this behavior. Suggest that the child may be responding to being bullied by others. Assess the conditions in the country of origin prior to immigration. Stress that this may be a normal response to feeling different at school.

Correct! To better understand why the child arguing, it is important to assess the conditions of the origin country. Is this type of behavior common in the origin country? How is the behavior handled? What was normal behavior before coming to this country?

How should you begin a well-child assessment for an eight-year-old child? Offer age-appropriate information about usual developmental tasks. Ask the child about school, friends, home activities, and sports. Discuss the purpose of the visit and explain the procedures that you will perform. Provide information about healthy nutrition and physical activities.

Correct! To build rapport with the child and parent, begin by asking direct questions to the child and encouraging the child to share information about daily routines. The information you obtain from this discussion will be used in other areas of the examination, including education about nutrition, physical activity, and other age appropriate information.

You are examining a 12-month-old infant who was six weeks premature. You observe that the infant uses a raking motion to pick up small objects. The Parents' Evaluation of Developmental Status (PEDS) questionnaire completed by the parent did not indicate significant developmental delays. What should you do first? Refer the infant to a developmental specialist. Suggest activities to improve fine motor skills. Perform an in-depth developmental assessment. Reassure the parent that this is normal for a premature infant.

Correct! To fully understand the developmental stage of your patient, you must do a full assessment. This may or may not be normal behavior. You will not know for certain without a full evaluation. If abnormalities are found during the full assessment, a referral would be appropriate. A full assessment may reveal other problems or none at all. If no other problems are revealed, you may suggest activities to improve fine motor skills, as appropriate. A full assessment, however, would be the first step.

You are examining a toddler who is below the third percentile for weight even though the parents claim that the child eats constantly. What should you do initially? Refer the child to a feeding evaluation clinic for a swallow study and an evaluation of possible gastroesophageal reflux disease (GERD). Suggest that the parents supplement the child's food intake with a high-calorie formula. Recommend that the parents give a multivitamin and offer high-calorie foods, such as ice cream. Evaluate the child's feeding and elimination behaviors and ask the family to describe mealtime routines.

Correct! When a child is not thriving, you should first evaluate feeding and elimination behaviors and mealtime routines. Doing so will allow you to determine what the child's actual intake is along with the types of foods that are eaten.

The parent of an adolescent female tells you that the child may be the victim of cyberbullying at school but will not discuss it. What should be your initial response? Suggest that the parent discuss this with the school counselor. Reassure the parent that suicide is a rare response to bullying. Ask about the adolescent's school performance and friends. Interview the adolescent separately from the parent.

Correct! When bullying is suspected, it is ideal to interview the youth separately from the parent. It is necessary, however, to get the history from both individuals because children are often reluctant to discuss bullying with their parents.

As you begin a well-child visit on a 12-month-old infant, you observe the child pull himself up using the chair for support and to "cruise." You note the child has a stepping gait in which the whole leg is raised at the hip to ensure adequate ground clearance. What is the appropriate course of action for you to take? Refer the infant to a pediatric orthopedist. Perform a comprehensive history and physical, with special attention to musculoskeletal and neurologic systems. Continue the well visit because most infants outgrow this gait pattern. Send the infant to the ER for immediate evaluation.

Correct! When evaluating an abnormal gait pattern, you should first perform a comprehensive history and physical with special attention to musculoskeletal and neurologic systems. A stepping gait may be a symptom of peripheral neuropathies, spina bifida, or polio.

You see a six-year-old child after a hospitalization for injuries sustained in a motor vehicle accident in which the child's grandfather was killed. The parent states that it is difficult to get the child to stop talking about the accident and is worried that the child will have permanent emotional scars. What should you suggest to the parent? Reassure the child that his grandfather is in a better place Redirect these conversations to happier topics Encourage the child to express and examine his feelings Assure the child that he is safe, and this will not happen again

Correct! When health crises occur, management goals should focus on helping children and families make sense of the events and better understand what is happening to help regain control. Helping the child to express and examine his feelings will help him put what happened in perspective

You are evaluating recurrent stomach pain in a school-age child. The child's examination is normal. You learn that the child reports pain most evenings after school and refuses to participate in sports but does not have nausea or vomiting. The child's grandmother recently had gallbladder surgery. What should you do? Refer the child to a counselor to discuss anxiety about health problems. Encourage the child to keep a log of the pain, stool patterns, and dietary intake. Order radiologic studies and laboratory tests to rule out systemic causes. Reassure the child and encourage resuming sports when symptoms subside.

Correct! You may suspect a somatic disorder after a normal examination and should encourage the child to keep a food or pain diary to help manage symptoms.

Mucocele

he individual should be referred to an oral surgeon for surgical excision (Fig 23.10).

Pericoronitis

inflammation around the crown of a partially erupted tooth

A primary care pediatric nurse practitioner is evaluating a two-year-old with a documented speech delay. The screenings to assess motor skills and cognition are normal, and the child passed a recent hearing test. What should the pediatric nurse practitioner do next? Suggest that the parents purchase age-appropriate music videos Refer the child to an early intervention program Ask the child's parents if they read to the child Give the parents educational materials to encourage speech

vCorrect! Language development requires oral-motor ability, auditory perception, and cognitive ability, which this child has demonstrated. Language also requires a psychosocial-cultural environment to motivate the child to engage in language use. Your initial step should be to determine whether the parents provide such an environment. Educational materials may be used after it is determined that these are useful. Early intervention may be used if the speech delay persists.


Ensembles d'études connexes

Continuous Renal Replacement Therapy (CRRT)

View Set

Psychology: Test 1, Human Growth & Development: Test 2, Human Growth & Development Test 3

View Set

CIN 3103 Wireless Networks FinalExam Review-NoDpNoShAns-v2

View Set

Geometry B, Assignment 10. Circles: Circumference and PI

View Set

Human A&P: Chapter 8- Special Senses Pearson Activities

View Set

Communication Law Exam Chapter #3

View Set